25.05.2018 Views

vector

You also want an ePaper? Increase the reach of your titles

YUMPU automatically turns print PDFs into web optimized ePapers that Google loves.

372 Vectors<br />

5<br />

Vectors<br />

5.1 VECTORS<br />

A <strong>vector</strong> is a quantity having both magnitude and direction such as force, velocity<br />

acceleration, displacement etc.<br />

5.2 ADDITION OF VECTORS<br />

Let a and b be two given <strong>vector</strong>s<br />

OA<br />

= a and AB<br />

= b then <strong>vector</strong> OB<br />

<br />

is called the<br />

sum of a and b .<br />

Symbolically<br />

— — <br />

OA AB = OB — <br />

<br />

a b = OB — <br />

5.3 RECTANGULAR RESOLUTION OF A VECTOR<br />

Let OX, OY, OZ be the three rectangular axes. Let ^i , ^j , ^k be three unit <strong>vector</strong>s and<br />

parallel to three axes.<br />

If OP<br />

= n and the co-ordinates of P be (x, y, z) Z<br />

OA<br />

= x ^i , OB<br />

= y ^j and OC<br />

<br />

= z ^k<br />

(x, y, z)<br />

C<br />

P<br />

<br />

<br />

OP<br />

<br />

= OF FP<br />

OP<br />

<br />

= ( OA AF)<br />

FP<br />

OP<br />

<br />

= OA OB OC<br />

<br />

r<br />

= x ^i + y ^j + z ^k<br />

OP 2 = OF 2 + FP 2<br />

= (OA 2 + AF 2 ) + FP 2 = OA 2 + OB 2 + OC 2 = x 2 + y 2 + z 2<br />

2 2 2<br />

OP = x y z<br />

<br />

2 2 2<br />

| r | = x y z<br />

5.4 UNIT VECTOR<br />

Let a <strong>vector</strong> be x ^i + y ^j + z ^k .<br />

Unit <strong>vector</strong> =<br />

^ ^ ^<br />

xi yj<br />

zk<br />

2 2 2<br />

x y z<br />

372<br />

X<br />

O<br />

<br />

xi<br />

A<br />

O<br />

<br />

k<br />

– r<br />

a + b<br />

a<br />

y j<br />

<br />

j<br />

zk <br />

A<br />

<br />

xi<br />

F<br />

(x, y)<br />

B<br />

b<br />

B<br />

Y


Vectors 373<br />

Example 1. If a and b be two unit <strong>vector</strong>s and be the angle between them, then find<br />

the value of such that a + b is a unit <strong>vector</strong>. (Nagpur, University, Winter 2001)<br />

Solution.Let<br />

<br />

OA = a be a unit <strong>vector</strong> and<br />

be the angle between a and b .<br />

If<br />

<br />

OB = c = a + b is also a unit <strong>vector</strong> then, we have<br />

<br />

| OA | = 1<br />

<br />

| OB | = 1<br />

<br />

| OB | = 1<br />

OAB is an equilateral triangle.<br />

Hence each angle of OAB is 3<br />

5.5 POSITION VECTOR OF A POINT<br />

<br />

AB = b is another unit <strong>vector</strong> and <br />

Ans.<br />

The position <strong>vector</strong> of a point A with respect to origin O is the <strong>vector</strong> OA which is<br />

used to specify the position of A w.r.t. O.<br />

—<br />

To find AB if the position <strong>vector</strong>s of the point A and point B are given.<br />

If the position <strong>vector</strong>s of A and B are a and b . Let the origin be O.<br />

Then<br />

<br />

<br />

= <br />

OA<br />

<br />

a,<br />

OB b<br />

<br />

OA AB = OB<br />

AB<br />

<br />

= OB OA<br />

AB<br />

= b <br />

<br />

O<br />

a<br />

AB<br />

<br />

= Position <strong>vector</strong> of B – Position <strong>vector</strong> of A<br />

Example 2. If A and B are (3, 4, 5) and (6, 8, 9), find AB<br />

Solution.<br />

AB<br />

<br />

.<br />

= Position <strong>vector</strong> of B – Position <strong>vector</strong> of A<br />

= (6 iˆ 8 ˆj 9 kˆ) (3ˆi 4ˆj 5 kˆ)<br />

= 3iˆ<br />

4 ˆj 4 kˆ<br />

Ans.<br />

5.6 RATIO FORMULA<br />

To find the position <strong>vector</strong> of the point which divides the line joining two given<br />

points.<br />

Let A and B be two points and a point C divides AB in the ratio of m : n.<br />

Let O be the origin, then<br />

OA<br />

= <br />

<br />

A<br />

m C n B<br />

a , and OB b, OC ? (a)<br />

(b)<br />

<br />

<br />

OC<br />

= OA AC<br />

<br />

m<br />

m <br />

= OA AB AC<br />

AB <br />

m n m n <br />

m <br />

<br />

= a .( b a ) ( AB b a)<br />

m n<br />

O<br />

a<br />

A<br />

<br />

c<br />

a + b = c<br />

<br />

B (b)<br />

O<br />

b<br />

B<br />

A (a)


374 Vectors<br />

OC<br />

=<br />

<br />

m n<br />

Cor. If m = n = 1, then C will be the mid-point, and<br />

<br />

mb na<br />

OC<br />

a b<br />

=<br />

2<br />

5.7 PRODUCT OF TWO VECTORS<br />

<br />

<br />

The product of two <strong>vector</strong>s results in two different ways, the one is a number and<br />

the other is <strong>vector</strong>. So, there are two types of product of two <strong>vector</strong>s, namely scalar<br />

product and <strong>vector</strong> product. They are written as <br />

a .<br />

<br />

b and a <br />

<br />

b .<br />

5.8 SCALAR, OR DOT PRODUCT<br />

The scalar, or dot product of two <strong>vector</strong>s a and b is defined to be <br />

a<br />

scalar where is the angle between a and b .<br />

Symbolically, <br />

a .<br />

<br />

b = a b cos <br />

<br />

b cos i.e.,<br />

Due to a dot between a and <br />

b this product is also called dot product.<br />

The scalar product is commutative<br />

To Prove.<br />

Proof.<br />

<br />

<br />

<br />

<br />

a . b = b . a<br />

<br />

<br />

b . a = b a<br />

cos ( )<br />

b<br />

B<br />

= <br />

a b cos <br />

0<br />

A<br />

a<br />

= a .<br />

<br />

b Proved.<br />

Geometrical interpretation. The scalar product of two <strong>vector</strong>s is the product of one<br />

<strong>vector</strong> and the length of the projection of the other in the direction of the first.<br />

Let<br />

then<br />

<br />

=<br />

OA<br />

<br />

<br />

<br />

<br />

a and OB b<br />

a . b = (OA) . (OB) cos <br />

ON<br />

= OA . OB . OB<br />

= OA . ON<br />

= (Length of a ) (projection of b along a )<br />

5.9 USEFUL RESULTS<br />

^<br />

^<br />

i . i = (1) (1) cos 0° = 1 Similarly, ^ j .<br />

^<br />

j = 1,<br />

^<br />

^<br />

^<br />

^<br />

k . k = 1<br />

i . j = (1) (1) cos 90° = 0 Similarly, ^ j . k ^<br />

= 0, k . i = 0<br />

Note. If the dot product of two <strong>vector</strong>s is zero then <strong>vector</strong>s are prependicular to each other.<br />

5.10 WORK DONE AS A SCALAR PRODUCT<br />

If a constant force F acting on a particle displaces it from A to B then,<br />

Work done = (component of F along AB). Displacement<br />

= F cos . AB<br />

= <br />

F . AB<br />

Work done = Force . Displacement<br />

0<br />

A<br />

<br />

<br />

^<br />

b<br />

^<br />

a<br />

N<br />

B<br />

F<br />

A<br />

B


Vectors 375<br />

5.11 VECTOR PRODUCT OR CROSS PRODUCT<br />

1.<br />

<br />

The <strong>vector</strong>, or cross product of two <strong>vector</strong>s a<br />

and b is defined to be a <strong>vector</strong> such that<br />

(i) Its magnitude is<br />

<br />

a<br />

<br />

b<br />

sin , where is the<br />

<br />

<br />

<br />

b<br />

angle between a and b .<br />

(ii) Its direction is perpendicular to both <strong>vector</strong>s<br />

<br />

a and b<br />

<br />

.<br />

(iii) It forms with a right handed system.<br />

Let ^<br />

be a unit <strong>vector</strong> perpendicular to both the <strong>vector</strong>s a and b .<br />

<br />

<br />

a b =<br />

2. Useful results<br />

<br />

a<br />

<br />

b<br />

<br />

sin .<br />

<br />

Since ^i , ^j , ^k are three mutually perpendicular unit <strong>vector</strong>s, then<br />

^<br />

^<br />

^ ^ ^ ^<br />

i i = j j k k 0<br />

^<br />

^<br />

i j =<br />

^ ^ ^<br />

j i k<br />

^ ^ ^ ^<br />

j i i j<br />

^ ^ ^<br />

ĵ k ˆ = k j <br />

^ ^ ^ ^<br />

i and k j j k<br />

^ ^ ^<br />

kˆ<br />

i ˆ<br />

^ ^ ^ ^<br />

= i k j i k k i<br />

5.12 VECTOR PRODUCT EXPRESSED AS A DETERMINANT<br />

If<br />

<br />

^ ^ ^<br />

a = a i a j a k<br />

1 2 3<br />

^ ^ ^<br />

b = b1 i b2 j b3<br />

k<br />

<br />

a b =<br />

=<br />

^ ^ ^ ^ ^ ^<br />

1 2 3 1 2 3<br />

( a i a j a k) ( b i b j b k)<br />

^ ^ ^ ^ ^ ^ ^ ^ ^ ^<br />

1 1( ) 1 2( ) 1 3( ) 2 1( ) 2 2 ( )<br />

^ ^ ^ ^ ^ ^ ^ ^<br />

ab 2 3 j k ab 3 1k i ab 3 2 k j ab 3 3 k k<br />

ab i i ab i j ab i k ab j i ab j j<br />

^ ^ ^ ^ ^ ^<br />

1 2 1 3 2 1 2 3 3 1 3 2<br />

= ab k a b j ab k a b i ab j a b i<br />

=<br />

=<br />

^ ^ ^<br />

2 3 3 2 1 3 3 1 1 2 2 1<br />

^ ^ ^<br />

( ab ab) i ( a b a b ) j ( ab ab)<br />

k<br />

i j k<br />

a a a<br />

1 2 3<br />

b b b<br />

1 2 3<br />

5.13 AREA OF PARALLELOGRAM<br />

( ) ( ) ( ) ( )<br />

Example 3. Find the area of a parallelogram whose adjacent sides are i – 2j + 3 k and<br />

2i + j – 4k.<br />

^ ^ ^<br />

i j k<br />

Solution. Vector area of gm = 1 2 3<br />

2 1 4<br />

<br />

<br />

a


376 Vectors<br />

^ ^ ^<br />

^ ^ ^<br />

= (8 3) i ( 4 6) j (1 4) k = 5i 10 j 5k<br />

2 2 2<br />

Area of parallelogram = (5) (10) (5) = 5 6 Ans.<br />

5.14 MOMENT OF A FORCE<br />

O<br />

Let a force F ( PQ ) act at a point P.<br />

Moment of <br />

F about O<br />

= Product of force F and perpendicular<br />

distance (ON. ^<br />

)<br />

<br />

= (PQ) (ON)( ^<br />

) = (PQ) (OP) sin (^<br />

) =<br />

<br />

M r F<br />

5.15 ANGULAR VELOCITY<br />

<br />

<br />

OP PQ<br />

Let a rigid body be rotating about the axis OA with the angular<br />

velocity which is a <strong>vector</strong> and its magnitude is radians per second<br />

and its direction is parallel to the axis of rotation OA.<br />

Let P be any point on the body such that OP = r and<br />

AOP = and AP OA. Let the velocity of P be V.<br />

Let be a unit <strong>vector</strong> perpendicular to and r .<br />

<br />

r = ( r sin ) ^ = ( AP) = (Speed of P) ^<br />

r<br />

N P F<br />

A<br />

Axis<br />

<br />

B<br />

<br />

V<br />

r<br />

Q<br />

P<br />

= Velocity of P to and r<br />

<br />

Hence V = <br />

r<br />

5.16 SCALAR TRIPLE PRODUCT<br />

Let a, b,<br />

<br />

c be three <strong>vector</strong>s then their dot product is written as a .( b <br />

c)or[ <br />

a b c ].<br />

If<br />

<br />

<br />

a =<br />

^ <br />

<br />

^ ^ ^ ^ ^ ^ ^ <br />

^<br />

1 2 3 , 1 2 3 , and 1 2 3<br />

a i a j a k b b i b j b k c c i c j c k<br />

^ ^ ^ ^ ^ ^ ^ ^ ^<br />

1 2 3 1 2 3 1 2 3<br />

a .( b c ) = ( a i a j a k).[( b i b j b k) ( c i c j c k)]<br />

=<br />

^ ^ ^ ^ ^ ^<br />

1 2 3 2 3 3 2 3 1 1 3 1 2<br />

2 1<br />

( a i a j a k).[( bc bc ) i ( b c b c ) j ( bc bc) k]<br />

= a 1<br />

(b 2<br />

c 3<br />

– b 3<br />

c 2<br />

) + a 2<br />

(b 3<br />

c 1<br />

– b 1<br />

c 3<br />

) + a 3<br />

(b 1<br />

c 2<br />

– b 2<br />

c 1<br />

)<br />

=<br />

a a a<br />

1 2 3<br />

b b b<br />

1 2 3<br />

c c c<br />

1 2 3<br />

Similarly, b .( c <br />

a)and c .( a <br />

<br />

b ) have the same value.<br />

<br />

a .( b c ) = b .( c <br />

<br />

a ) = c .( a <br />

<br />

b )<br />

The value of the product depends upon the cyclic order of the <strong>vector</strong>, but is<br />

independent of the position of the dot and cross. These may be interchanged.<br />

The value of the product changes if the order is non-cyclic.<br />

Note.<br />

<br />

a ( b . c) and ( a . b)<br />

c are meaningless.<br />

O


Vectors 377<br />

5.17 GEOMETRICAL INTERPRETATION<br />

The scalar triple product a .( b <br />

<br />

c ) represents the volume of the parallelopiped<br />

having a , b , c as its co-terminous edges.<br />

<br />

a .( b c ) = a .Area of gm OBDC <br />

= Area of gm OBDC × perpendicular distance<br />

A<br />

between the parallel faces OBDC and AEFG. a<br />

–<br />

= Volume of the parallelopiped<br />

Note. (1) If a .( b <br />

<br />

c ) = 0, then a, b,<br />

<br />

c are<br />

coplanar.<br />

1 <br />

(2) Volume of tetrahedron ( )<br />

6 a b c .<br />

Example 4. Find the volume of parallelopiped if<br />

^<br />

^ ^ ^ ^ ^ ^ ^ ^ ^<br />

a 3 i 7 j 5k, b 3i 7j 3k,<br />

and c 7 i 5 j 3k<br />

are the three co-terminous edges of the parallelopiped.<br />

Solution.<br />

Volume = a .( b <br />

<br />

c )<br />

3 7 5<br />

=<br />

3 7 3<br />

7 5 3<br />

= 108 – 210 – 170 = – 272<br />

Volume = 272 cube units.<br />

= – 3 (–21 – 15) – 7 (9 + 21) + 5 (15 – 49)<br />

Example 5. Show that the volume of the tetrahedron having<br />

<br />

Ans.<br />

A B, B C,<br />

C A as<br />

concurrent edges is twice the volume of the tetrahendron having A , B , C <br />

as concurrent edges.<br />

1 <br />

Solution. Volume of tetrahendron = ( ) .[( ) ( )]<br />

6 A B B C C A<br />

1 <br />

= ( ) .[ ]<br />

6 A B B C B A C C C A <br />

[ C C 0]<br />

1 <br />

= ( ) .( )<br />

6 A B B C B A C A<br />

1 <br />

= [ .( ) .( ) .( ) .( ) .( ) .( )]<br />

6 A B C A B A A C A B B C B B A B C A<br />

1 1 <br />

= [ A.( BC) B.( CA)] A.( B<br />

C)<br />

6 3<br />

<br />

= 2 1 [ ]<br />

6 ABC<br />

= 2 Volume of tetrahedron having A , B , C <br />

, as concurrent edges. Proved.<br />

EXERCISE 5.1<br />

1. Find the volume of the parallelopiped with adjacent sides.<br />

<br />

OA = 3 i j, OB j 2 k, and OC i 5 j 4k<br />

extending from the origin of co-ordinates O. Ans. 20<br />

2. Find the volume of the tetrahedron whose vertices are the points A (2, –1, –3), B (4, 1, 3)<br />

C (3, 2, –1) and D (1, 4, 2).<br />

O<br />

n^<br />

–<br />

b<br />

G<br />

B<br />

–<br />

c<br />

C<br />

E<br />

Ans.<br />

F<br />

D<br />

1<br />

7 3


378 Vectors<br />

3. Choose y in order that the <strong>vector</strong>s<br />

^ ^ ^ ^ ^<br />

<br />

a 7 i yj kˆ<br />

, b 3 i 2 j k,<br />

^ ^ ^<br />

c 5 i 3 j k are linearly dependent. Ans. y = 4<br />

4. Prove that<br />

<br />

[ a b, b c, c a] 2[ a b c]<br />

5.18 COPLANARITY QUESTIONS<br />

Example 6. Find the volume of tetrahedron having vertices<br />

^ ^ ^<br />

^ ^ ^<br />

^ ^ ^ ^ ^<br />

( j k), ( 4i 5j qk), ( 3i 9j<br />

4k ) and 4( i j k)<br />

.<br />

Also find the value of q for which these four points are coplanar.<br />

(Nagpur University, Summer 2004, 2003, 2002)<br />

<br />

^ ^ ^ ^ ^ ^ ^ ^ ^ ^ ^<br />

Solution. Let A = j k, B4 i 5 jqk, C 3 i 9 j4 k, D 4( i j k)<br />

AB = <br />

^ ^ ^ ^ ^ ^ ^<br />

B A 4 i 5 j qk ( jkˆ<br />

) 4i 6 j( q<br />

1) k<br />

AC = <br />

^ ^ ^ ^ ^ ^ ^<br />

C A (3 i 9ˆj 4 k) ( jk) 3 i 10 j5<br />

k<br />

AD = ^ ^ ^ ^ ^ ^ ^ ^<br />

D A 4( i jk) ( jk) 4i 5 j<br />

5k<br />

Volume of the tetrahedron = 1 [ AB AC AD]<br />

6<br />

4 6 q 1<br />

1<br />

= 3 10 5 = 1 {4(50 25) 6(15 20) ( q 1)(15 40)}<br />

6<br />

6<br />

4 5 5<br />

= 1 {100 210 55 ( q 1)} = 1 ( 110 55 55 q)<br />

6<br />

6<br />

= 1 ( 5555 q) 55 ( q1)<br />

6 6<br />

If four points A, B, C and D are coplanar, then ( AB AC AD ) = 0<br />

i.e., Volume of the tetrahedron = 0<br />

55<br />

<br />

( q 1) = 0 q = 1 Ans.<br />

6<br />

Example 7. If four points whose position <strong>vector</strong>s are a, b, c,<br />

d are coplanar, show that<br />

<br />

[ a b c] [ a d b] [ a d c] [ d b c ] (Nagpur University, Summer 2005)<br />

Solution. Let A, B, C, D be four points whose position <strong>vector</strong>s are a, b, c,<br />

<br />

d .<br />

<br />

AD = d a, BD d b and CD d <br />

<br />

c<br />

<br />

If AD, BD,<br />

CD are coplanar, then<br />

<br />

<br />

AD .( BD CD ) = 0<br />

<br />

( d a) .[( d b) ( d c )] = 0<br />

<br />

<br />

<br />

( d a) .[ d d d c b d b c ] = 0<br />

<br />

( d a) .[ d c b d b c ] = 0<br />

<br />

d .( d c) d .( b d) d .( b c) a .( d c) a .( b a) a .( b c ) = 0<br />

<br />

0 0 [ dbc] [ ddc] [ dbd] [ abc ] = 0<br />

<br />

<br />

[ abc ] [ abd] [ adc] [ dbc]<br />

Proved.


Vectors 379<br />

EXERCISE 5.2<br />

1. Determine such that<br />

^ ^ ^ ^ ^ ^ ^ ^<br />

a i j k, b 2i 4 k, and c i j 3 k are coplanar. Ans. = 5/3<br />

2. Show that the four points<br />

^ ^ ^ ^ ^ ^ ^ ^ ^ ^ ^ ^<br />

6 i 3 j 2 k,3i 2 j 4 k,5i 7 j 3 k and 13 i 17 j k are coplanar.<br />

3. Find the constant a such that the <strong>vector</strong>s<br />

^ ^ ^ ^ ^ ^ ^ ^ ^<br />

2 i j k, i 2 j 3 k,and 3 i a j 5 k are coplanar. Ans. – 4<br />

4. Prove that four points<br />

^ ^ ^ ^ ^ ^ ^ ^ ^ ^ ^<br />

4 i 5 j k, ( j k), 3 i 9 j 4 k,4( i j k)<br />

are coplanar.<br />

<br />

5. If the <strong>vector</strong>s a, b and c are coplanar, show that<br />

<br />

a b c<br />

<br />

a . a a . b a . c<br />

<br />

b . a b . b b . c<br />

5.19 VECTOR PRODUCT OF THREE VECTORS (A.M.I.E.T.E., Summer, 2004, 2000)<br />

= 0<br />

Let a , b and c be three <strong>vector</strong>s then their <strong>vector</strong> product is written as a ( b × <br />

c ).<br />

Let<br />

<br />

^ ^ ^<br />

a = a i a j a k<br />

1 2 3 ,<br />

^ ^ ^<br />

b = b1 i b2 j b3 k,<br />

^ ^ ^<br />

c = c1 i c2 j c3<br />

k<br />

^ ^ ^ ^ ^ ^ ^ ^ ^<br />

( a1 i a2 j a3 k) ( b1 i b2 j b3 k) ( c1 i c2 j c3<br />

k)<br />

^ ^ ^ ^ ^ ^<br />

= a1 i a2 j a3 k bc 2 3 bc 3 2 i b3c1 b1c3 j bc 1 2<br />

bc 2 1 k<br />

a ( b c ) = <br />

=<br />

( ) [( ) ( ) ( ) ]<br />

^<br />

2 1 2 2 1 3 3 1 1 3 3 2 3 3 2 1 1 2<br />

2 1<br />

[ a ( b c b c ) a ( bc bc )] i [ a ( bc bc ) a ( b c b c )] j<br />

[ a ( b c b c ) a ( bc bc) k]<br />

1 3 1 1 3 2 2 3 3 2<br />

^ ^ ^ ^ ^ ^<br />

= ac 1 1 ac 2 2 ac 3 3 b1 i b2 j b3 k a1 b1 ab 2 2 ab 3 3 c1 i c2 j c3<br />

k<br />

= <br />

( )( ) ( )( )<br />

( a . c) b ( a . b) c .<br />

Ans.<br />

Example 8. Prove that :<br />

<br />

a ( b c) b ( c a) c ( a b ) 0 (Nagpur University, Winter 2008)<br />

Solution. Here, we have<br />

<br />

a ( b c) b ( c a) c ( a b )<br />

= <br />

[( a . c) b ( a . b) c] [( b . a) c ( b . c) a] [( c . b) b ( c . a) b]<br />

= <br />

[( b . a) c ( a . b) c] [( c . b) a ( b . c) a] [( a . c) b ( c . a) b]<br />

<br />

= [( a . b) c ( a . b) c] [( b . c) a ( b . c) a] [( c . a) b ( c . a) b]<br />

= 0 + 0 + 0 = 0 Proved.<br />

Example 9. Prove that :<br />

^ ^ ^ ^ ^ ^ <br />

i ( a i) j ( a j) k ( a k) 2 a (Nagpur University, Winter 2003)<br />

^ ^ ^<br />

Solution. Let a = a i a j a k<br />

1 2 3<br />

^<br />

^


380 Vectors<br />

Now, L.H.S. =<br />

^ ^ ^ ^ ^ ^<br />

i ( a i) j ( a j) k ( a k)<br />

<br />

<br />

= ^ i ( a ^ ^ ^ ^ ^ ^ ^ ^ ^<br />

1 i a2 j a3 k) i j ( a1 i a2 j a3<br />

k)<br />

j<br />

=<br />

<br />

<br />

^ ^ ^ ^ ^<br />

k ( a1 i a2 j a3<br />

k)<br />

k<br />

^ ^ ^ ^ ^ ^ ^ ^ ^ ^ ^ ^ ^ ^ <br />

i a1( i i) a2( j i) a3( k i) j a1( i j) a2( j j) a3<br />

( k<br />

j)<br />

<br />

^ ^ ^ ^ ^ ^ ^ <br />

k a1( i k) a2( j k) a3( k k)<br />

<br />

0 0 0<br />

<br />

<br />

= ^ i a ^ ^ ^ ^ ^ ^ ^ ^<br />

2 k a3 j j a1 k a3 i k a1 j a2<br />

i<br />

=<br />

=<br />

=<br />

^ ^ ^ ^ ^ ^ ^ ^ ^ ^ ^ ^<br />

2 3 1 3 1 2<br />

a ( i k) a ( i j) a ( j k) a ( j i) a ( k j) a ( k i)<br />

^ ^ ^ ^ ^ ^<br />

2 3 1 3 1 2<br />

^ ^ ^<br />

1 2 3<br />

a j a k a i a k a i a j = 2a i 2a j 2a k<br />

^ ^ ^<br />

1 2 3<br />

<br />

2( a i a j a k) 2 a Proved.<br />

Example 10. Show that for any scalar , the <strong>vector</strong>s <br />

x,<br />

<br />

y given by<br />

<br />

<br />

q ( a b) (1 p ) p ( a b )<br />

x a , y a <br />

satisfy the equations<br />

2 2<br />

a<br />

q<br />

a<br />

<br />

px qy aand x y b . (Nagpur University, Winter 2004)<br />

Solution. The given equations are<br />

<br />

<br />

px qy = a ...(1)<br />

<br />

<br />

x y = b ...(2)<br />

Multiplying equation (1) <strong>vector</strong>ially by x , we get<br />

<br />

x ( px qy ) = x <br />

<br />

a<br />

<br />

p ( x x) q ( x y ) = x <br />

<br />

a<br />

<br />

q ( x y ) = x <br />

a,<br />

as x <br />

x 0<br />

<br />

x a = qb ,<br />

[From (2) x <br />

y <br />

<br />

b ] ...(3)<br />

<br />

<br />

Multiplying (3) <strong>vector</strong>ially by a , we have<br />

<br />

a ( x a ) = a q <br />

b<br />

<br />

( a . a) x ( a . x)<br />

a =<br />

2<br />

<br />

a x ( a . x)<br />

a =<br />

<br />

<br />

q ( a b )<br />

<br />

<br />

q ( a b ) <br />

2<br />

<br />

a x =<br />

<br />

( a . x) a q ( a b)<br />

x = <br />

2 2<br />

a a<br />

<br />

( a . x) a q ( a b)


Vectors 381<br />

<br />

x =<br />

a <br />

<br />

<br />

q ( a b )<br />

Substituting the value of x in (1), we get<br />

<br />

<br />

<br />

q ( a b ) <br />

qy = a p a <br />

2 <br />

a <br />

<br />

<br />

<br />

(1 p) a p ( a b )<br />

y = <br />

2<br />

q a<br />

EXERCISE 5.3<br />

1. Show that a ( b a) ( a b)<br />

<br />

a<br />

2. Write the correct answer<br />

(a)<br />

<br />

A B C lies in the plane of<br />

( )<br />

(i)<br />

<br />

A and B (ii) B and<br />

<br />

C (iii)<br />

(b) The value of a .( b c) ( a + b <br />

<br />

c ) is<br />

(i) Zero<br />

(ii)<br />

<br />

[ a , b , c] [ b , c , a ] (iii)<br />

5.20 SCALAR PRODUCT OF FOUR VECTORS<br />

Prove the identity<br />

<br />

( a b) .( c d ) = ( a . c) ( b . d) ( a . d) ( b . c)<br />

Proof.<br />

<br />

( a b) .( c d ) =<br />

= <br />

<br />

<br />

( a b) . r<br />

a<br />

2<br />

where =<br />

<br />

<br />

q ( a b ) <br />

p a qy<br />

2<br />

= a<br />

a <br />

<br />

<br />

<br />

2<br />

<br />

a . x<br />

a<br />

Ans.<br />

<br />

C and A Ans. (ii)<br />

<br />

[ a , b , c ] (iv) None of these<br />

Ans. (ii)<br />

a .( b r)<br />

dot and cross can be interchanged. Put c <br />

d <br />

<br />

r<br />

= <br />

<br />

<br />

a .[ b ( c d )] = a .[( b . d) c ( b . c) <br />

d ]<br />

= <br />

=<br />

( a . c) ( b . d) ( a . d) ( b . c)<br />

<br />

a . c a . d<br />

<br />

b . c b . d<br />

EXERCISE 5.4<br />

Proved.<br />

1. If a 2i 3 j k, b i 2j 4 k, c i j k,find ( a b) .( a <br />

c).<br />

Ans. –74<br />

2. Prove that<br />

<br />

2<br />

<br />

( a b) .( a c) a ( b . c) ( a . b)( a . c ).<br />

5.21 VECTOR PRODUCT OF FOUR VECTORS<br />

Let a, b,<br />

<br />

c and d be four <strong>vector</strong>s then their <strong>vector</strong> product is written as<br />

Now,<br />

<br />

( a b) ( c d)<br />

<br />

( a b) ( c d ) = r ( c <br />

<br />

d )<br />

[Put a <br />

b <br />

<br />

r ]<br />

= <br />

( r . d) c ( r . c)<br />

d


382 Vectors<br />

<br />

=<br />

<br />

[( a b). d] c [( a b) . c]<br />

d<br />

= <br />

[ a b d] c [ a b c]<br />

d<br />

( a b) ( c d)<br />

lies in the plane of c and d . ...(1)<br />

<br />

Again, ( a b) ( c d ) = ( a <br />

b)<br />

<br />

<br />

s [Put = c <br />

d <br />

<br />

s ]<br />

<br />

<br />

= s ( a b ) = ( s . b) a ( s . a)<br />

b<br />

=<br />

<br />

[( c d). b] a [( c d) . a]<br />

b =<br />

<br />

[( b c d] a [ a c d]<br />

b<br />

<br />

( a b) ( c d)<br />

lies in the plane of a and b . ...(2)<br />

<br />

Geometrical interpretation : From (1) and (2) we conclude that ( a b) ( c d ) is<br />

a <strong>vector</strong> parallel to the line of intersection of the plane containing a , <br />

b and plane<br />

containing c , d .<br />

Example 11. Show that<br />

<br />

( B C) ( A D) ( C A) ( B D) ( A B) ( C D) 2 ( A BC)<br />

D<br />

Solution. L.H.S. =<br />

=<br />

=<br />

=<br />

=<br />

<br />

( B C) ( A D) ( C A) ( B D) ( A B) ( C D)<br />

<br />

[( BCD) A ( B CA) D] [( C AD) B ( C AB) D] [( B CD) A ( A CD) B]<br />

<br />

( BCD) A ( B CD) A ( C AD) B ( A CD) B ( B CA) D ( C AB)<br />

D<br />

<br />

( ACD) B ( ACD) B ( AB C) D ( ABC)<br />

D<br />

<br />

<br />

2( AB C)<br />

D = R.H.S.<br />

Show that:<br />

<br />

1. ( b c) ( c a) c ( a b c)<br />

when<br />

2.<br />

3.<br />

4.<br />

5.<br />

6.<br />

7.<br />

8.<br />

EXERCISE 5.5<br />

<br />

2<br />

[ b c, c a, a b)] [ a v c]<br />

<br />

d [ a { b ( c d)}] [( b . d)[ a . ( c d)]<br />

<br />

2<br />

<br />

a [ a [ a ( a b)] a ( b a)<br />

<br />

[( a b) ( a c)] . d ( a . d) [ a b c]<br />

2 2 <br />

2<br />

2 ^ ^ ^<br />

2a a i a j a k<br />

^ ^ ^ ^ ^ ^<br />

a b [( i a). b] i [( j a) . b] j [( k a). b]<br />

k<br />

<br />

( a b c ) stands for scalar triple product.<br />

<br />

p [( a q) ( b r)] q [( a r) ( b p)] r [( a p) ( b q )] 0<br />

Proved.


Vectors 383<br />

5.22 VECTOR FUNCTION<br />

If <strong>vector</strong> r is a function of a scalar variable t, then we write<br />

<br />

<br />

r = r()<br />

t<br />

If a particle is moving along a curved path then the position <strong>vector</strong> <br />

r of the particle is a<br />

function of t. If the component of f (t) along x-axis, y-axis, z-axis are f 1<br />

(t), f 2<br />

(t), f 3<br />

(t) respectively.<br />

Then,<br />

— <br />

f()<br />

t<br />

5.23 DIFFERENTIATION OF VECTORS<br />

<br />

= f1() t i f2() t j f3()<br />

t k<br />

Let O be the origin and P be the position of a moving particle at time t.<br />

—<br />

Let<br />

OP = <br />

r<br />

Let Q be the position of the particle at the time t + t and<br />

the position <strong>vector</strong> of Q is OQ — = r r<br />

<br />

—<br />

<br />

<br />

PQ = — <br />

OQ OP<br />

— <br />

<br />

<br />

= ( r r)<br />

r r<br />

r<br />

is a <strong>vector</strong>. As t 0, Q tends to P and the chord<br />

t<br />

becomes the tangent at P.<br />

<br />

<br />

dr r<br />

We define<br />

=<br />

lim<br />

t<br />

0<br />

dt t , then<br />

<br />

dr<br />

dt<br />

<br />

dr<br />

dt<br />

Similarly,<br />

<br />

is a <strong>vector</strong> in the direction of the tangent at P.<br />

is also called the differential coefficient of r <br />

d<br />

2<br />

<br />

dt<br />

2<br />

r<br />

is the second order derivative of r .<br />

r + r<br />

O<br />

with respect to ‘t’.<br />

dr<br />

d r<br />

gives the velocity of the particle at P, which is along the tangent to its path. Also<br />

dt<br />

2<br />

dt<br />

gives the acceleration of the particle at P.<br />

5.24 FORMULAE OF DIFFERENTIATION<br />

<br />

<br />

(i) d <br />

( F G)<br />

d F dG (ii) d <br />

( F )<br />

d F d<br />

<br />

F (U.P. I semester, Dec. 2005)<br />

dt dt dt dt dt dt<br />

<br />

<br />

d<br />

<br />

dG d F d<br />

<br />

dG d F<br />

<br />

(iii) ( FG . ) F. . G(iv) ( F G)<br />

F G<br />

dt dt dt dt dt dt<br />

<br />

d d a d b d c<br />

(v) [ abc]<br />

bc a c ab<br />

<br />

dt dt <br />

dt <br />

dt<br />

<br />

<br />

d d a d b dc<br />

(vi) [ a ( b c)] ( b c)<br />

a c a b<br />

<br />

dt dt dt dt <br />

The order of the functions <br />

F,<br />

G <br />

is not to be changed.<br />

<br />

<br />

<br />

Q<br />

r<br />

r<br />

Tangent<br />

P (r)<br />

2


384 Vectors<br />

Example 12. A particle moves along the curve<br />

<br />

3 2 2 3<br />

r ( t 4 t) i ( t 4) t j (8t 3 t ) k ,<br />

where t is the time. Find the magnitude of the tangential components of its acceleration<br />

at t = 2.<br />

(Nagpur University, Summer 2005)<br />

Solution. We have, r =<br />

Velocity =<br />

<br />

3 2 2 3<br />

( t 4 t) i ( t 4) t j (8t 3 t ) k<br />

<br />

dr<br />

<br />

2 2<br />

(3t 4) i (2t 4) j (16t 9 t ) k<br />

dt<br />

<br />

At t = 2, Velocity = 8i 8 j 4<br />

k<br />

2 <br />

Acceleration = d r <br />

a = 6ti 2 j (1618)<br />

t k<br />

2<br />

dt<br />

<br />

At t = 2 a 12 i 2 j 20 k<br />

The direction of velocity is along tangent.<br />

So the tangent <strong>vector</strong> is velocity.<br />

Unit tangent <strong>vector</strong>,<br />

Tangential component of acceleration, a t<br />

= aT .<br />

<br />

<br />

<br />

<br />

<br />

v 8i 8 j 4k 8i 8 j 4k 2i 2 j k<br />

T = <br />

| v | 64 64 16<br />

12 3<br />

<br />

2i 2 j k<br />

24 4<br />

20 48<br />

= (12i 2 j 20 k).<br />

= = 16 Ans.<br />

3<br />

3 3<br />

<br />

<br />

Example 13. If da u a and db d <br />

u b then prove that [ a b ] u ( a b )<br />

dt<br />

dt<br />

dt<br />

(M.U. 2009)<br />

Solution. We have,<br />

d <br />

[ a b ] =<br />

dt<br />

<br />

<br />

db da <br />

<br />

a b = a ( u b) ( u a)<br />

b<br />

dt dt<br />

<br />

= a ( u b) b ( u a)<br />

<br />

= ( a. b) u ( a. u) b [( b. a) u ( b. u) a]<br />

(Vector triple product)<br />

<br />

= ( a. b) u ( u. a) b ( a. b) u ( u. b)<br />

a<br />

<br />

= ( u. b) a ( u. a)<br />

b<br />

<br />

= u ( a b )<br />

Proved.<br />

Example 14. Find the angle between the surface x 2 + y 2 + z 2 = 9 and z = x 2 + y 2 – 3 at<br />

(2, –1, 2). (M.D.U. Dec. 2009)<br />

Solution. Here, we have<br />

x 2 + y 2 + z 2 = 9 ...(1)<br />

z = x 2 + y 2 – 3 ...(2)<br />

Normal to (1) 1<br />

= (x 2 + y 2 + z 2 – 9)<br />

=<br />

<br />

<br />

<br />

ˆ <br />

<br />

x y z<br />

ˆ ˆ 2 2 2<br />

i j k ( x y z – 9)<br />

= 2 x î + 2 y ĵ + 2 z ˆk<br />

Normal to (1) at (2, – 1, 2), 1<br />

= 4î – 2 ĵ + 4 ˆk ...(3)


Vectors 385<br />

Normal to (2), 2<br />

= (z – x 2 – y 2 + 3)<br />

=<br />

<br />

<br />

<br />

ˆ <br />

<br />

x y z<br />

ˆ ˆ 2 2<br />

i j k ( z– x – y 3)<br />

= – 2 x î – 2 y ĵ + ˆk<br />

Normal to (2) at (2, – 1, 2), 2<br />

= – 4î + 2 ĵ + ˆk ...(4)<br />

1.<br />

2 = | 1|| 2|cos<br />

<br />

1.<br />

2<br />

(4 iˆ –2ˆj 4 kˆ).(– 4iˆ 2 ˆj kˆ) –16 –44<br />

cos = =<br />

=<br />

| 1|| 2|<br />

|4 iˆ –2ˆj 4 kˆ||– 4iˆ 2 ˆj kˆ|<br />

16 416 16 41<br />

= –16 –8<br />

=<br />

6 21 3 21<br />

–1 –8 <br />

= cos <br />

3 21 <br />

–1 –8 <br />

Hence the angle between (1) and (2) cos <br />

Ans<br />

3 21 <br />

EXERCISE 5.6<br />

2<br />

3<br />

t<br />

t<br />

1. The coordinates of a moving particle are given by x = 4t and y = 3 6 t . Find the<br />

2<br />

6<br />

velocity and acceleration of the particle when t = 2 secs. Ans. 4.47, 2.24<br />

2. A particle moves along the curve<br />

x = 2t 2 , y = t 2 – 4t and z = 3t – 5<br />

where t is the time. Find the components of its velocity and acceleration at time t = 1, in the<br />

<br />

8 14 14<br />

direction i 3 j 2 k.<br />

(Nagpur, Summer 2001) Ans. , <br />

7 7<br />

3. Find the unit tangent and unit normal <strong>vector</strong> at t = 2 on the curve x = t 2 – 1, y = 4t – 3,<br />

z = 2t 2 – 6t where t is any variable.<br />

Ans.<br />

1 1 <br />

(2 i 2 j k), (2i 2 k)<br />

3 3 5<br />

<br />

d<br />

<br />

dG d F <br />

4. Prove that ( F G)<br />

F G<br />

dt dt dt<br />

<br />

5. Find the angle between the tangents to the curve 2 3<br />

r t i 2 t j t k,<br />

at the points t = ± 1.<br />

Ans. 1 9<br />

cos<br />

<br />

<br />

<br />

17<br />

<br />

6. If the surface 5x 2 – 2byz = 9x be orthogonal to the surface 4x 2 y + z 3 = 4 at the point (1, –1, 2)<br />

then b is equal to<br />

(a) 0 (b) 1 (c) 2 (d) 3 (AMIETE, Dec. 2009) Ans. (b)<br />

5.25 SCALAR AND VECTOR POINT FUNCTIONS<br />

Point function. A variable quantity whose value at any point<br />

N<br />

^<br />

in a region of space depends upon the position of the point, is<br />

R<br />

Q<br />

called a point function. There are two types of point functions.<br />

n r<br />

(i) Scalar point function. If to each point P (x, y, z) of a<br />

region R in space there corresponds a unique scalar f (P), then f is<br />

P<br />

called a scalar point function. For example, the temperature<br />

distribution in a heated body, density of a body and potential due to gravity are the examples of<br />

a scalar point function.<br />

(ii) Vector point function. If to each point P (x, y, z) of a region R in space there corresponds<br />

a unique <strong>vector</strong> f (P), then f is called a <strong>vector</strong> point function. The velocity of a moving fluid,<br />

gravitational force are the examples of <strong>vector</strong> point function.<br />

(U.P., I Semester, Winter 2000)<br />

= c<br />

<br />

+ d = c


386 Vectors<br />

Vector Differential Operator Del i.e. <br />

The <strong>vector</strong> differential operator Del is denoted by . It is defined as<br />

<br />

= i j k<br />

x y z<br />

5.26 GRADIENT OF A SCALAR FUNCTION<br />

<br />

If (x, y, z) be a scalar function then i j k is called the gradient of the scalar<br />

x y z<br />

function .<br />

And is denoted by grad .<br />

<br />

Thus, grad = i j k<br />

x y z<br />

<br />

gard = i j k ( x, y, z)<br />

x y z<br />

gard = <br />

5.27 GEOMETRICAL MEANING OF GRADIENT, NORMAL<br />

( is read del or nebla)<br />

(U.P. Ist Semester, Dec 2006)<br />

If a surface (x, y, z) = c passes through a point P. The value of the function at each point<br />

on the surface is the same as at P. Then such a surface is called a level surface through P. For<br />

example, If (x, y, z) represents potential at the point P, then equipotential surface (x, y, z) = c<br />

is a level surface.<br />

Two level surfaces can not intersect.<br />

Let the level surface pass through the point P at which the value of the function is . Consider<br />

another level surface passing through Q, where the value of the function is + d.<br />

Let r and r r be the position <strong>vector</strong> of P and Q then PQ<br />

— <br />

r<br />

.dr =<br />

<br />

i j k <br />

.( idx jdy kdz <br />

<br />

)<br />

x y z<br />

=<br />

<br />

dx dy dz d<br />

x y z<br />

If Q lies on the level surface of P, then d = 0<br />

Equation (1) becomes . dr = 0. Then is to dr (tangent).<br />

Hence, is normal to the surface (x, y, z) = c<br />

Let = || N , where N is a unit normal <strong>vector</strong>. Let n be the perpendicular distance<br />

between two level surfaces through P and R. Then the rate of change of in the direction of the<br />

normal to the surface through P is<br />

<br />

.<br />

n<br />

...(1)<br />

d = lim lim<br />

dn n<br />

0 n<br />

n<br />

0<br />

=<br />

=<br />

| | Ndr .<br />

lim<br />

n<br />

n 0<br />

n<br />

0<br />

<br />

<br />

.<br />

dr<br />

n<br />

<br />

| | n<br />

lim | |<br />

n<br />

<br />

<br />

N . r | N || r<br />

|cos <br />

<br />

<br />

<br />

| r|cos<br />

n


Vectors 387<br />

<br />

|| =<br />

n<br />

Hence, gradient is a <strong>vector</strong> normal to the surface = c and has a magnitude equal to the<br />

rate of change of along this normal.<br />

5.28 NORMAL AND DIRECTIONAL DERIVATIVE<br />

(i) Normal. If (x, y, z) = c represents a family of surfaces for different values of the constant<br />

c. On differentiating , we get d = 0<br />

But d = .<br />

<br />

dr so . d r = 0<br />

The scalar product of two <strong>vector</strong>s and dr being zero, and<br />

each other. dr <br />

is in the direction of tangent to the given surface.<br />

Thus is a <strong>vector</strong> normal to the surface (x, y, z) = c.<br />

<br />

<br />

dr are perpendicular to<br />

(ii) Directional derivative. The component of in the direction of a <strong>vector</strong> d is equal to<br />

.d<br />

and is called the directional derivative of in the direction of d .<br />

<br />

r<br />

<br />

r<br />

lim<br />

=<br />

r<br />

0<br />

<br />

r<br />

where, r = PQ<br />

is called the directional derivative of at P in the direction of PQ.<br />

Let a unit <strong>vector</strong> along PQ be N .<br />

Now<br />

n<br />

r<br />

<br />

r<br />

n<br />

n<br />

= cos r = cos <br />

<br />

N.<br />

N<br />

<br />

lim N.<br />

N<br />

<br />

= r<br />

0<br />

<br />

<br />

n<br />

n<br />

<br />

NN<br />

.<br />

<br />

<br />

<br />

<br />

<br />

...(1)<br />

<br />

n<br />

<br />

From (1), r<br />

<br />

<br />

N.<br />

N<br />

<br />

<br />

<br />

<br />

<br />

= N . N | | = N . ( N | | )<br />

<br />

Hence,<br />

r<br />

, directional derivative is the component of in the direction N .<br />

<br />

= N . | |cos | |<br />

r<br />

Hence, is the maximum rate of change of .<br />

Example 15. For the <strong>vector</strong> field (i)<br />

<br />

A miˆ<br />

and (ii) A m r . Find . <br />

A and <br />

A .<br />

Draw the sketch in each case. (Gujarat, I Semester, Jan. 2009)<br />

<br />

<br />

Solution. (i) Vector A mi is represented in the figure (i).<br />

(ii) A = mr is represented in the figure (ii).<br />

(iii)<br />

. <br />

<br />

<br />

A = i j k .( xi y j zk) 1113<br />

x y z<br />

. <br />

A = 3 is represented on the number line at 3.<br />

(iv)<br />

<br />

A = i j k ( xi y j zk)<br />

x y z


388 Vectors<br />

<br />

i j k<br />

=<br />

<br />

x y z<br />

x y z<br />

are represented in the adjoining figure.<br />

^<br />

i<br />

^<br />

k<br />

^<br />

m . i<br />

k<br />

r<br />

<br />

j<br />

O<br />

O<br />

Number line<br />

j 0 1 2 3 O<br />

i<br />

i<br />

(i) (ii) (iii) (iv)<br />

m r<br />

Example 16. If = 3x 2 y – y 3 z 2 ; find grad at the point (1, –2, –1).<br />

(AMIETE, June 2009, U.P., I Semester, Dec. 2006)<br />

Solution. grad = <br />

2 3 2<br />

= i j k (3 x y y z )<br />

x y z<br />

<br />

2 3 2 <br />

2 3 2 2 3 2<br />

= i (3 x y y z ) j (3 x y yz) k (3 x y y z )<br />

x y z<br />

=<br />

= 0<br />

<br />

2 2 2 3<br />

i (6 xy) j (3x 3 y z ) k ( 2 yz)<br />

<br />

grad at (1, –2, –1) = i (6) (1) ( 2) j[(3) (1) 3(4) (1)] k ( 2)( 8) ( 1)<br />

<br />

= 12 i 9 j 16 k<br />

Ans.<br />

Example 17. If u = x + y + z, v = x 2 + y 2 + z 2 , w = yz + zx + xy prove that grad u,<br />

grad v and grad w are coplanar <strong>vector</strong>s. [U.P., I Semester, 2001]<br />

Solution. We have,<br />

grad u =<br />

<br />

<br />

i j k ( x y z)<br />

i j k<br />

x y z<br />

grad v =<br />

<br />

<br />

2 2 2<br />

i j k ( x y z ) 2xi 2yj<br />

2zk<br />

x y z<br />

grad w =<br />

<br />

<br />

i j k ( yz zx xy) i( z y) jz ( x) k( y x)<br />

x y z<br />

[For <strong>vector</strong>s to be coplanar, their scalar triple product is 0]<br />

Now, grad u.(grad v × grad w) =<br />

=<br />

=<br />

1 1 1 1 1 1<br />

2x 2y 2z 2 x y z<br />

z y z x y x z y z x y x<br />

1 1 1<br />

2 x y z x y z x y z<br />

z y z x y x<br />

1 1 1<br />

2( x y z) 1 1 1 0<br />

y z z x x y<br />

k<br />

[Applying R 2<br />

R 2<br />

+ R 3<br />

]<br />

j


Vectors 389<br />

Since the scalar product of grad u, grad v and grad w are zero, hence these <strong>vector</strong>s are<br />

coplanar <strong>vector</strong>s.<br />

Proved.<br />

Example 18. Find the directional derivative of x 2 y 2 z 2 at the point (1, 1, –1) in the direction<br />

of the tangent to the curve x = e t , y = sin 2t + 1, z = 1 – cos t at t = 0.<br />

(Nagpur University, Summer 2005)<br />

Solution. Let = x 2 y 2 z 2<br />

Directional Derivative of <br />

2 2 2<br />

= = i j k ( x y z )<br />

x y z<br />

<br />

2 2 2 2 2 2<br />

= 2xy z i 2yx z j 2zx y k<br />

Directional Derivative of at (1, 1, –1)<br />

Tangent <strong>vector</strong>,<br />

=<br />

Tangent(at t = 0) =<br />

<br />

2 2 2 2 2 2<br />

2(1)(1) ( 1) i 2(1)(1) ( 1) j 2( 1)(1) (1) k<br />

<br />

= 2 i 2 j 2k<br />

...(1)<br />

t<br />

<br />

<br />

r<br />

= xi yj zk e i (sin 2t 1) j (1<br />

cos t)<br />

k<br />

<br />

T dr <br />

t<br />

= e i 2cos 2tj<br />

sin tk<br />

dt<br />

<br />

0<br />

e i 2(cos 0) j (sin 0) k i 2 j<br />

...(2)<br />

( i 2 j)<br />

Required directional derivative along tangent = (2 i 2 j 2 k)<br />

1<br />

4<br />

[From (1), (2)]<br />

= 2 4 0 6 Ans.<br />

5 5<br />

Example 19. Find the unit normal to the surface xy 3 z 2 = 4 at (–1, –1, 2). (M.U. 2008)<br />

Solution. Let (x, y, z) = xy 3 z 2 = 4<br />

We know that is the <strong>vector</strong> normal to the surface (x, y, z) = c.<br />

<br />

Normal <strong>vector</strong> = = i <br />

x j <br />

y k<br />

<br />

<br />

z<br />

Now =<br />

Normal <strong>vector</strong> =<br />

<br />

i ( xyz) j ( xyz) k ( xyz)<br />

x y z<br />

<br />

3 2 3 2 3 2<br />

<br />

3 2 2 2 3<br />

yz i 3xy z j 2xyzk<br />

<br />

Normal <strong>vector</strong> at (–1, –1, 2) = 4i 12 j 4k<br />

Unit <strong>vector</strong> normal to the surface at (–1, –1, 2).<br />

<br />

4i 12 j 4k<br />

1 <br />

= ( i 3 j k ) Ans.<br />

| | 16 144 16 11<br />

Example 20. Find the rate of change of = xyz in the direction normal to the surface<br />

x 2 y + y 2 x + yz 2 = 3 at the point (1, 1, 1). (Nagpur University, Summer 2001)<br />

Solution. Rate of change of =<br />

<br />

<br />

= i j k ( x yz)<br />

iyz jxz kxy<br />

x y z


390 Vectors<br />

<br />

Rate of change of at (1, 1, 1) = ( i j k)<br />

Normal to the surface = x 2 y + y 2 x + yz 2 – 3 is given as -<br />

=<br />

=<br />

() (1, 1, 1)<br />

= 3i 4 j 2 k<br />

2 2 2<br />

i j k ( x y y x yz 3)<br />

x y z<br />

<br />

2 2 2<br />

i(2 xy y ) jx ( 2 xy z ) k2yz<br />

<br />

<br />

Unit normal = 3 i 4 j 2 k<br />

916<br />

4<br />

<br />

Required rate of change of =<br />

(3 i 4 j 2 k)<br />

( i j k).<br />

= 3 4 2 9 <br />

916<br />

4 29 29<br />

Ans.<br />

Example 21. Find the constants m and n such that the surface m x 2 – 2nyz = (m + 4)x will<br />

be orthogonal to the surface 4x 2 y + z 3 = 4 at the point (1, –1, 2).<br />

(M.D.U. Dec. 2009, Nagpur University, Summer 2002)<br />

Solution. The point P (1, –1, 2) lies on both surfaces. As this point lies in<br />

mx 2 – 2nyz = (m + 4)x, so we have<br />

m – 2n (–2) = (m + 4)<br />

m + 4n = m + 4 n = 1<br />

Let 1<br />

= mx 2 – 2yz – (m + 4)x and 2<br />

= 4x 2 y + z 3 – 4<br />

Normal to 1<br />

= 1<br />

=<br />

2<br />

i j k [ mx 2 yz ( m<br />

4) x]<br />

x y z<br />

<br />

= i(2mx m 4) 2zj2yk<br />

Normal to 1<br />

at (1, –1, 2) =<br />

<br />

<br />

i(2m m 4) 4 j 2k<br />

= ( m 4) i 4 j 2 k<br />

Normal to 2<br />

= 2<br />

=<br />

2 3<br />

<br />

i j k (4x y z 4)<br />

2 2<br />

= i8xy 4x j 3z k<br />

x y z<br />

<br />

Normal to 2<br />

at (1, –1, 2) = – 8i 4 j 12<br />

k<br />

Sinec 1<br />

and 2<br />

are orthogonal, then normals are perpendicular to each other.<br />

1<br />

. 2<br />

= 0<br />

<br />

[( m 4) i 4 j 2 k].[ 8i 4 j 12 k]<br />

= 0<br />

– 8 (m – 4) – 16 + 24 = 0<br />

m – 4 = –2 + 3 m = 5<br />

Hence m = 5, n = 1<br />

Ans.<br />

Example 22. Find the values of constants and so that the surfaces x 2 – yz = (+ 2) x,<br />

4x 2 y + z 3 = 4 intersect orthogonally at the point (1, – 1, 2).<br />

(AMIETE, II Sem., Dec. 2010, June 2009)<br />

Solution. Here, we have<br />

x 2 – yz = ( + 2) x ...(1)<br />

4x 2 y + z 3 = 4 ...(2)


Vectors 391<br />

<br />

Normal to the surface (1), λ x yz(λ2)<br />

x<br />

<br />

<br />

<br />

ˆ<br />

<br />

ˆ<br />

ˆ 2<br />

i j k x yz ( 2) x<br />

<br />

x y z<br />

<br />

iˆ(2x2) ˆj ( z) kˆ<br />

( y)<br />

Normal at (1, –1, 2) = î (2 – – 2) – ĵ (–2) + ˆk ...(3)<br />

Normal at the surface (2)<br />

= î ( – 2) + ĵ z (2) + ˆk <br />

<br />

ˆ<br />

<br />

ˆ<br />

ˆ 2 3<br />

i j k (4x yz<br />

4)<br />

x y z<br />

= î (8 × y) + ĵ (4x2 ) + ˆk (3z 2 )<br />

Normal at the point (1, –1, 2) = – 8 î + 4 ĵ + 12 ˆk ...(4)<br />

Since (3) and (4) are orthogonal so<br />

iˆ( 2) ˆj(2 ) kˆ. 8iˆ4ˆj12kˆ<br />

0<br />

<br />

8( 2) 4(2 ) 12 0 8 168120<br />

8 20160<br />

4( 2 54) 0<br />

2 540<br />

25 4<br />

...(5)<br />

Point (1, – 1, 2) will satisfy (1)<br />

<br />

2<br />

(1) ( 1)(2) ( 2)(1) + 2 = + 2 = 1<br />

Putting = 1 in (5), we get<br />

9<br />

25 4 <br />

2<br />

9<br />

Hence and =1<br />

Ans.<br />

2<br />

Example 23. Find the angle between the surfaces x 2 + y 2 + z 2 = 9 and z = x 2 + y 2 – 3 at<br />

the point (2, –1, 2). (Nagpur University, Summer 2002)<br />

Solution. Normal on the surface (x 2 + y 2 + z 2 – 9 = 0)<br />

=<br />

<br />

<br />

2 2 2<br />

i j k ( x y z 9) (2xi 2yj<br />

2 zk)<br />

x y z<br />

<br />

Normal at the point (2, –1, 2) = 4i 2 j 4 k<br />

...(1)<br />

Normal on the surface (z = x 2 + y 2 2 2<br />

– 3) = i j k ( x y z 3)<br />

x y z<br />

<br />

= 2xi 2yj<br />

k<br />

<br />

Normal at the point (2, –1, 2) = 4i 2<br />

j k<br />

...(2)<br />

Let be the angle between normals (1) and (2).<br />

<br />

(4 i 2 j 4 k).(4 i 2 j k)<br />

= 16 416 16 41cos<br />

<br />

16 + 4 – 4 = 6 21cos 16 = 6 21cos


392 Vectors<br />

cos =<br />

8<br />

3 21<br />

=<br />

cos<br />

1 8<br />

3 21<br />

Example 24. Find the directional derivative of 1 r in the direction r where r xi y j zk .<br />

Solution. Here, (x, y, z) =<br />

Now<br />

=<br />

1<br />

<br />

<br />

=<br />

r<br />

<br />

=<br />

Ans.<br />

<br />

(Nagpur University, Summer 2004, U.P., I Semester, Winter 2005, 2002)<br />

1<br />

<br />

2 2 2 2<br />

1 1<br />

( x y z )<br />

r 2 2 2<br />

x y z<br />

1<br />

<br />

<br />

2 2 2 2<br />

<br />

i j k ( x y z )<br />

x y z<br />

<br />

<br />

<br />

1 1 1<br />

<br />

2 2 2 2 2 2 2 2 2 2 2 2<br />

<br />

( x y z ) i ( x y z ) j ( x y z ) k<br />

x y z<br />

<br />

3 3<br />

1 <br />

<br />

2 2 2 1<br />

<br />

<br />

3<br />

2 2 2 2<br />

1<br />

( x y z ) 2 xi ( x y z )<br />

2<br />

2y<br />

j (<br />

2 2 2 )<br />

2 2<br />

2 2<br />

2 x y z <br />

z <br />

<br />

<br />

<br />

k<br />

<br />

( xi yj<br />

zk)<br />

= 2 2 2 3/2<br />

( x y z )<br />

<br />

and r = unit <strong>vector</strong> in the direction of xi yj<br />

zk<br />

<br />

xi yj<br />

zk<br />

=<br />

2 2 2<br />

x y z<br />

So, the required directional derivative<br />

=<br />

<br />

2 2 2<br />

xi yj zk xi yj zk x y z<br />

. r .<br />

<br />

( x y z ) ( x y z ) ( x y z )<br />

1 <br />

1<br />

x y z r<br />

=<br />

2 2 2 2<br />

2 2 2 3/2 2 2 2 1/2 2 2 2 2<br />

[From (1), (2)]<br />

Example 25. Find the direction in which the directional derivative of (x, y) =<br />

x<br />

2 2<br />

y<br />

xy<br />

...(1)<br />

...(2)<br />

(1, 1) is zero and hence find out component of velocity of the <strong>vector</strong> 3 2<br />

r ( t 1)<br />

i t j in<br />

the same direction at t = 1. (Nagpur University, Winter 2000)<br />

2 2<br />

x y <br />

Solution. Directional derivative = = i j k <br />

x y z xy <br />

<br />

2 2 2 2<br />

=<br />

xy .2 x ( x y ) y xy .2 y x ( y x<br />

i<br />

j<br />

) <br />

2 2 2 2 <br />

x y x y <br />

2 3 2 3<br />

=<br />

x y y i<br />

j<br />

xy x <br />

2 2 2 2 <br />

x y x y <br />

<br />

<br />

Directional Derivative at (1, 1) = i 0 j 0 0<br />

Since () (1, 1)<br />

= 0, the directional derivative of at (1, 1) is zero in any direction.<br />

Again r =<br />

<br />

3 2<br />

( t 1)<br />

i t<br />

j<br />

<br />

Ans.<br />

<br />

at


Vectors 393<br />

Velocity, v =<br />

dr <br />

2<br />

3t i 2tj<br />

dt<br />

<br />

<br />

Velocity at t = 1 is = 3i<br />

2 j<br />

The component of velocity in the same direction of velocity<br />

<br />

3i<br />

2 j<br />

9<br />

4<br />

=<br />

(3 i 2 j). 13<br />

9<br />

4 13<br />

Ans.<br />

<br />

Example 26. Find the directional derivative of (x, y, z) = x 2 y z + 4 x z 2 at (1, –2, 1) in<br />

<br />

the direction of 2i j 2 k . Find the greatest rate of increase of .<br />

(Uttarakhand, I Semester, Dec. 2006)<br />

Solution. Here, (x, y, z) = x 2 y z + 4xz 2<br />

Now, =<br />

=<br />

at (1, – 2, 1) =<br />

2 2<br />

i j k ( x yz 4 xz )<br />

x y z<br />

<br />

2 2 2<br />

(2xyz 4 z ) i ( x z) j ( x y 8 xz)<br />

k<br />

2<br />

<br />

{2(1) ( 2)(1) 4(1) } i (11) j {1( 2) 8(1)(1)} k<br />

<br />

= ( 4 4) i j ( 2 8) k = j 6 k<br />

<br />

Let a = unit <strong>vector</strong> = 2 i j 2 k 1 <br />

(2 i j 2 k)<br />

41<br />

4 3<br />

So, the required directional derivative at (1, –2, 1)<br />

1 <br />

= . a ( j 6 k). (2 i j 2 k)<br />

= 1 <br />

( 112)<br />

<br />

13<br />

3<br />

3 3<br />

Greatest rate of increase of = j 6k<br />

= 1<br />

36<br />

<br />

<br />

= 37 Ans.<br />

Example 27. Find the directional derivative of the function = x 2 – y 2 + 2z 2 at the point P<br />

(1, 2, 3) in the direction of the line PQ where Q is the point (5, 0, 4).<br />

(AMIETE, Dec. 20010, Nagpur University, Summer 2008, U.P., I Sem., Winter 2000)<br />

Solution. Directional derivative = <br />

=<br />

<br />

i j k ( x y 2 z ) 2xi 2yj<br />

4zk<br />

x y z<br />

<br />

2 2 2<br />

Directional Derivative at the point P (1, 2, 3) = 2i 4 j 12 k<br />

...(1)<br />

<br />

<br />

PQ = Q P = (5, 0, 4) – (1, 2, 3) = (4, –2, 1) ...(2)<br />

<br />

(4 i 2 j k)<br />

Directional Derivative along PQ = (2 i 4 j 12 k).<br />

[From (1) and (2)]<br />

16 41<br />

=<br />

8812 28<br />

<br />

21 21<br />

Ans.<br />

x<br />

Example 28. For the function (x, y) = 2 2 , find the magnitude of the directional<br />

x y<br />

derivative along a line making an angle 30° with the positive x-axis at (0, 2).<br />

(A.M.I.E.T.E., Winter 2002)


394 Vectors<br />

Solution. Directional derivative = <br />

<br />

x<br />

x y z<br />

x y<br />

<br />

= i j k 2 2<br />

<br />

2 2<br />

y x 2xy<br />

= i j<br />

2 2 2 2 2 2<br />

( x y ) ( x y )<br />

Directional derivative at the point (0, 2)<br />

4<br />

0 2(0) (2) i<br />

<br />

(0 4) (0 4) 4<br />

<br />

<br />

= i j<br />

2 2<br />

<br />

<br />

1 x(2 x) x(2 y)<br />

<br />

<br />

= i<br />

<br />

j<br />

2 2 2 2 2 <br />

2 2 2<br />

x y ( x y ) ( x y )<br />

Directional derivative at the point (0, 2) in the direction CA — 3 1 <br />

i.e. i j<br />

2 2 <br />

<br />

<br />

<br />

CA OB BA i cos 30 j sin 30<br />

i 3 1 <br />

<br />

<br />

= . i j<br />

<br />

3 1 <br />

<br />

4 2 2 <br />

<br />

<br />

i j<br />

<br />

<br />

2 2 <br />

3<br />

=<br />

Ans.<br />

8<br />

<br />

Example 29. Find the directional derivative of V where 2 2 2<br />

V xy i zy j xz k, at the<br />

point (2, 0, 3) in the direction of the outward normal to the sphere x 2 + y 2 + z 2 = 14 at the<br />

point (3, 2, 1). (A.M.I.E.T.E., Dec. 2007)<br />

<br />

Solution. V 2 = VV .<br />

<br />

2 2 2 2 2 2<br />

2<br />

,<br />

<br />

= ( xy i zy j xz k).( x y i zy j xz k)<br />

= x 2 y 4 + z 2 y 4 + x 2 z 4<br />

Directional derivative = 2<br />

V<br />

=<br />

2 4 2 4 2 4<br />

i j k ( x y z y x z )<br />

x y z<br />

=<br />

<br />

4 4 2 3 3 2 4 2 3<br />

(2xy 2 xz ) i (4x y 4 y z ) j (2y z 4 xz)<br />

k<br />

<br />

Directional derivative at (2, 0, 3) = (0 2281) i (0 0) j (0 4427)<br />

k<br />

<br />

= 324 i 432k 108 (3 i 4 k)<br />

...(1)<br />

Normal to x 2 + y 2 + z 2 – 14 = <br />

2 2 2<br />

= i j k ( x y z 14)<br />

x y z<br />

<br />

= (2xi 2yj<br />

2 zk)<br />

<br />

Normal <strong>vector</strong> at (3, 2, 1) = 6i 4 j 2 k<br />

...(2)<br />

Unit normal <strong>vector</strong> = 6 4 2 2(3 2 ) 3 <br />

i j k i j k i 2<br />

<br />

j k<br />

<br />

<br />

36 16 4 2 14 14<br />

<br />

3i 2 j k<br />

Directional derivative along the normal = 108(3 i 4 k). .<br />

14<br />

108 (9 4) 1404<br />

= <br />

14 14<br />

j<br />

—2<br />

1<br />

(0, 2) 30°<br />

i<br />

C —2<br />

3<br />

i<br />

1<br />

A<br />

[From (1), (2)]<br />

j<br />

Ans.


Vectors 395<br />

Example 30. Find the directional derivative of ( f) at the point (1, – 2, 1) in the direction<br />

of the normal to the surface xy 2 z = 3x + z 2 , where f = 2x 3 y 2 z 4 . (U.P., I Semester, Dec 2008)<br />

Solution. Here, we have<br />

f = 2x 3 y 2 z 4<br />

<br />

3 2 4<br />

f = i j k <br />

(2 x y z ) 2 2 4 3 4<br />

<br />

x y z<br />

= 3 2 3<br />

6 x y z i 4 xyz j 8x y z k<br />

<br />

<br />

<br />

2 2 4 3 4 3 2 3<br />

(f) = i j k (6x y z <br />

<br />

i 4x yz j 8 x y z k)<br />

x y z<br />

<br />

<br />

<br />

= 12xy 2 z 4 + 4x 3 z 4 + 24x 3 y 2 z 2<br />

Directional derivative of ( f )<br />

<br />

2 4 3 4 3 2 2<br />

= i j k (12xy z 4xz 24 x y z )<br />

x y z<br />

<br />

<br />

<br />

2 4 2 4 2 2 2<br />

= (12y z 12x z 72 x y z ) i (24xyz4<br />

48x3yz2)<br />

j<br />

+ (48xy 2 z 3 + 16x 3 z 3 + 48x 3 y 2 z) k<br />

Directional derivative at (1, – 2, 1) = (48 + 12 + 288) i + (– 48 – 96) j + (192 + 16 + 192) k <br />

= 348 i<br />

– 144 j 400k<br />

Normal to(xy 2 z – 3x – z 2 ) = (xy 2 z – 3x – z 2 )<br />

<br />

2 2<br />

= i j k <br />

( xy z –3 x – z )<br />

x y z<br />

<br />

<br />

<br />

2<br />

= 2<br />

( y z – 3) i (2 xyz) j ( xy – 2 z)<br />

k<br />

Normal at(1, – 2, 1) = i<br />

– 4 j 2k<br />

i –4j 2k<br />

Unit Normal Vector =<br />

= 1 ( – 4 2 )<br />

116<br />

4 21 i j k<br />

Directional derivative in the direction of normal<br />

1 <br />

= (348 i –144 j 400 k) ( i –4j 2 k)<br />

21<br />

1<br />

= (348 576 800) = 1724<br />

Ans.<br />

21<br />

21<br />

Example 31. If the directional derivative of = a x 2 y + b y 2 z + c z 2 x at the point<br />

x 1 y 3<br />

z<br />

(1, 1, 1) has maximum magnitude 15 in the direction parallel to the line ,<br />

2 2 1<br />

find the values of a, b and c. (U.P. I semester, Winter 2001)<br />

Solution. Given = a x 2 y + b y 2 z + c z 2 x<br />

<br />

= i j k <br />

x y z (a x2 y + b y 2 z + c z 2 x)<br />

=<br />

<br />

2 2 2<br />

i(2 axy cz ) jax ( 2 byz) kby ( 2 czx)<br />

<br />

at the point (1, 1, 1) = i(2 a c) j( a 2 b) kb ( 2 c)<br />

...(1)<br />

We know that the maximum value of the directional derivative is in the direction of . <br />

i.e. || = 15 (2a + c) 2 + (2b + a) 2 + (2c + b) 2 = (15) 2<br />

But, the directional derivative is given to be maximum parallel to the line


396 Vectors<br />

x 1 y 3 z<br />

= <br />

<br />

i.e., parallel to the <strong>vector</strong><br />

2 2 1<br />

2i 2 j k .<br />

On comparing the coefficients of (1) and (2)<br />

...(2)<br />

<br />

2a<br />

c<br />

= 2 b a 2 c b<br />

<br />

2 2 1<br />

2a + c = – 2b – a 3a + 2b + c = 0 ...(3)<br />

and 2b + a = – 2(2c + b)<br />

2b + a = – 4c – 2b a + 4b + 4c = 0 ...(4)<br />

Rewriting (3) and (4), we have<br />

3a 2b c<br />

0 <br />

<br />

<br />

b <br />

c<br />

a 4b 4c<br />

0<br />

4 11 10<br />

= k (say)<br />

a = 4k, b = –11k and c = 10k.<br />

Now, we have<br />

(2a + c) 2 + (2b + a) 2 + (2c + b) 2 = (15) 2<br />

(8k + 10k) 2 + (–22k + 4k) 2 + (20k – 11k) 2 = (15) 2<br />

k =<br />

a =<br />

<br />

5<br />

<br />

9<br />

20<br />

, b =<br />

9<br />

Example 32. If r xi y j zk,<br />

show that :<br />

<br />

r<br />

1<br />

(i) grad r =<br />

<br />

r<br />

r<br />

<br />

Solution. (i) r = xi yj zk r =<br />

r<br />

2r<br />

x<br />

r<br />

Similarly,<br />

y<br />

= 2x <br />

= y r<br />

grad r = r =<br />

and<br />

r<br />

(ii) grad .<br />

3<br />

r<br />

r<br />

x <br />

x<br />

r<br />

r<br />

z <br />

z<br />

r<br />

55<br />

and c =<br />

9<br />

<br />

50<br />

<br />

9<br />

Ans.<br />

(Nagpur University, Summer 2002)<br />

2 2 2<br />

x y z r 2 = x 2 + y 2 + z 2<br />

r r r<br />

i j k r i j k<br />

x y z<br />

x y z<br />

<br />

x y z xi yj<br />

zk r<br />

= i j k Proved.<br />

r r r r r<br />

1<br />

<br />

(ii) grad <br />

r<br />

= 1 1<br />

<br />

1 1 1<br />

i j k = i <br />

j k <br />

r x y zr<br />

xr yr zr<br />

<br />

= 1 r 1 r 1 r<br />

<br />

i j k<br />

2 2 <br />

2 <br />

r x r y<br />

r z<br />

<br />

=<br />

<br />

1 x 1 y 1 z<br />

i j k<br />

2 2 <br />

2 <br />

r r r r r r = xi yj<br />

zk r<br />

Proved.<br />

3 3<br />

r r<br />

2 2<br />

Example 33. Prove that f () r f ´´ () r f´( r)<br />

. (K. University, Dec. 2008)<br />

r<br />

Solution.


Vectors 397<br />

f () r<br />

<br />

i j k f () r<br />

x y z<br />

<br />

2 2 2 2 r r x r y r z <br />

r x y z 2r 2 x , and <br />

<br />

x x r y r z r <br />

r r r<br />

x y z <br />

if´( r) j f´() r k f´( r)<br />

f´( r)<br />

i j k<br />

x y z<br />

<br />

r r r<br />

<br />

<br />

<br />

xi<br />

yj<br />

zk<br />

f´( r) r<br />

2<br />

f () r [ f ( r)]<br />

<br />

xi yj zk <br />

i j k f´( r) x y z <br />

r<br />

<br />

<br />

<br />

x y z<br />

f´( r) <br />

f´() r <br />

f´( r)<br />

x <br />

r<br />

<br />

y <br />

r<br />

<br />

z <br />

r<br />

<br />

<br />

r<br />

r<br />

r1<br />

x<br />

r.1<br />

y<br />

r x r<br />

f´´( r) f´( r) x<br />

y <br />

y<br />

f´´( r)<br />

2 f´( r)<br />

<br />

xr r y<br />

r<br />

2<br />

<br />

r<br />

z<br />

r.1<br />

z<br />

r z f´´( r) f´( r)<br />

r<br />

2<br />

z r <br />

<br />

r<br />

2<br />

2<br />

2<br />

x<br />

y<br />

z<br />

r <br />

r <br />

r <br />

= x x f´´( r) f´( r)<br />

r y y<br />

<br />

f´´( r) f´( r)<br />

r z z<br />

<br />

2 <br />

´´ () ´( )<br />

r<br />

2 f r <br />

f r<br />

2<br />

r r r r r r r r<br />

r<br />

2 2 2 2<br />

2 2<br />

x x r x y<br />

r y z z<br />

r z<br />

= f´´( r) f´( r) f´´( r) f´()<br />

r<br />

3 <br />

<br />

3 f´´ () r <br />

f´( r)<br />

r r r r<br />

3<br />

r r r r<br />

2 2 2 2 2 2 2 2 2<br />

x y z y x z z x y<br />

f´´( r) f´( r) f´´( r) f´() r f´´( r) f´()<br />

r<br />

2 3 2 3 2 3<br />

r r r r r r<br />

<br />

2 2 2 2 2 2 2 2 2<br />

x y z y z z x x y <br />

= f´´( r) f´( r)<br />

2 2 2 <br />

3 3 3 <br />

r r r r r r <br />

2 2 2 2 2 2<br />

2 2<br />

x y x 2( x y z<br />

) r 2r<br />

f´´ () r f´( r)<br />

2 3<br />

= f´´( r) f´( r)<br />

2 3<br />

r<br />

r<br />

r r<br />

2<br />

= f´´() r f´( r)<br />

r<br />

Ans.<br />

EXERCISE 5.7<br />

1. Evaluate grad if = log (x 2 + y 2 + z 2 ) Ans.<br />

<br />

2( xi yj<br />

zk)<br />

2 2 2<br />

x y z<br />

2. Find a unit normal <strong>vector</strong> to the surface x 2 + y 2 + z 2 1<br />

= 5 at the point (0, 1, 2). Ans. ( ˆ 2 ˆ)<br />

5 j k<br />

(AMIETE, June 2010)<br />

3. Calculate the directional derivative of the function (x, y, z) = xy 2 + yz 3 at the point<br />

5<br />

(1, –1, 1) in the direction of (3, 1, –1) (A.M.I.E.T.E. Winter 2009, 2000) Ans.<br />

11<br />

4. Find the direction in which the directional derivative of f (x, y) = (x 2 – y 2 )/xy at (1, 1) is zero.<br />

(Nagpur Winter 2000)<br />

<br />

i j<br />

Ans.<br />

2


398 Vectors<br />

5. Find the directional derivative of the scalar function of (x, y, z) = xyz in the direction of the outer<br />

normal to the surface z = xy at the point (3, 1, 3).<br />

Ans.<br />

27<br />

11<br />

6. The temperature of the points in space is given by T(x, y, z) = x 2 + y 2 – z. A mosquito located at<br />

(1, 1, 2) desires to fly in such a direction that it will get warm as soon as possible. In what direction<br />

should it move?<br />

Ans.<br />

1 (2 i <br />

2 j <br />

<br />

k )<br />

3<br />

7. If (x, y, z) = 3xz 2 y – y 3 z 2 , find grad at the point (1, –2, –1) Ans.<br />

<br />

(16i 9j 4 k)<br />

8. Find a unit <strong>vector</strong> normal to the surface x 2 y + 2xz = 4 at the point (2, –2, 3).<br />

1<br />

Ans. ( i 2 j 2 k<br />

<br />

)<br />

3<br />

9. What is the greatest rate of increase of the function u = xyz 2 at the point (1, 0, 3)? Ans. 9<br />

10. If is the acute angle between the surfaces xyz 2 = 3x + z 2 and 3x 2 – y 2 + 2z = 1 at the point<br />

(1, –2, 1) show that cos = 3/7 6 .<br />

11. Find the values of constants a, b, c so that the maximum value of the directional directive of<br />

= axy 2 + byz + cz 2 x 3 at (1, 2, –1) has a maximum magnitude 64 in the direction parallel to the<br />

axis of z. Ans. a = b, b = 24, c = –8<br />

12. Find the values of and µ so that surfaces x 2 – µ y z = ( + 2)x and 4 x 2 y + z 3 = 4 intersect<br />

orthogonally at the point (1, –1, 2). Ans. = 9 , 1<br />

2 <br />

13. The position <strong>vector</strong> of a particle at time t is R = cos (t – 1) i + sinh (t – 1) j + at 2 k. If at t = 1,<br />

the acceleration of the particle be perpendicular to its position <strong>vector</strong>, then a is equal to<br />

1<br />

(a) 0 (b) 1 (c)<br />

2<br />

5.29 DIVERGENCE OF A VECTOR FUNCTION<br />

The divergence of a <strong>vector</strong> point function F <br />

<br />

(d)<br />

1<br />

2<br />

(AMIETE, Dec. 2009) Ans. (d)<br />

is denoted by div F and is defined as below.<br />

Let F = Fi 1 F 2 j Fk 3<br />

div F1 F2 F F = . F i j k ( iF1 jF2 kF3)<br />

= 3<br />

x y z<br />

x y z<br />

It is evident that div F is scalar function.<br />

5.30 PHYSICAL INTERPRETATION OF DIVERGENCE<br />

Let us consider the case of a fluid flow. Consider a small rectangular parallelopiped of<br />

dimensions dx, dy, dz parallel to x,y and z axes respectively.<br />

<br />

Let V Vx i Vy j Vz<br />

k<br />

be the velocity of the<br />

fluid at P(x, y, z).<br />

Mass of fluid flowing in through the face ABCD in unit time<br />

= Velocity × Area of the face = V x<br />

(dy dz )<br />

Mass of fluid flowing out across the face PQRS per unit time<br />

= V x<br />

(x + dx) (dy dz)<br />

Vx<br />

<br />

= Vx<br />

dx ( dydz)<br />

x<br />

<br />

<br />

Y<br />

Net decrease in mass of fluid in the parallelopiped<br />

corresponding to the flow along x-axis per unit time<br />

O<br />

Z<br />

A<br />

C<br />

D<br />

dz<br />

V x B<br />

S<br />

P<br />

R<br />

Q<br />

X


Vectors 399<br />

Vx<br />

<br />

= Vx<br />

dydz Vx<br />

dx dy dz<br />

x<br />

<br />

<br />

<br />

Vx<br />

= dx dy dz<br />

(Minus sign shows decrease)<br />

x<br />

Vy<br />

Similarly, the decrease in mass of fluid to the flow along y-axis = dx dy dz<br />

y<br />

Vz<br />

and the decrease in mass of fluid to the flow along z-axis = dx dy dz<br />

z<br />

V V<br />

x y Vz<br />

<br />

Total decrease of the amount of fluid per unit time = dx dy dz<br />

x y z<br />

<br />

V<br />

V<br />

x y Vz<br />

Thus the rate of loss of fluid per unit volume = <br />

x y z<br />

= i j k <br />

.( iV <br />

x jV y kV<br />

<br />

z)<br />

= . V div V<br />

x y z<br />

If the fluid is compressible, there can be no gain or loss in the volume element. Hence<br />

div V = 0 ...(1)<br />

and V is called a Solenoidal <strong>vector</strong> function.<br />

Equation (1) is also called the equation of continuity or conservation of mass.<br />

<br />

xi yj<br />

zk<br />

Example 34. If v <br />

,<br />

2 2 2<br />

x y z<br />

find the value of div v .<br />

(U.P., I Semester, Winter 2000)<br />

Solution. We have, v =<br />

<br />

xi yj<br />

zk<br />

2 2 2<br />

x y z<br />

div v =<br />

<br />

xi yj zk<br />

. v = i j k . <br />

<br />

x y z 2 2 2 1/2<br />

( x y z ) <br />

<br />

=<br />

x y z<br />

<br />

<br />

2 2 2 1/2 2 2 2 1/2 2 2 2 1/2<br />

x ( x y z ) y ( x y z ) z<br />

( x y z )<br />

<br />

1<br />

2 2 2 1/2 1<br />

<br />

2 2 2<br />

( x y z ) x. ( x y z )<br />

2<br />

.2x<br />

<br />

2<br />

<br />

=<br />

2 2 2<br />

( x y z )<br />

<br />

1 1<br />

2 2 2 2<br />

1<br />

2 2 2 1/2 1 2 2 2 1/2<br />

<br />

2 2 2<br />

( x y z ) y. ( x y z ) 2 2y<br />

<br />

( x y z ) z. ( x y z ) .2z<br />

<br />

<br />

2<br />

<br />

2<br />

<br />

<br />

<br />

<br />

2 2 2<br />

2 2 2<br />

( x y z )<br />

( x y z )<br />

=<br />

=<br />

2 2 2 2 2 2 2 2 2 2 2 2<br />

( x y z ) x ( x y z ) y ( x y z ) z<br />

<br />

<br />

2 2 2 3/2 2 2 2 3/2 2 2 2 3/2<br />

( x y z ) ( x y z ) ( x y z )<br />

2 2 2 2 2 2<br />

y z x z x y<br />

2 2 2 3/2<br />

( x y z )<br />

=<br />

2 2 2<br />

2( x y z ) 2<br />

<br />

2 2 2 3/2<br />

( x y z ) 2 2 2<br />

( x y z )<br />

<br />

Example 35. If u = x 2 + y 2 + z 2 , and r xi y j zk,<br />

then find div ( ur ) in terms of u.<br />

(A.M.I.E.T.E., Summer 2004)<br />

Ans.


400 Vectors<br />

Solution. div ( ur ) =<br />

<br />

<br />

2 2 2<br />

i j k .[( x y z )( xi yj<br />

zk)]<br />

x y z<br />

=<br />

<br />

<br />

2 2 2 2 2 2 2 2 2<br />

i j k .[( x y z ) xi( x y z ) yj( x y z ) zk]<br />

x y z<br />

=<br />

3 2 2 2 3 2 2 2 3<br />

( x xy xz ) ( x y y yz ) ( x z y z z )<br />

x y z<br />

= (3x 2 + y 2 + z 2 ) + (x 2 + 3y 2 + z 2 ) + (x 2 + y 2 + 3z 2 ) = 5 (x 2 + y 2 + z 2 ) = 5 u Ans.<br />

Example 36. Find the value of n for which the <strong>vector</strong><br />

<br />

r xi y j zk .<br />

Solution. Divergence F =<br />

=<br />

n<br />

r r is solenoidal, where<br />

n 2 2 2 n/2<br />

<br />

. F . r r .( x y z ) ( xi yj<br />

zk)<br />

<br />

<br />

2 2 2 n/2 2 2 2 n/2 2 2 2 n/2<br />

i j k .[( x y z ) xi( x y z ) yj( x y z ) zk]<br />

x y z<br />

= 2<br />

n (x 2 + y 2 + z 2 ) n/2 – 1 (2x 2 ) + (x 2 + y 2 + z 2 ) n/2 + 2<br />

n (x 2 + y 2 + z 2 ) n/2 – 1 (2y 2 )<br />

+ (x 2 + y 2 + z 2 ) n/2 + 2<br />

n (x 2 + y 2 + z 2 ) n/2 – 1 (2z 2 ) + (x 2 + y 2 + z 2 ) n/2<br />

= n(x 2 + y 2 + z 2 ) n/2 – 1 (x 2 + y 2 + z 2 ) + 3 (x 2 + y 2 + z 2 ) n/2<br />

= n(x 2 + y 2 + z 2 ) n/2 + 3(x 2 + y 2 + z 2 ) n/2 = (n + 3) (x 2 + y 2 + z 2 ) n/2<br />

n<br />

If r r is solenoidal, then (n + 3) (x 2 + y 2 + z 2 ) n/2 = 0 or n + 3 = 0 or n = –3. Ans.<br />

<br />

<br />

Example 37. Show that<br />

( a. r) a n( a. r)<br />

r<br />

<br />

. (M.U. 2005)<br />

n n n 2<br />

r <br />

r r<br />

Solution. We have,<br />

Let =<br />

<br />

<br />

x<br />

<br />

a.<br />

r<br />

r<br />

=<br />

n<br />

=<br />

<br />

But r 2 = x 2 + y 2 + z 2 r<br />

2r<br />

x<br />

<br />

( a i a j a k).( xi yj<br />

zk)<br />

1 2 3<br />

a.<br />

r ax 1 ay 2 az<br />

3<br />

<br />

n<br />

n<br />

r r<br />

n<br />

n1<br />

r . a ( ax a y az) nr ( r/ x)<br />

1 1 2 3<br />

2n<br />

n<br />

r<br />

r<br />

= 2x <br />

r x <br />

x<br />

r<br />

n<br />

=<br />

axa y az<br />

1 2 3<br />

n<br />

<br />

ar 1 ( ax 1 a2y az 3 ). nr . x a1<br />

nax ( 1 a2y a3zx<br />

)<br />

=<br />

2n<br />

= <br />

x<br />

r<br />

n<br />

n 2<br />

r r <br />

=<br />

<br />

i j k<br />

x y z<br />

=<br />

1 n<br />

<br />

( a1 i a<br />

n<br />

2 j a3k ) <br />

n 2 [( ax 1 a2y az 3 )( xi yj<br />

zk<br />

<br />

)]<br />

r<br />

r<br />

=<br />

a n<br />

( ar . ) r<br />

n n 2<br />

r r <br />

n<br />

2<br />

r


Vectors 401<br />

<br />

<br />

<br />

Example 38. Let r xi y j zk, r | r | and a is a constant <strong>vector</strong>. Find the value of<br />

<br />

<br />

a r<br />

div <br />

n<br />

r<br />

<br />

<br />

<br />

Solution. Let a = a 1 i a 2 j a 3 k<br />

=<br />

<br />

<br />

<br />

a r = ( a1i a2 j a3<br />

k) ( xi yj<br />

zk)<br />

<br />

<br />

a r<br />

<br />

| r | n<br />

<br />

a r<br />

div <br />

<br />

| r | n<br />

<br />

<br />

<br />

i j k<br />

a a a<br />

= 1 2 3<br />

=<br />

x y z<br />

<br />

= ( az 2 ayi 3 ) ( a1z a3x) j ( ay 1 axk 2 )<br />

<br />

( az ayi ) ( a z a x) j ( ay<br />

axk )<br />

2 3 1 3 1 2<br />

2 2 2 /2<br />

<br />

<br />

a r<br />

= .<br />

<br />

| r | n<br />

( x y z ) n<br />

<br />

( az 2 ayi 3 ) ( a1z a3x) j ( ay 1 axk 2 )<br />

= i j k .<br />

x y z<br />

2 2 2 /2<br />

<br />

( x y z ) n<br />

az 2 ay 3 az 1 ax 3 ( ay 1 ax 2 )<br />

<br />

<br />

2 2 2 n/2 2 2 2 n/2 2 2 2 n/2<br />

x ( x y z ) y ( x y z ) z<br />

( x y z )<br />

n ( azay)2 x n ( a z a x)2 y n ( ay<br />

ax)2z<br />

<br />

n2 n2 n2<br />

2 2 2<br />

2 2 2 2 2 2 2 2 2 2 2<br />

( x y z ) ( x y z ) ( x y z )<br />

2<br />

n<br />

[( azayx ) ( azaxy ) ( a y a xz ) ]<br />

=<br />

2 3 1 3 1 2<br />

=<br />

n 2 2 3 1 3 1 2<br />

2 2 2 2<br />

( x y z )<br />

n<br />

[ azxa xy ayz a xy ayz a zx]<br />

= 0 Ans.<br />

=<br />

n 2 2 3 1 3 1 2<br />

2 2 2 2<br />

( x y z )<br />

Example 39. Find the directional derivative of div ( u ) at the point (1, 2, 2) in the direction<br />

of the outer normal of the sphere x 2 + y 2 + z 2 <br />

4 4 4<br />

= 9 for u x i y j z k <br />

.<br />

<br />

Solution. div ( u ) = . u<br />

<br />

4 4 4 3 3 3<br />

= i j k .( x i y j z k) 4x 4y 4z<br />

x y z<br />

Outer normal of the sphere = (x 2 + y 2 + z 2 – 9)<br />

<br />

<br />

2 2 2<br />

= i j k ( x y z 9) 2xi 2yj<br />

2zk<br />

x y z<br />

Outer normal of the sphere at (1, 2, 2) = 2i 4 j 4 k<br />

...(1)<br />

Directional derivative =<br />

=<br />

<br />

3 3 3<br />

(4x 4y 4 z )<br />

<br />

<br />

<br />

i j k (4x 4y 4 z ) 12x i 12y j 12z k<br />

x y z<br />

<br />

3 3 3 2 2 2<br />

<br />

Directional derivative at (1, 2, 2) = 12 i 48 j 48 k<br />

...(2)


402 Vectors<br />

<br />

2i 4 j 4k<br />

Directional derivative along the outer normal = (12i 48 j 48 k).<br />

416 16<br />

[From (1), (2)]<br />

24 192 192<br />

= = 68 Ans.<br />

6<br />

Example 40. Show that div (grad r n ) = n (n + 1)r n – 2 , where<br />

2 2 2<br />

r = x y z<br />

2 1<br />

<br />

Hence, show that = 0. (U.P. I Semester, Dec. 2004, Winter 2002)<br />

r<br />

<br />

Solution. grad (r n <br />

n <br />

n n<br />

) = i r j r k r by definition<br />

x y z<br />

<br />

n 1 r <br />

n 1 r <br />

n<br />

1r<br />

<br />

n 1 r r <br />

r<br />

= inr j nr knr . = nr i j k<br />

x y z<br />

<br />

x y z<br />

<br />

n 1 x y z<br />

<br />

<br />

n2 n 2<br />

= nr i j k nr ( xi y j zk) nr r.<br />

r r r<br />

<br />

<br />

2 2 2 2 r r x <br />

<br />

r x y z 2r 2x etc.<br />

x x r<br />

<br />

<br />

<br />

<br />

Thus, grad (r n ) = n 2 n 2 n 2<br />

nr xi nr yj nr zk<br />

...(1)<br />

<br />

div grad r n = div [ n2 n2 n 2<br />

nr xi nr yj nr zk ]<br />

=<br />

=<br />

=<br />

<br />

<br />

n2 n 2 n 2<br />

i j k .( nr xi nr y j nr zk)<br />

x y z<br />

n 2 n 2 n 2<br />

( nr <br />

x) ( nr <br />

y) ( nr z)<br />

x y z<br />

n2 n3 r<br />

n2 n 3<br />

r<br />

<br />

nr nx ( n 2) r nr ny ( n 2) r <br />

x y<br />

<br />

nr nz ( n<br />

2) r<br />

<br />

n n r r r<br />

3 nr nn ( 2) r <br />

x y z<br />

<br />

<br />

x y z<br />

<br />

= 2 3<br />

[From (1)]<br />

(By definition)<br />

<br />

<br />

z<br />

<br />

n2 n3<br />

r<br />

n 2 n 3 x y z<br />

= 3 nr nn ( 2) r <br />

x y <br />

z <br />

<br />

r r r<br />

<br />

<br />

2 2 2 2 r r x <br />

<br />

r x y z 2r 2x etc.<br />

x x r<br />

<br />

<br />

<br />

= 3nr n – 2 + n (n – 2)r n – 4 [x 2 + y 2 + z 2 ]<br />

= 3nr n – 2 + n (n – 2) r n – 4 .r 2 ( r 2 = x 2 + y 2 + z 2 )<br />

= r n – 2 [3n + n 2 – 2n] = r n – 2 (n 2 + n) = n(n + 1) r n – 2<br />

If we put n = –1<br />

div grad (r – 1 ) = –1 (–1 + 1) r – 1 – 2<br />

2 1<br />

<br />

<br />

r<br />

= 0<br />

<br />

r <br />

1<br />

Ques. If r xi y j zk,<br />

and r = |r| find div .<br />

2 (U.P. I Sem., Dec. 2006) Ans. 2<br />

r<br />

<br />

r


Vectors 403<br />

EXERCISE 5.8<br />

<br />

<br />

<br />

1. If r = xi yj zk<br />

r <br />

and r = | r | , show that (i) div<br />

3<br />

| r |<br />

<br />

= 0,<br />

<br />

(ii) div (grad r n ) = n (n + 1) r n – 2 (AMIETE, June 2010) (iii) div (r ) = 3 + r grad .<br />

<br />

2. Show that the <strong>vector</strong> V = ( x3 y) i ( y 3 z) j ( x 2 z)<br />

k is solenoidal.<br />

(R.G.P.V., Bhopal, Dec. 2003)<br />

3. Show that .( A) = .A + (.A)<br />

4. If , , z are cylindrical coordinates, show that grad (log ) and grad are solenoidal <strong>vector</strong>s.<br />

5. Obtain the expression for 2 f in spherical coordinates from their corresponding expression in<br />

orthogonal curvilinear coordinates.<br />

Prove the following:<br />

<br />

6. .( F) ( ). F ( . F )<br />

<br />

7. (a) .() = 2 <br />

(b)<br />

( A<br />

R) (2 n) A n( A. R)<br />

R <br />

, | |<br />

n n n 2 r R<br />

r r r<br />

8. div ( f g) – div (g f) = f 2 g – g 2 f<br />

5.31 CURL (U.P., I semester, Dec. 2006)<br />

The curl of a <strong>vector</strong> point function F is defined as below<br />

Curl F <br />

<br />

<br />

curl F = F<br />

<br />

= i j k ( F1 i F2 j F3<br />

k)<br />

x y z<br />

<br />

i j k<br />

=<br />

1 2 3<br />

is a <strong>vector</strong> quantity.<br />

5.32 PHYSICAL MEANING OF CURL<br />

<br />

<br />

<br />

( F F i F j F k)<br />

1 2 3<br />

F<br />

3 F2 <br />

F<br />

3 F1<br />

F2 F1<br />

i j k <br />

x y z <br />

y z<br />

<br />

x z<br />

<br />

x y<br />

<br />

F F F<br />

(M.D.U., Dec. 2009, U.P. I Semester, Winter 2009, 2000)<br />

We know that V r,<br />

where is the angular velocity, V is the linear velocity and <br />

r<br />

is the position <strong>vector</strong> of a point on the rotating body.<br />

<br />

Curl V 1 i 2 j 3<br />

k <br />

<br />

= V<br />

<br />

r x i y j zk <br />

<br />

<br />

= ( r ) = [( 1i 2 j 3<br />

k) ( xi y j zk)]<br />

=<br />

<br />

i j k<br />

<br />

= <br />

[( 2z 3y) i ( 1z 3x) j ( 1y 2x) k]<br />

<br />

1 2 3<br />

x y z<br />

<br />

x y z<br />

<br />

= i j k 2z 3y i 1z 3x j 1y 2x k<br />

<br />

[( ) ( ) ( ) ]


404 Vectors<br />

=<br />

<br />

i j k<br />

<br />

x y z<br />

z y xz y x<br />

2 3 3 1 1 2<br />

<br />

= ( 1 2) i ( 2 2) j ( 3 3)<br />

k<br />

<br />

= 2( 1 i 2 j 3<br />

k) 2<br />

Curl V = 2 which shows that curl of a <strong>vector</strong> field is connected with rotational properties<br />

of the <strong>vector</strong> field and justifies the name rotation used for curl.<br />

If Curl F = 0, the field F is termed as irrotational.<br />

<br />

<br />

2 2 2<br />

Example 41. Find the divergence and curl of v ( xyz) i (3 x y) j ( xz y z)<br />

k at<br />

(2, –1, 1) (Nagpur University, Summer 2003)<br />

Solution. Here, we have<br />

<br />

2 2 2<br />

<br />

v = ( xyz) i (3 x y) j ( xz y z)<br />

k<br />

Div. <br />

v = <br />

Div 2 2 2<br />

v = ( xyz) (3 x y) ( xz y z)<br />

x y z<br />

= yz + 3x 2 + 2x z – y 2 = –1 + 12 + 4 – 1 = 14 at (2, –1, 1)<br />

Curl v =<br />

<br />

i j k<br />

<br />

x y z<br />

2 2 2<br />

xyz 3x y xz y z<br />

<br />

2<br />

= 2 yz i ( xy z ) j (6 xy xz)<br />

k<br />

Curl at (2, –1, 1)<br />

<br />

= 2( 1)(1) i {(2) ( 1) 1} j {6(2)( 1) 2(1)} k<br />

Example 42. If<br />

<br />

=<br />

<br />

2<br />

2 yz i ( z xy) j (6 xy xz)<br />

k<br />

= 2i 3 j 14k<br />

Ans.<br />

<br />

xi yj<br />

zk<br />

V <br />

x y z<br />

2 2 2 ,<br />

<br />

find the value of curl V .<br />

Solution. Curl V = V<br />

<br />

xi yj<br />

zk<br />

= i j k <br />

x y z 2 2 2 1/2<br />

( x y z ) <br />

<br />

=<br />

<br />

<br />

i j k<br />

<br />

x y z<br />

x y z<br />

(U.P., I Semester, Winter 2000)<br />

2 2 2 1/2 2 2 2 1/2 2 2 2 1/2<br />

( x y z ) ( x y z ) ( x y z )


Vectors 405<br />

=<br />

z y <br />

z <br />

i <br />

j<br />

2 2 2 1/2 2 2 2 1/2 2 2 2 1/2<br />

y <br />

( x y z ) z<br />

<br />

<br />

<br />

<br />

<br />

( x y z )<br />

<br />

x<br />

<br />

( x y z )<br />

<br />

<br />

x y x <br />

k<br />

<br />

2 2 2 1/2 2 2 2 1/2 2 2 2 1/2<br />

z <br />

( x y z ) x<br />

<br />

( x y z ) y<br />

<br />

<br />

( x y z ) <br />

yz yz . zx zx <br />

<br />

( x y z ) ( x y z ) ( x y z ) ( x y z<br />

) <br />

xy<br />

xy <br />

k <br />

0<br />

2 2 2 3/2 2 2 2 3/2 <br />

( x y z ) ( x y z<br />

) <br />

<br />

<br />

= i<br />

j<br />

<br />

2 2 2 3/2 2 2 2 3/2 2 2 2 3/2 2 2 2 3/2<br />

Example 43. Prove that<br />

2 2<br />

<br />

<br />

y z yz x i xz xy j xy xz z k <br />

Ans.<br />

( – 3 –2 ) (3 2 ) (3 –2 2 ) is both<br />

solenoidal and irrotational. (U.P., I Sem, Dec. 2008)<br />

Solution. Let F =<br />

2 2<br />

<br />

<br />

y z yz x i xz xy j xy xz z k <br />

( – 3 –2 ) (3 2 ) (3 –2 2)<br />

<br />

For solenoidal, we have to prove .F = 0.<br />

<br />

<br />

2 2<br />

<br />

Now, .F = i j k ( y – z 3 yz –2 x) i (3xz 2 xy) j (3 xy –2xz 2 z)<br />

k<br />

x y z<br />

<br />

<br />

<br />

= – 2 + 2x – 2x + 2 = 0<br />

Thus, F is solenoidal. For irrotational, we have to prove Curl F = 0.<br />

Now, Curl F =<br />

Thus, F is irrotational.<br />

<br />

i j k<br />

<br />

x y z<br />

2 2<br />

y – z 3 yz –2x 3xz 2xy 3 xy – 2xz 2z<br />

<br />

= (3z 2 y – 2y 3 z) i – (– 2z 3 y –3y 2 z)<br />

j <br />

<br />

= 0i 0 j 0 k = 0<br />

(3z 2 y – 2 y – 3) z k<br />

Hence, F is both solenoidal and irrotational.<br />

Proved.<br />

Example 44. Determine the constants a and b such that the curl of <strong>vector</strong><br />

2 2<br />

<br />

A = (2xy 3 yz) i ( x axz –4 z ) j –(3 xy byz)<br />

k is zero.<br />

(U.P. I Semester, Dec 2008)<br />

Solution. Curl A =<br />

<br />

<br />

<br />

<br />

2 2<br />

i j k [(2xy 3 yz) i ( x axz – 4 z ) j<br />

x y z<br />

<br />

<br />

<br />

=<br />

<br />

i j k<br />

<br />

x y z<br />

2 2<br />

2xy 3 yz x axz –4 z –3 xy – byz<br />

<br />

<br />

<br />

(3 xy byz) k]


406 Vectors<br />

<br />

= [– 3 x – bz – ax 8 z] i –[– 3 y –3 y] j [2 x az –2 x –3 zk ]<br />

<br />

= [– x(3 a) z(8– b)] i 6 y j z(–3 a)<br />

k<br />

= 0 (given)<br />

i.e., 3 + a = 0 and 8 – b = 0, – 3 + a = 0<br />

a = – 3, 3 b = 8 a = 3 Ans.<br />

Example 45. If a <strong>vector</strong> field is given by<br />

<br />

2 2<br />

F ( x – y x) i – (2 xy y)<br />

j . Is this field irrotational ? If so, find its scalar potential.<br />

Solution. Here, we have<br />

<br />

=<br />

2 2<br />

F<br />

Curl F =<br />

=<br />

=<br />

<br />

( x – y x) i – (2 xy y)<br />

j<br />

F <br />

<br />

<br />

<br />

<br />

<br />

<br />

2 2<br />

i j k ( x – y x) i – (2 xy y)<br />

j<br />

x y z<br />

<br />

i j k<br />

<br />

x y z<br />

2 2<br />

x – y x – 2 xy – y 0<br />

Hence, <strong>vector</strong> field F is irrotational.<br />

To find the scalar potential function <br />

F <br />

= <br />

<br />

<br />

(U.P. I Semester, Dec 2009)<br />

= i(0 –0)– j(0 –0) k(–2y 2 y)<br />

= 0<br />

d = dx dy <br />

<br />

<br />

dz = i j k ( i dx jdy kdz)<br />

x y z<br />

x y z<br />

<br />

<br />

<br />

<br />

<br />

= i j k ( d r )<br />

x y z<br />

<br />

<br />

= dr = Fdr <br />

<br />

2 2<br />

= [( x – y x) i<br />

–(2 xy y) j]( i dx jdy kdz)<br />

= (x 2 – y 2 + x)dx – (2xy + y)dy.<br />

2 2<br />

= [( x – y x) dx –(2 xy ydy ) ] c<br />

=<br />

Hence, the scalar potential is<br />

<br />

<br />

2 2<br />

x d x xdx ydy y dx 2 xy dy<br />

c =<br />

<br />

Example 46. Find the scalar potential function f for<br />

Solution. We have, A =<br />

3 2 2<br />

x 2<br />

<br />

<br />

x – y – xy c<br />

3 2 2<br />

3 2 2<br />

x x y 2<br />

– – xy c<br />

Ans.<br />

3 2 2<br />

Curl A = A <br />

=<br />

<br />

2 2<br />

y i 2xy j z k<br />

<br />

2 2<br />

A y i 2xyj z k .<br />

(Gujarat, I Semester, Jan. 2009)<br />

<br />

2 2<br />

i j k ( y i 2 xyj<br />

z k)<br />

x y z


Vectors 407<br />

=<br />

<br />

i j k<br />

<br />

x y z<br />

2 2<br />

y 2xy z<br />

<br />

= i(0) j(0) k(2y 2 y)<br />

= 0<br />

Hence, A is irrotational. To find the scalar potential function f.<br />

<br />

A<br />

= f<br />

df = f dx <br />

f dy <br />

f<br />

dz<br />

x y z<br />

<br />

= i j k f.<br />

dr<br />

x y z<br />

<br />

= Adr .<br />

f f f<br />

<br />

= i j k .( i dx jdy kdz)<br />

x y z<br />

<br />

= f.<br />

dr <br />

<br />

2 2<br />

= ( y i 2 xy j z k).( i dx jdy kdz)<br />

= y 2 dx + 2xy dy – z 2 dz = d (xy 2 ) – z 2 dz<br />

<br />

=<br />

f = d 2 2<br />

( xy ) z dz<br />

xy<br />

2<br />

3<br />

(A = f)<br />

z<br />

C<br />

Ans.<br />

3<br />

Example 47. A <strong>vector</strong> field is given by <br />

A = (x 2 + xy 2 ) i + (y 2 + x 2 y) j . Show that the field<br />

is irrotational and find the scalar potential.(Nagpur Univeristy, Summer 2003, Winter 2002)<br />

Solution. A is irrotational if curl A = 0<br />

<br />

i j k<br />

Curl A =<br />

Hence, A <br />

<br />

A<br />

<br />

<br />

A <br />

x y z<br />

2 2 2 2<br />

x xy y x y<br />

is irrotational. If is the scalar potential, then<br />

= grad <br />

d = dx <br />

dy <br />

dz<br />

x y z<br />

<br />

= i j k .( i dx jdy kdz)<br />

= grad . dr<br />

x y z<br />

<br />

<br />

0<br />

<br />

2 2 2 2<br />

<br />

= i(0 0) j(0 0) k(2xy 2 xy) 0<br />

[Total differential coefficient]<br />

= Adr . = [( x xy ) i ( y x y) j].( idx jdy kdz)<br />

= (x 2 + xy 2 ) dx + (y 2 + x 2 y) dy = x 2 dx + y 2 dy + (x dx)y 2 + (x 2 ) (y dy)<br />

=<br />

2 2 2 2<br />

xdx y dy [( xdx ) y ( x )( y dy )] =<br />

<br />

2 2<br />

3 3 2 2<br />

x y x y<br />

c Ans.<br />

3 3 2<br />

Example 48. Show that V( x, y, z) 2 x yzi ( x z 2 y)<br />

j x yk is irrotational and find a<br />

scalar function u(x, y, z) such that V <br />

= grad (u).<br />

Solution. V (x, y, z) =<br />

<br />

2 2<br />

2 xyzi ( x z 2 y)<br />

j x yk


408 Vectors<br />

Curl V =<br />

=<br />

=<br />

<br />

<br />

2 2<br />

i j k [2 x yzi ( x z 2 y) j x yk]<br />

x y z<br />

<br />

i j k<br />

<br />

x y z<br />

2 2<br />

2xyz x z 2y x y<br />

2 2<br />

<br />

( x x ) i (2xy 2 xy) j (2xz 2 xz) k 0<br />

Hence, V (x, y, z) is irrotational.<br />

To find corresponding scalar function u, consider the following relations given<br />

V <br />

= grad (u)<br />

<br />

or V = ( u)<br />

...(1)<br />

u u u<br />

du =<br />

dx <br />

dy <br />

dz (Total differential coefficient)<br />

x y z<br />

u u u<br />

<br />

= i j k .( i dx jdy k dz)<br />

x y z<br />

<br />

<br />

= udr<br />

. V.<br />

d r<br />

[From (1)]<br />

<br />

2 2<br />

= [2 xyzi ( xz 2 y) j x yk].( i dx jdy kdz)<br />

= 2 x y z dx + (x 2 z + 2y) dy + x 2 y dz<br />

= y(2x z dx + x 2 dz) + (x 2 z) dy + 2y dy<br />

= [yd (x 2 z) + (x 2 z) dy] + 2y dy = d(x 2 yz) + 2y dy<br />

Integrating, we get u = x 2 yz + y 2 Ans.<br />

<br />

<br />

Example 49. A fluid motion is given by v ( y z) i ( z x) j ( x y) k.<br />

Show that the<br />

motion is irrotational and hence find the velocity potential.<br />

(Uttarakhand, I Semester 2006; U.P., I Semester, Winter 2003)<br />

Solution. Curl v = v <br />

<br />

<br />

<br />

= i j k [( y z) i ( z x) j ( x y) k]<br />

x y z<br />

=<br />

<br />

i j k<br />

<br />

x y z<br />

y z z x x y<br />

<br />

= (1 1) i (1 1) j (1 1) k 0<br />

Hence, <br />

v is irrotational.<br />

To find the corresponding velocity potential , consider the following relation.<br />

v = <br />

d = dx <br />

dy <br />

dz<br />

[Total Differential coefficient]<br />

x y z


Vectors 409<br />

<br />

<br />

= i j k .( idx jdykdz)<br />

= i j k . d r .<br />

d r = vdr .<br />

x y z<br />

<br />

x y z<br />

<br />

= [( y z) i ( z x) j ( x y) k].( i dx jdy kdz)<br />

= (y + z) dx + (z + x) dy + (x + y) dz<br />

= y dx + z dx + z dy + x dy + x dz + y dz<br />

<br />

= xy + yz + zx + c<br />

= ( ydx x dy) ( zdy y dz) ( zdx x dz)<br />

Velocity potential = xy + yz + zx + c<br />

Example 50. A fluid motion is given by<br />

<br />

v = (y sin z – sin x) i + (x sin z + 2yz) j + (xy cos z + y 2 ) k <br />

is the motion irrotational? If so, find the velocity potential.<br />

Solution. Curl v = v<br />

=<br />

=<br />

<br />

<br />

<br />

i j k ( ysin zsin x) i +( xsin z+2 yz) j+( xy cos z + y ) k<br />

x y z<br />

<br />

i j k<br />

<br />

2<br />

<br />

x y z<br />

ysin z sin x xsin z 2yz xy cos z y<br />

= (x cos z + 2y – x cos z – 2y) i – [y cos z – y cos z] j + (sin z – sin z) k = 0<br />

Hence, the motion is irrotational.<br />

So,<br />

2<br />

Ans.<br />

v = where is called velocity potential.<br />

d = dx <br />

dy <br />

dz<br />

[Total differential coefficient]<br />

x y z<br />

<br />

= i j k .( i dx jdy kdz)<br />

= .d r = v.<br />

dr<br />

x y z<br />

<br />

= [(y sin z – sin x) i + (x sin z + 2yz) j + (xy cos z + y 2 ) k <br />

].[ idx jdy kdz]<br />

= (y sin z – sin x) dx + (x sin z + 2 y z) dy + (x y cos z + y 2 ) dz<br />

= (y sin z dx + x dy sin z + x y cos z dz) – sin x dx + (2 y z dy + y 2 dz)<br />

= d (x y sin z) + d (cos x) + d (y 2 z)<br />

<br />

2<br />

= d ( xy sin z) d (cos x) d( y z)<br />

= xy sin z + cos x + y 2 z + c<br />

Hence, Velocity potential = xy sin z + cos x + y 2 z + c. Ans.<br />

Example 51. Prove that<br />

<br />

Solution. Given F =<br />

Consider<br />

<br />

<br />

F =<br />

<br />

2<br />

F r r is conservative and find the scalar potential such that<br />

F = . (Nagpur University, Summer 2004)<br />

<br />

i j k<br />

<br />

x y z<br />

2 2 2<br />

r x r y r z<br />

2<br />

r r =<br />

<br />

2 2 2<br />

<br />

r 2 ( xi y j zk)<br />

= r xi r yj<br />

r zk


410 Vectors<br />

<br />

F <br />

= 0<br />

2 2 <br />

2 2 <br />

2 2 <br />

= i r z r y j r z r x k r y r x<br />

y z <br />

x z <br />

<br />

<br />

x y<br />

<br />

<br />

r r r r r r<br />

= i 2rz 2ry j 2rz 2rx k 2ry 2rx<br />

y z <br />

x z <br />

<br />

<br />

x y<br />

<br />

<br />

2 2 2 2 r x r y r z<br />

But r x y z<br />

, , , <br />

x r y r z r<br />

<br />

<br />

<br />

y z x z x y<br />

= i<br />

<br />

2rz 2ry j 2rz 2rx k 2ry 2rx<br />

r r r r r r<br />

<br />

<br />

= i(2yz 2 yz) j(2zx 2 zx) k(2xy 2 xy)<br />

= 0i 0 j 0k<br />

0<br />

F is irrotational F is conservative.<br />

Consider scalar potential such that F = .<br />

d = dx <br />

dy <br />

dz<br />

x y z<br />

<br />

= i j k .( i dx jdy kdz)<br />

x y z<br />

<br />

<br />

= i j k .( i dx jdy kdz)<br />

x y z<br />

<br />

= F.( idx jdy kdz)<br />

=<br />

= 2 2 2<br />

[Total differential coefficient]<br />

<br />

= .( idx jdy kdz)<br />

<br />

2<br />

<br />

r r.( idx jdy kdz)<br />

( = F )<br />

( x y z )( ix jy kz).( idx jdy kdz)<br />

= (x 2 + y 2 + z 2 ) (x dx + y dy + z dz)<br />

= x 3 dx + y 3 dy + z 3 dz + (x dx) y 2 + (x 2 ) (y dy)<br />

+ (x dx)z 2 + z 2 (y dy) + x 2 (z dz) + y 2 (z dz)<br />

3 3 3 2 2<br />

= x dx y dy z dz [( x dx) y ( y dy) x ]<br />

=<br />

<br />

4 4 4<br />

x y z 1 2 2 1 2 2 1 2 2<br />

x y xz y z c<br />

4 4 4 2 2 2<br />

<br />

2 2 2 2<br />

[( xdxz ) ( zdzx ) ] [( ydyz ) ( zdz) y ]<br />

= 1 4 (x4 + y 4 + z 4 + 2x 2 y 2 + 2x 2 z 2 + 2y 2 z 2 ) + c Ans.<br />

r<br />

Example 52. Show that the <strong>vector</strong> field F is irrotational as well as solenoidal. Find<br />

3<br />

| r |<br />

the scalar potential.<br />

(Nagpur University, Summer 2008, 2001, U.P. I Semester Dec. 2005, 2001)<br />

<br />

<br />

r xi y j zk<br />

Solution. F = <br />

2 2 2 3/2<br />

3<br />

| r |<br />

( x y z )<br />

<br />

Curl F xi yj<br />

zk<br />

= F = i j k <br />

x y z 2 2 2 3/2<br />

( x y z )


Vectors 411<br />

=<br />

<br />

i j k<br />

<br />

x y z<br />

x y z<br />

2 2 2 3/2 2 2 2 3/2 2 2 2 3/2<br />

( x y z ) ( x y z ) ( x y z )<br />

3 2yz<br />

3 2yz<br />

<br />

2 ( x y z ) 2 ( x y z )<br />

<br />

= i<br />

<br />

2 2 2 5/2 2 2 2 5/2<br />

= 0<br />

Hence, F is irrotational.<br />

F = , where is called scalar potential<br />

d = dx <br />

dy <br />

dz<br />

x y z<br />

<br />

= i j k .( i dx jdy kdz)<br />

x y z<br />

<br />

<br />

xi yj<br />

zk<br />

= .( <br />

idx jdy kdz )<br />

2 2 2 3/2<br />

( x y z )<br />

1 2xdx 2y dy 2zdz<br />

=<br />

2 2 2 3/2<br />

2<br />

<br />

( x y z )<br />

=<br />

Now, Div F <br />

= .F<br />

=<br />

3 2xz<br />

3<br />

2xz<br />

j <br />

<br />

2 <br />

( x y z ) 2 <br />

( x y z )<br />

3 2xy<br />

3<br />

2xy<br />

k <br />

2 <br />

( x y z ) 2 <br />

( x y z )<br />

1<br />

<br />

2 2 2 2<br />

<br />

<br />

<br />

2 2 2 5/2 2 2 2 5/2<br />

2 2 2 5/2 2 2 2 5/2<br />

<br />

<br />

<br />

<br />

<br />

<br />

[Total differential coefficient]<br />

<br />

= . dr Fdr .<br />

xdx y dy zdz<br />

= 2 2 2 3/2<br />

( x y z )<br />

1<br />

2<br />

1 1<br />

( x y z ) <br />

1<br />

2<br />

1 <br />

| r |<br />

2 2 2<br />

( x y z ) 2<br />

<br />

xi yj<br />

zk<br />

= i j k . (<br />

2 2 2 )<br />

3/2<br />

x y z<br />

x y z<br />

x y z<br />

<br />

<br />

x ( x y z ) y ( x y z ) z<br />

( x y z )<br />

2 2 2 3/2 3<br />

2 2 2 1/2<br />

( x y z ) (1) x<br />

( x y z ) (2 x)<br />

2<br />

<br />

2 2 2 3<br />

( x y z )<br />

=<br />

2 2 2 3/2 2 2 2 3/2 2 2 2 3/2<br />

Ans.<br />

2 2 2 3/2 3<br />

2 2 2 1/2<br />

( x y z ) (1) y<br />

( x y z ) (2 y)<br />

<br />

2<br />

<br />

2 2 2 3<br />

( x y z )<br />

2 2 2 3/2 3<br />

2 2 2 1/2<br />

( x y z ) (1) z<br />

( x y z ) (2 z)<br />

<br />

2<br />

<br />

2 2 2 3<br />

( x y z )


412 Vectors<br />

=<br />

= 0<br />

2 2 2 1/2<br />

( x y z )<br />

2 2 2 3<br />

( x y z )<br />

[x 2 + y 2 + z 2 – 3x 2 + x 2 + y 2 + z 2 – 3y 2 + x 2 + y 2 + z 2 – 3z 2 ]<br />

Hence, F is solenoidal.<br />

Proved.<br />

<br />

2 2 2 2<br />

Example 53. Given the <strong>vector</strong> field V ( x y 2 xz) i ( xz xy yz) j ( z x ) k find<br />

curl V. Show that the <strong>vector</strong>s given by curl V at P 0<br />

(1, 2, –3) and P 1<br />

(2, 3, 12) are orthogonal.<br />

<br />

<br />

Solution. Curl V = V<br />

<br />

<br />

2 2 2 2<br />

= i j k [( x y 2 xz) i( xz xy yz) j( z x ) k]<br />

x y z<br />

curl V =<br />

<br />

i j k<br />

<br />

x y z<br />

2 2 2 2<br />

x y 2xz xz xy yz z x<br />

<br />

= ( x y) i (2x2 x) j ( z y 2 yk ) = ( x y) i ( y z)<br />

k<br />

<br />

curl V at P 0<br />

(1, 2, –3) = (1 2) i (23) k 3<br />

i k<br />

<br />

curl V at P 1<br />

(2, 3, 12) = (2 3) i (312) k 5i 15k<br />

The curl V <br />

at (1, 2, –3) and (2, 3, 12) are perpendicular since<br />

<br />

( 3 i k).( 5i 15 k)<br />

= +15 – 15 = 0 Proved.<br />

Example 54. Find the constants a, b, c, so that<br />

<br />

F = ( x2 y az) i ( bx 3 y z) j (4xcy 2 z)<br />

k<br />

...(1)<br />

is irrotational and hence find function such that F <br />

= .<br />

(Nagpur University, Summer 2005, Winter 2000; R.G.P.V., Bhopal 2009)<br />

Solution. We have,<br />

<br />

F <br />

=<br />

As F is irrotational, 0<br />

F <br />

<br />

i j k<br />

<br />

x y z<br />

( x 2 y az) ( bx 3 y z) (4x cy 2) z<br />

<br />

= ( c 1) i (4 a) j ( b 2) k<br />

<br />

<br />

i.e., ( c 1) i (4 a) j ( b 2) k 0i 0 j 0k<br />

c + 1 = 0, 4 – a = 0 and b – 2 = 0<br />

i.e., a = 4, b = 2, c = –1<br />

Putting the values of a, b, c in (1), we get<br />

<br />

F = ( x 2y 4 z) i (2x 3 y z) j(4x y 2 z)<br />

k


Vectors 413<br />

Now we have to find such that F <br />

= <br />

We know that<br />

d = dx <br />

dy <br />

dz<br />

x y z<br />

[Total differential coefficient]<br />

=<br />

<br />

i j k .( i dx jdy kdz)<br />

x y z<br />

<br />

=<br />

<br />

<br />

i j k .( i dx jdy kdz)<br />

= .( idx jdy kdz)<br />

x y z<br />

<br />

= F.( i dx jdy kdz)<br />

<br />

= [( x 2y 4 z) i (2x3 y z) j (4x y 2 z) k)].( i dx jdy kdz)<br />

= (x + 2y + 4z) dx + (2x – 3y – z) dy + (4x – y + 2z) dz<br />

= x dx – 3y dy + 2z dz + (2y dx + 2x dy) + (4z dx + 4x dz) + (–z dy – y dz)<br />

( zdy<br />

ydz)<br />

= xdx3 ydy 2 zdz (2ydx 2 xdy) (4zdx 4 xdz)<br />

<br />

=<br />

2 2<br />

x 3y<br />

+ z 2 + 2xy + 4zx – yz + c Ans.<br />

2 2<br />

Example 55. Let V (x, y, z) be a differentiable <strong>vector</strong> function and (x, y, z) be a scalar<br />

function. Derive an expression for div ( V <br />

) in terms of .V , div V and .<br />

(U.P. I Semester, Winter 2003)<br />

<br />

Solution. Let V = V 1 i V 2 j V 3 k<br />

div ( V <br />

) = .( F)<br />

<br />

<br />

= i j k .[ V1 i V2 j V3<br />

k]<br />

= ( V1) ( V2) ( V3)<br />

x y z<br />

x y z<br />

V1 V2<br />

V3<br />

<br />

= V1 V2 V3<br />

x x y y <br />

z z<br />

<br />

<br />

<br />

V1 V2<br />

V3<br />

<br />

= V1 V2 V3<br />

x y z <br />

x y z<br />

<br />

<br />

<br />

= i j k .( V1 i V2 j V3<br />

k)<br />

i j k .( V1 i V2 j V3<br />

k)<br />

x y z<br />

x y z<br />

<br />

<br />

= ( . V) ( ). V (div V) (grad ).<br />

V<br />

Ans.<br />

Example 56. If A is a constant <strong>vector</strong> and R = x î + y ĵ + z ˆk , then prove that<br />

<br />

<br />

<br />

<br />

Curl A.<br />

R<br />

A A R<br />

(K. University, Dec. 2009)<br />

<br />

<br />

<br />

Solution. Let A = A1 î + A 2<br />

ĵ + A 3<br />

ˆk , R = xî + y ĵ + z ˆk<br />

<br />

A.<br />

R (A 1 î + A 2<br />

ĵ + A 3<br />

ˆk ) . (x î + y ĵ + z ˆk ) = A 1<br />

x + A 2<br />

y + A 3<br />

z<br />

<br />

[ A. R]<br />

R = (A1 x + A 2<br />

y + A 3<br />

z) (x î + y ĵ + z ˆk )<br />

= (A 1<br />

x 2 + A 2<br />

xy + A 3<br />

zx) î + (A 1<br />

xy + A 2<br />

y 2 + A 3<br />

yz) ĵ + (A 1 xz + A 2 yz + A 3 z2 ) ˆk


414 Vectors<br />

iˆ<br />

ˆj kˆ<br />

<br />

Curl ( A. R)<br />

R<br />

= <br />

x y z<br />

2 2 2<br />

Ax Axy A zx Axy A y A yz Axz A yz Az<br />

L.H.S. = A<br />

R<br />

1 2 3 2 2 3 1 2 3<br />

= (A 2<br />

z – A 3<br />

y) î – [A 1<br />

z – A 3<br />

x) ĵ [A 1 y – A 2 x] ˆk ... (1)<br />

<br />

<br />

= (A 1 î + A 2<br />

ĵ + A 3<br />

k) ×(x î + y ĵ + z ˆk )<br />

iˆ<br />

ˆj k<br />

A A A<br />

= 1 2 3<br />

x y z<br />

Example 57. Suppose that UV ,<br />

= (A 2<br />

z – A 3<br />

y) î – (A 1<br />

z – A 3<br />

x) ĵ + (A y – A x) 1 2<br />

ˆk<br />

= R.H.S. [From (1)]<br />

<br />

<br />

and f are continuously differentiable fields then<br />

Prove that, div ( U V) Vcurl . U U.<br />

curl V . (M.U. 2003, 2005)<br />

<br />

Solution. Let U = ui 1 u2 ju3k,<br />

V v1i v2 j<br />

v3k<br />

<br />

<br />

U V =<br />

<br />

i j k<br />

u u u<br />

1 2 3<br />

v v v<br />

1 2 3<br />

<br />

= ( uv 2 3 uv 3 2) i ( u1v3 u3v1) j + ( uv 1 2 uv 2 1)<br />

k<br />

<br />

<br />

div ( U V)<br />

= i j k .[( u2v3u3v2) i( uv 1 3 uv 3 1) j+ ( u1v2u2v1) k]<br />

x y z<br />

<br />

= ( uv 2 3uv 3 2) ( uv 1 3uv 3 1) ( uv 1 2uv<br />

2 1)<br />

x y z<br />

v3 u2 v2 u3<br />

v3<br />

u1<br />

v1<br />

u3<br />

<br />

= u2 v3 u3 v2 u1 v3<br />

x x x x <br />

u3 v1<br />

y y<br />

y<br />

y<br />

<br />

<br />

v2 u1 v1 u2<br />

u1 v2 u2 v1<br />

z z z z<br />

<br />

<br />

<br />

u3 u2 u3<br />

u1 u2 u1<br />

<br />

= v1 v2 v3<br />

<br />

y z <br />

x z <br />

<br />

x y<br />

<br />

<br />

v3 v2 v3<br />

v1 v1 v2<br />

<br />

u1 u2<br />

u3<br />

<br />

y z <br />

x z <br />

<br />

y x<br />

<br />

<br />

u3 u2 u 1 u3 <br />

<br />

u2 u1<br />

= ( v1i v2 j v3<br />

k).<br />

i j<br />

k<br />

<br />

y z z x <br />

<br />

x y<br />

<br />

<br />

<br />

v2 v3 v3<br />

v1 v1 v2<br />

<br />

( u1 i u2 j u3<br />

k).<br />

i j k <br />

z y x z <br />

<br />

y x<br />

<br />

<br />

<br />

<br />

= V.( U) U.( V) V.curl U U.curl<br />

V<br />

Proved.


Vectors 415<br />

Example 58. Prove that<br />

<br />

<br />

( F G)<br />

= F( . G) G( . F) ( G. ) F ( F. ) G (M.U. 2004, 2005)<br />

Solution.<br />

<br />

( F G)<br />

= i ( F G)<br />

x<br />

=<br />

F G F G<br />

<br />

i G F i G i F<br />

<br />

x x x x<br />

<br />

=<br />

F F G<br />

G<br />

( iG . ) i G i F ( i. F)<br />

<br />

x x x x<br />

<br />

=<br />

F F G<br />

G<br />

( Gi . ) Gi. Fi. ( F. i)<br />

x x x x<br />

=<br />

G <br />

F <br />

F G<br />

F i Gi. ( G. i) ( F. i)<br />

x x x x<br />

<br />

= F ( G) G ( . F) ( G. ) F ( F. )<br />

G<br />

Proved.<br />

Questions for practice:<br />

Prove that<br />

<br />

( FG . ) = ( G. ) F ( F. ) G G( F) F ( G)<br />

Example 59. Prove that, for every field V ; div curl V = 0.<br />

(Nagpur University, Summer 2004; AMIETE, Sem II, June 2010)<br />

<br />

Solution. Let V = V1 i V2 j V3<br />

k<br />

=<br />

<br />

div (curl V ) = .( V<br />

)<br />

<br />

i j k<br />

<br />

.<br />

x y z<br />

V V V<br />

1 2 3<br />

V3 V2 V<br />

3 V1 V 2 V1<br />

<br />

= i j k .<br />

i j<br />

k <br />

x y z y z x z x y<br />

<br />

V3 V2 V3<br />

V1 V2 V1<br />

<br />

= <br />

x y z y<br />

<br />

x z<br />

<br />

z x y<br />

<br />

2 2 2 2 2 2<br />

V3 V2 V3<br />

V1 V2 V1<br />

= <br />

xy xz yx yz zx zy<br />

<br />

2 2 2 2<br />

2 2<br />

V1 V <br />

1 V2 V <br />

2 V3 V <br />

3<br />

=<br />

<br />

yz zy zx xz xy yx<br />

<br />

= 0 Ans.<br />

Example 60. If a <br />

is a constant <strong>vector</strong>, show that<br />

<br />

<br />

<br />

a ( r)<br />

= ( a. r) ( a. ) r.<br />

(U.P., Ist Semester, Dec. 2007)<br />

<br />

Solution. a = a1 i a2 j a3k,<br />

r r1 i r2 j r3k


416 Vectors<br />

<br />

<br />

r =<br />

<br />

<br />

i j k<br />

<br />

x y z<br />

r r r<br />

=<br />

1 2 3<br />

<br />

i j k<br />

a ( r)<br />

=<br />

a1 a2 a3<br />

r3 r2 r3<br />

r1 r2 r1<br />

<br />

y z x z x y<br />

r<br />

r r<br />

r r r<br />

<br />

i j k<br />

y z<br />

<br />

x z<br />

<br />

x y<br />

<br />

<br />

3 2 3 1 2 1<br />

r2 r1 r3 r1 r2 r1 r3<br />

r2<br />

<br />

a2 a2 a3 a3 i a1 a1 a3 a3<br />

j<br />

= <br />

x y <br />

x z <br />

x y y z<br />

<br />

<br />

<br />

<br />

r3 r1 r3<br />

r2<br />

<br />

a1 a1 a2 a2<br />

k<br />

x z y z<br />

<br />

<br />

r <br />

1 r <br />

2 r3 r <br />

1 r<br />

<br />

2 r3<br />

<br />

= a1i a2 i a3 i a1 j a2 j a3<br />

j <br />

x x x y y y<br />

<br />

r <br />

1 r <br />

2 r3 <br />

r <br />

1 r<br />

<br />

2 r3<br />

<br />

ak 1 a2k a3k <br />

a1 i a1 j ak 1 <br />

z z z <br />

x x x<br />

<br />

r <br />

1 r <br />

2 r3 r <br />

1 r<br />

<br />

2 r3<br />

<br />

a2 i a2 j a2k a3 i a3 j a3k<br />

<br />

y y y z z z<br />

<br />

<br />

= i j k ( a11 r a2r2 a33 r ) a1 a2 a3 ( r1 i r2 j<br />

r3<br />

k)<br />

x y z x y z<br />

<br />

<br />

= ( a. r) ( a. )<br />

r<br />

Proved.<br />

Example 61. If r is the distance of a point (x, y, z) from the origin, prove that<br />

1 1<br />

Curl k grad <br />

grad kgrad<br />

.<br />

= 0, where k is the unit <strong>vector</strong> in the direction OZ.<br />

r r (U.P., I Semester, Winter 2000)<br />

Solution. r 2 = (x – 0) 2 + (y – 0) 2 + (z – 0) 2 = x 2 + y 2 + z 2<br />

<br />

1<br />

r<br />

= (x 2 + y 2 + z 2 ) – 1/2<br />

grad 1 1 2 2 2 1/2<br />

= = i j k ( x y z )<br />

r<br />

<br />

r x y z<br />

=<br />

1 ( 2 2 2 3/2<br />

x y z (2 xi 2 yj 2 zk<br />

)<br />

2<br />

k × grad 1 r<br />

2 2 2 3/2<br />

<br />

= – ( x y z ) ( xi yj<br />

zk)<br />

=<br />

2 2 2 3/2<br />

<br />

k [ ( x y z ) ( xi yj<br />

zk)]<br />

curl<br />

1 <br />

k<br />

grad <br />

r <br />

=<br />

=<br />

2 2 2 3/2<br />

( x y z ) ( xj<br />

yi)<br />

1 <br />

k<br />

grad <br />

r


Vectors 417<br />

<br />

= i j k <br />

x y z × [–(x2 + y 2 + z 2 ) –3/2 <br />

( xj yi)<br />

]<br />

<br />

i j k<br />

<br />

= x y z<br />

y<br />

x<br />

0<br />

2 2 2 3/2 2 2 2 3/2<br />

( x y z ) ( x y z )<br />

3 ( x)(2) z 3 y (2 z) 3 ( x)(2 x)<br />

= <br />

i j <br />

2 2 2 5/2 2 2 2 5/2 <br />

2 2 2 5/2<br />

2 ( x y z ) 2( x y z ) 2 ( x y z )<br />

1 ( 3/2) ( y)(2 y) 1 <br />

k<br />

2 2 2 3/2 2 2 2 5/2 2 2 2 3/2 <br />

( x y z ) ( x y z ) ( x y z ) <br />

2 2 2 2 2 2 2 2<br />

3xz 3 yz (3x x y z 3 y x y z<br />

) <br />

=<br />

i <br />

j <br />

k<br />

2 2 2 5/2 2 2 2 5/2 2 2 2 5/2<br />

( x y z ) ( x y z ) ( x y z )<br />

<br />

2 2 2<br />

<br />

3xz i 3 yzj ( x y 2 z ) k<br />

=<br />

2 2 2 5/2<br />

( x y z )<br />

...(1)<br />

k . grad 1 r = <br />

2 2 2 3/2<br />

z<br />

k.[ ( x y z ) ( xi yj zk)]<br />

<br />

2 2 2 3/2<br />

( x y z )<br />

1 z<br />

grad k.grad<br />

= i j k 2 2 2 3/2<br />

r x y z<br />

( x y z )<br />

=<br />

<br />

<br />

3 i( z)(2 x) 3 j( z)(2 y)<br />

<br />

<br />

2<br />

2 2 2 5/2<br />

2<br />

2 2 2 5/2<br />

( x y z ) ( x y z<br />

)<br />

3 ( z)(2 z) 1 <br />

<br />

<br />

k<br />

2 2 2 5/2 2 2 2 3/2<br />

2<br />

<br />

( x y z ) ( x y z<br />

) <br />

<br />

2 2 2 2 2 2 2<br />

= 3xz i 3 yzj(3 z x y z ) k 3xz i 3 yzj( x y 2 z ) k<br />

<br />

2 2 2 5/2 2 2 2 5/2<br />

( x y z ) ( x y z )<br />

...(2)<br />

Adding (1) and (2), we get<br />

1 1<br />

Curl kgrad <br />

grad k.grad<br />

<br />

<br />

r r = 0 Proved.<br />

<br />

<br />

a r (2 n) a n( a. r)<br />

r<br />

Example 62. Prove that <br />

.<br />

n n n 2<br />

r r r<br />

<br />

(M.U. 2009, 2005, 2003, 2002; AMIETE, II Sem. June 2010)<br />

Solution. We have,<br />

<br />

a r<br />

r<br />

n<br />

<br />

1<br />

i j k<br />

a a a<br />

=<br />

n 1 2 3<br />

r<br />

<br />

x y z<br />

1 1 1<br />

<br />

= ( az 2 ayi 3 ) ( a3x a1z) j ( ay 1 axk 2 )<br />

n n n<br />

r r r


418 Vectors<br />

<br />

i j k<br />

<br />

<br />

( a r ) <br />

n<br />

= x y z<br />

r<br />

azay ax az ay<br />

ax<br />

2 3 3 1 1 2<br />

n n n<br />

r r r<br />

<br />

ay 1 ax 2 ax 3 az 1 <br />

ay 1 ax<br />

2 az 2 ay<br />

3 <br />

= i j<br />

n <br />

n n n<br />

y r z<br />

<br />

<br />

r x r z<br />

<br />

<br />

r <br />

ax 3 az 1 az 2 ay 3 <br />

k <br />

n<br />

n<br />

x<br />

<br />

<br />

r y<br />

<br />

<br />

r <br />

Now, r 2 = x 2 + y 2 + z 2 <br />

r<br />

r<br />

x<br />

2r<br />

= 2x <br />

x<br />

x<br />

r<br />

Similarly,<br />

r<br />

y<br />

= ,<br />

r<br />

z <br />

y<br />

r<br />

z<br />

r<br />

<br />

a r <br />

<br />

n<br />

1<br />

y <br />

1 <br />

= i . ( n<br />

r<br />

nr a1y a2x)<br />

a<br />

n 1<br />

r <br />

r <br />

<br />

n<br />

1<br />

z <br />

1 <br />

nr ( ax 3 az 1 ) ( a1<br />

)<br />

r<br />

n + two similar terms<br />

<br />

r <br />

=<br />

<br />

n 2 a1 n<br />

2 a1<br />

i ( a<br />

2 1y a2xy) ( axz<br />

2 3 a1z<br />

) <br />

n<br />

n n<br />

n<br />

r r r r <br />

+ two similar terms<br />

=<br />

<br />

2a1<br />

n 2 2 n<br />

<br />

i a<br />

2 1( y z ) ( axy<br />

2 2 a3xz)<br />

n n<br />

n<br />

<br />

+ two similar terms<br />

r r r<br />

<br />

Adding and subtracting<br />

n 2<br />

ax to third and from second term, we get<br />

n 2<br />

r <br />

1<br />

<br />

a r<br />

<br />

<br />

2a1 na1<br />

2 2 2 n 2<br />

<br />

n<br />

r<br />

= i ( x y z ) ( ax<br />

2 2 1 axy 2 a3xz)<br />

n n<br />

n <br />

<br />

r r r<br />

<br />

<br />

+ two similar terms<br />

2a1 na1<br />

2 n<br />

<br />

= i r xax (<br />

2 2 1 a2y a3<br />

z)<br />

n n<br />

n <br />

+ two similar terms<br />

r r r<br />

<br />

2a1 na1<br />

n<br />

<br />

<br />

2a2 na2<br />

n<br />

<br />

= i x( ax<br />

2 1 ay 2 az 3 )<br />

n n n <br />

<br />

j<br />

y( a<br />

2 2 y az 3 ax 1 )<br />

n n n <br />

<br />

r r r<br />

r r r<br />

<br />

2a3 na3<br />

n<br />

<br />

k<br />

z( az<br />

2 3 ax 1 a2<br />

y)<br />

n n n <br />

<br />

r r r<br />

<br />

2 n n<br />

= ( a<br />

n 1 i a2 j a3k ) <br />

n ( a1 i a2 j a3k<br />

) ( 2 1 2 3 )( <br />

ax a y az xi yj<br />

zk )<br />

n <br />

r<br />

r<br />

r<br />

2 n n<br />

<br />

= ( a1 i a2 j a3k) ( ax<br />

2 1 a2y az 3 )( xi yj<br />

zk)<br />

n<br />

n <br />

r<br />

r<br />

2 n n <br />

= a ( a. r)<br />

r<br />

n n<br />

2<br />

Proved.<br />

r r<br />

Example 63. If f and g are two scalar point functions, prove that<br />

div (f g) = f 2 g + f g. (U.P., I Semester, compartment, Winter 2001)


Vectors 419<br />

g g g<br />

<br />

Solution. We have, g = i j k<br />

x y z<br />

g g g<br />

<br />

f g = f i f j f k<br />

x y z<br />

g g g<br />

<br />

div (f g) = f <br />

f <br />

f <br />

x x y y z z<br />

<br />

<br />

2 2 2<br />

g g g f g f g f g<br />

= f<br />

<br />

<br />

2 2 2<br />

x y z<br />

<br />

<br />

x x y y z z<br />

<br />

<br />

2 2 2<br />

f f f g g g<br />

<br />

= f <br />

g i j k . i j k<br />

2 2 2<br />

x y z<br />

<br />

<br />

x y z x y z<br />

<br />

= f 2 g + f.g Proved.<br />

Example 64. For a solenoidol <strong>vector</strong> F , show that curl curl curl curl F = 4<br />

F .<br />

(M.D.U., Dec. 2009)<br />

Solution. Since <strong>vector</strong> F is solenoidal, so div F = 0 ... (1)<br />

We know that curl curl F = grad div ( F – 2<br />

F ) ... (2)<br />

Using (1) in (2), grad div F = grad (0) = 0 ... (3)<br />

On putting the value of grad div F in (2), we get<br />

curl curl F = – 2<br />

F ... (4)<br />

Now, curl curl curl curl F = curl curl (– 2<br />

F ) [Using (4)]<br />

= – curl curl ( 2<br />

F ) = – [grad div ( 2<br />

F ) – 2 ( 2<br />

F ) ] [Using (2)]<br />

= – grad ( . 2<br />

F ) + 2<br />

( 2<br />

F ) = – grad ( 2<br />

. F<br />

<br />

) + 4<br />

F [ . F = 0]<br />

= 0 + 4 F = 4 F [Using (1)] Proved.<br />

EXERCISE 5.9<br />

1. Find the divergence and curl of the <strong>vector</strong> field V = (x 2 – y 2 ) i + 2xy j + (y 2 – xy) k .<br />

Ans. Divergence = 4x, Curl = (2y – x) i + y j + 4y k <br />

2. If a is constant <strong>vector</strong> and r is the radius <strong>vector</strong>, prove that<br />

<br />

<br />

(i) ( a. r)<br />

a (ii) div ( r a) 0 (iii) curl ( r <br />

a) 2<br />

a<br />

where r<br />

<br />

= xi yj zk and a a1i a2 j a3k<br />

.<br />

3. Prove that:<br />

(i) .(A) = .A + (.A)<br />

(ii) (A.B) = (A.)B + (B.)A + A × ( × B) + B × ( × A) (R.G.P.V. Bhopal, June 2004)<br />

(iii) × (A × B) = (B.)A – B(.A) – (A.)B + A(.B)<br />

4. If F = (x + y + 1) i + j – (x + y) k , show that F.curl F = 0.<br />

(R.G.P.V. Bhopal, Feb. 2006, June 2004)<br />

Prove that<br />

<br />

<br />

5. ( F) ( ) F ( F )<br />

6. .( F G) G.( F) F.( G)<br />

<br />

7. Evaluate div ( A r)<br />

if curl A <br />

= 0. 8. Prove that curl ( a r)<br />

= 2a


420 Vectors<br />

9. Find div F and curl F where F = grad (x 3 + y 3 + z 3 – 3xyz). (R.G.P.V. Bhopal Dec. 2003)<br />

Ans. div F = 6(x + y + z), curl F = 0<br />

10. Find out values of a, b, c for which v = (x + y + az) i + (bx + 3y – z) j + (3x + cy + z) k <br />

is irrotational.<br />

Ans. a = 3, b = 1, c = –1<br />

11. Determine the constants a, b, c, so that F = (x + 2y + az) i + (bx – 3y – z) j + (4x + cy + 2z) k is<br />

irrotational. Hence find the scalar potential such that F = grad .<br />

(R.G.P.V. Bhopal, Feb. 2005) Ans. a = 4, b = 2, c = 1<br />

Choose the correct alternative:<br />

Potential =<br />

12. The magnitude of the <strong>vector</strong> drawn in a direction perpendicular to the surface<br />

x 2 + 2y 2 + z 2 = 7 at the point (1, –1, 2) is<br />

2 2<br />

x 3y<br />

2<br />

<br />

z 2xy yz 4zx<br />

<br />

2 2<br />

<br />

2<br />

3<br />

(i)<br />

(ii) (iii) 3 (iv) 6 (A.M.I.E.T.E., Summer 2000) Ans. (iv)<br />

3<br />

2<br />

13.If u = x 2 – y 2 + z 2 and V xi yj zk<br />

then ( uV ) is equal to<br />

(i) 5u (ii) 5| V | (iii) 5( u | |) (iv) 5( u | |) (A.M.I.E.T.E., June 2007)<br />

14.A unit normal to x 2 + y 2 + z 2 = 5 at (0, 1, 2) is equal to<br />

1 <br />

(i) ( )<br />

5 i j k<br />

1 <br />

(ii) ( )<br />

5 i j k<br />

1 <br />

(iii) ( 2 )<br />

5 j <br />

k 1 <br />

(iv) ( )<br />

5 i j k<br />

<br />

<br />

V <br />

V <br />

(A.M.I.E.T.E., Dec. 2008)<br />

15.The directional derivative of = x y z at the point (1, 1, 1) in the direction i is:<br />

1<br />

(i) –1 (ii) <br />

1<br />

(iii) 1 (iv)<br />

Ans. (iii)<br />

3<br />

3<br />

(R.G.P.V. Bhopal, II Sem., June 2007)<br />

<br />

<br />

16.If r xi y j zk and r = | r | then (r) is:<br />

(i) (r) r <br />

(ii)<br />

<br />

()<br />

r r<br />

r<br />

(iii)<br />

<br />

()<br />

r r<br />

r<br />

(iv) None of these<br />

Ans. (iii)<br />

(R.G.P.V. Bhopal, II Semester, Feb. 2006)<br />

17. If <br />

r = xi yj zk is position <strong>vector</strong>, then value of (log r) is (U.P., I Sem, Dec 2008)<br />

<br />

<br />

<br />

r<br />

r<br />

(i) <br />

(ii) <br />

(iii) – r (iv) none of the above. Ans. (ii)<br />

2<br />

3<br />

r<br />

r<br />

r<br />

<br />

18. If r xi y j zk and | r |<br />

= r, then div r is:<br />

(i) 2 (ii) 3 (iii) –3 (iv) –2 Ans. (ii)<br />

(R.G.P.V. Bhopal, II Semester, Feb. 2006)<br />

<br />

2<br />

<br />

2<br />

<br />

2<br />

<br />

19. If V xy i 2yx zj 3yz k then curl V at point (1, –1, 1) is<br />

<br />

<br />

<br />

<br />

(i) ( j 2 k)<br />

(ii) ( i 3 k)<br />

(iii) ( i 2 k)<br />

(iv) ( i 2 j k)<br />

(R.G.P.V. Bhopal, II Semester, Feb 2006)<br />

Ans. (iii)<br />

20. If A is such that A = 0 then A is called<br />

(i) Irrotational (ii) Solenoidal (iii) Rotational (iv) None of these<br />

(A.M.I.E.T.E., Dec. 2008)<br />

21. If F is a conservative force field, then the value of curl F is<br />

(i) 0 (ii) 1 (iii) F (iv) –1 (A.M.I.E.T.E., June 2007)


Vectors 421<br />

5.33 LINE INTEGRAL<br />

<br />

Let y,<br />

F ( x, z)<br />

be a <strong>vector</strong> function and a curve AB.<br />

Line integral of a <strong>vector</strong> function F along the curve AB is defined as integral of the component<br />

of F along the tangent to the curve AB.<br />

Component of F along a tangent PT at P<br />

= Dot product of F<br />

<br />

and unit <strong>vector</strong> along PT<br />

<br />

<br />

<br />

dr dr<br />

= F is aunit <strong>vector</strong> along tangent PT<br />

ds ds<br />

<br />

<br />

<br />

<br />

dr<br />

Line integral = F from A to B along the curve<br />

ds<br />

<br />

<br />

<br />

<br />

dr<br />

Line integral = F <br />

ds<br />

c<br />

ds<br />

=<br />

<br />

F<br />

<br />

dr<br />

<br />

<br />

c<br />

<br />

Note (1) Work. If F represents the variable force acting on a particle along arc AB, then the<br />

total work done =<br />

<br />

B<br />

F<br />

<br />

A<br />

<br />

dr<br />

(2) Circulation. If V represents the velocity of a liquid then<br />

V<br />

<br />

c<br />

<br />

dr is called the circulation<br />

of V round the closed curve c.<br />

If the circulation of V round every closed curve is zero then V is said to be irrotational there.<br />

(3) When the path of integration is a closed curve then notation of integration is in place<br />

<br />

of .<br />

22.If 2<br />

[(1 – x) (1 – 2x)] is equal to<br />

(i) 2 (ii) 3 (iii) 4 (iv) 6 (A.M.I.E.T.E., Dec. 2009) Ans. (iii)<br />

23.If R = xi + yj + zk and A <br />

is a constant <strong>vector</strong>, curl ( A R ) is equal to<br />

(i) R (ii) 2 R (iii) A (iv) 2 A (A.M.I.E.T.E., Dec. 2009) Ans. (iv)<br />

1<br />

24. If r is the distance of a point (x, y, z) from the origin, the value of the expression ˆj grad 2<br />

equals<br />

(i) 2 2 2<br />

3<br />

<br />

2 ˆ<br />

( x y z ) ( ˆjz kx)<br />

(ii)<br />

(iii) zero (iv)<br />

<br />

2<br />

Example 65. If a force F 2x yiˆ 3xyj<br />

ˆ displaces a particle in the xy-plane from (0, 0) to<br />

(1, 4) along a curve y = 4 x 2 . Find the work done.<br />

Solution. Work done = F <br />

. dr<br />

<br />

<br />

c r xiˆ yj ˆ<br />

<br />

<br />

2 <br />

<br />

=<br />

ˆ ˆ ˆ ˆ<br />

(2 x yi 3 xyj).( dx i dyj)<br />

dr dxiˆdyj<br />

ˆ<br />

<br />

=<br />

<br />

c<br />

c<br />

2<br />

(2 x y dx 3 xy dy)<br />

3<br />

<br />

2 2 2<br />

( x y z ) 2 ( ˆjz<br />

iz ˆ )<br />

3<br />

<br />

2 2 2 2 ˆ<br />

( x y z ) ( ˆjy<br />

kx)<br />

(AMIETE, Dec. 2010) Ans. (ii)


422 Vectors<br />

Putting the values of y and dy, we get<br />

<br />

1 2 2 2<br />

= [2 x (4 x ) dx 3 x (4 x )8 xdx]<br />

0<br />

5<br />

1<br />

1 4<br />

x 104<br />

= 104 x dx 104<br />

<br />

0 <br />

5 <br />

5<br />

0<br />

<br />

2<br />

y 4 x <br />

<br />

dy 8 xdx <br />

<br />

Ans.<br />

Example 66. Evaluate<br />

<br />

C<br />

<br />

F.<br />

drwhere F x<br />

2ˆ i xyj ˆand C is the boundary of the square in the<br />

plane z = 0 and bounded by the lines x = 0, y = 0, x = a and y = a.<br />

(Nagpur University, Summer 2001)<br />

<br />

<br />

Solution. F. dr F. dr F. dr F. dr F.<br />

dr<br />

C OA AB BC CO<br />

<br />

Here r xiˆ yj ˆ, dr dxiˆ dyj ˆ, F x<br />

2ˆ i xyj ˆ<br />

On OA, y = 0 2<br />

F.<br />

dr x dx<br />

<br />

F.<br />

dr = x 2 dx + xydy ...(1)<br />

<br />

<br />

F . dr =<br />

OA<br />

On AB, x = a dx = 0<br />

(1) becomes<br />

<br />

F.<br />

dr = aydy<br />

2<br />

a<br />

3<br />

F <br />

. dr<br />

<br />

= a y a<br />

aydy a <br />

<br />

<br />

Ab<br />

0<br />

2 2<br />

...(3)<br />

0<br />

On BC, y = a dy = 0<br />

(1) becomes 2<br />

F.<br />

dr x dx<br />

<br />

<br />

<br />

F . dr =<br />

BC<br />

On CO, x = 0, F. dr 0<br />

(1) becomes<br />

<br />

<br />

<br />

<br />

<br />

3<br />

0<br />

3<br />

0 2<br />

x – a<br />

xdx <br />

a<br />

3 3<br />

...(4)<br />

<br />

<br />

a<br />

F . dr = 0 ...(5)<br />

CO<br />

3 3 3 3<br />

On adding (2), (3), (4) and (5), we get F <br />

. dr<br />

<br />

=<br />

a a – a 0 a<br />

Ans.<br />

C 3 2 3 2<br />

Example 67. A <strong>vector</strong> field is given by<br />

<br />

F = (2y 3) iˆ<br />

xzj ˆ ( yz – x) kˆ<br />

. Evaluate F <br />

. dr<br />

<br />

along the path c is x = 2t,<br />

C<br />

y = t, z = t 3 from t = 0 to t = 1. (Nagpur University, Winter 2003)<br />

<br />

<br />

<br />

C<br />

<br />

<br />

a<br />

3 3<br />

a 2<br />

x a<br />

xdx <br />

0<br />

3 3<br />

...(2)<br />

0<br />

Solution. F . dr = (2y 3) dx ( xz) dy ( yz – x)<br />

dz<br />

C C<br />

<br />

3<br />

Since x 2t y t z t <br />

<br />

<br />

dx dy dz 2<br />

2 1 3t<br />

<br />

<br />

dt dt dt <br />

Y<br />

O<br />

B<br />

A<br />

X


Vectors 423<br />

1 3 4 2<br />

= (2 t 3) (2 dt ) (2)( t t ) dt ( t – 2)(3 t t dt ) =<br />

0<br />

<br />

2<br />

t 2 5 3 7 6 4<br />

= 4 6 t t t – t <br />

2 5 7 4 <br />

1<br />

0<br />

<br />

1 4 6 3<br />

(4t 6 2t 3 t – 6 t ) dt<br />

0<br />

2 2 5 3 7 3 4<br />

= 2t 6 t t t – t<br />

5 7 2<br />

<br />

<br />

<br />

2 3 3<br />

= 2 6 – = 7.32857. Ans.<br />

5 7 2<br />

Example 68. The acceleration of a particle at time t is given by<br />

a<br />

= 18 cos 3tiˆ<br />

8sin 2tj ˆ 6 tk ˆ .<br />

If the velocity v and displacement r be zero at t = 0, find v and r at any point t.<br />

2 <br />

Solution. Here, a d r<br />

=<br />

2<br />

dt<br />

On integrating, we have<br />

<br />

= 18 cos 3tiˆ<br />

8sin 2tj ˆ 6 tkˆ<br />

.<br />

<br />

v = dr<br />

iˆ<br />

18 cos 3t dt ˆj 8sin 2t dt k ˆ 6t dt<br />

dt<br />

<br />

2<br />

v = 6sin 3tiˆ 4cos 2tj ˆ 3t kˆ<br />

<br />

c ...(1)<br />

At t = 0, v = 0 <br />

Putting t = 0 and <br />

v = 0 in (1), we get<br />

<br />

0 = 4ˆj <br />

c c 4ˆj<br />

<br />

dr<br />

2<br />

v = 6sin 3tiˆ 4(cos 2t 1) ˆj 3tkˆ<br />

dt<br />

Again integrating, we have<br />

2<br />

r<br />

= iˆ<br />

6sin 3 ˆ 4(cos 2 1) ˆ<br />

t dt j t dt k3t dt<br />

3<br />

r = 2cos 3 tiˆ(2sin 2t 4 t)<br />

ˆj t kˆ<br />

c <br />

...(2)<br />

At, t = 0, r = 0<br />

Putting t = 0 and r = 0 in (2), we get<br />

<br />

<br />

0 = 2iˆC ˆ<br />

1 C1<br />

2i<br />

Hence, r<br />

3<br />

= 2(1cos 3) t iˆ<br />

2(sin 2t 2 t)<br />

ˆj t kˆ<br />

Ans.<br />

Example 69. If A 2 2<br />

(3x 6 y) iˆ<br />

–14yzj ˆ 20 xz kˆ<br />

, evaluate the line integral Adr . from<br />

(0, 0, 0) to (1, 1, 1) along the curve C.<br />

x = t, y = t 2 , z = t 3 . (Uttarakhand, I Semester, Dec. 2006)<br />

Solution. We have,<br />

<br />

<br />

A . dr =<br />

C<br />

=<br />

<br />

<br />

C<br />

C<br />

2 2<br />

[(3x 6 y) iˆ –14yzj ˆ 20 xz kˆ].[ iˆ dx ˆjdy kˆ<br />

dz]<br />

2 2<br />

[(3x 6 y) dx –14yzdy 20 xz dz]<br />

If x = t, y = t 2 , z = t 3 , then points (0, 0, 0) and (1, 1, 1) correspond to t = 0 and t = 1 respectively.<br />

<br />

<br />

Now, A . dr =<br />

C<br />

=<br />

<br />

t 1 2 2 2 3 2 3 2 3<br />

t 0<br />

[(3t 6 t ) d ( t)–14 t t d ( t ) 20 t ( t ) d ( t )]<br />

t 1 2 5 7 2<br />

[9 t dt –14 t .2tdt 20 t .3 t dt]<br />

=<br />

t 0<br />

<br />

1<br />

1 2 6 9<br />

0<br />

1<br />

0<br />

<br />

<br />

(9 t –28 t 60 t ) dt


424 Vectors<br />

=<br />

<br />

3 7 10<br />

t t t<br />

<br />

9 – 28 60 <br />

3 7 10<br />

<br />

<br />

<br />

1<br />

0<br />

= 3 – 4 + 6 = 5 Ans.<br />

Example 70. Evaluate<br />

<br />

S<br />

<br />

2<br />

A. nˆ<br />

ds where A ( x y ) iˆ<br />

– 2xj ˆ 2yzkˆ<br />

and S is the surface of<br />

the plane 2x + y + 2z = 6 in the first octant. (Nagpur University, Summer 2000)<br />

Solution. A <strong>vector</strong> normal to the surface “S” is given by<br />

<br />

(2x y 2) z = ˆ<br />

<br />

ˆ<br />

ˆ <br />

i j k (2x y 2) z 2iˆ ˆj 2kˆ<br />

<br />

x y z<br />

And ˆn = a unit <strong>vector</strong> normal to surface S<br />

Z<br />

2iˆ<br />

ˆj 2kˆ<br />

2 1 2 N<br />

=<br />

i ˆ ˆ j k ˆ<br />

41<br />

4 3 3 3<br />

K<br />

kˆ<br />

2 1 2 2<br />

n<br />

. n ˆ = ˆ <br />

k.<br />

iˆ<br />

ˆj kˆ<br />

–<br />

<br />

3 3 3 3<br />

O<br />

M Y<br />

3<br />

A . nds<br />

dx dy<br />

R<br />

ˆ = A.<br />

nˆ<br />

S<br />

R<br />

k ˆ L<br />

. n<br />

Where R is the projection of S.<br />

X<br />

Now, A . nˆ<br />

= [( x y 2 ) iˆ – 2xj ˆ 2 yzkˆ].<br />

2<br />

iˆ 1 ˆj 2 kˆ<br />

<br />

3 3 3 <br />

2 2 2 4 2 2 4<br />

= ( x y ) – x yz y yz<br />

...(1)<br />

3 3 3 3 3<br />

Putting the value of z in (1), we get<br />

on the plane 2x y 2z<br />

6, <br />

<br />

A.<br />

nˆ<br />

2 2 4 6<br />

2 x y<br />

<br />

<br />

= y y <br />

(6 2 x y)<br />

3 3 2 z <br />

<br />

2<br />

<br />

<br />

A.<br />

nˆ<br />

= 2 y ( y 6 –2 x – y) 4 y (3 – x)<br />

...(2)<br />

3 3<br />

M<br />

Hence,<br />

A <br />

. nds<br />

dx dy<br />

ˆ = A.<br />

n<br />

S<br />

R<br />

| k ˆ<br />

...(3)<br />

. n |<br />

<br />

Putting the value of A.<br />

nˆ<br />

from (2) in (3), we get<br />

A<br />

<br />

. nds 4 3<br />

3 62x<br />

ˆ = (3 – ). 2 (3 )<br />

S y x dx dy y x dydx<br />

R 3 2<br />

<br />

0 0<br />

=<br />

<br />

3<br />

0<br />

<br />

2<br />

y <br />

2(3– x)<br />

<br />

2 <br />

6–2x<br />

0<br />

dx<br />

= <br />

3 (3 – )(6 –2 ) 2 4 3<br />

(3 – )<br />

3<br />

0 0<br />

4<br />

3<br />

x x dx x dx<br />

(3 – x)<br />

<br />

= 4. –(0–81) 81<br />

4(–1) <br />

0<br />

<br />

iy ˆ ˆjx<br />

Example 71. Compute F. dr,<br />

where F <br />

c<br />

2 2<br />

and c is the circle x 2 + y 2 = 1 traversed<br />

x y<br />

counter clockwise.<br />

O<br />

2x + 3y = 6<br />

L<br />

Ans.<br />

X


Vectors 425<br />

Solution. r = ix ˆ ˆjy kz ˆ , dr idx ˆ ˆjdy kdz ˆ<br />

<br />

F . dr<br />

iy ˆ ˆjx<br />

= ( ˆ ˆ ˆ )<br />

c idx jdy kdz<br />

c 2 2<br />

x y<br />

ydx xdy<br />

= ( ydx xdy)<br />

c 2 2<br />

x y<br />

c<br />

Parametric equation of the circle are x = cos , y = sin .<br />

Putting x = cos , y = sin , dx = – sin d , dy = cos d in (1), we get<br />

Fd<br />

<br />

r<br />

2<br />

= sin ( sin d ) cos (cos d )<br />

C <br />

=<br />

...(1) [ x 2 + y 2 = 1]<br />

0<br />

2<br />

2 2<br />

2<br />

= 2 0<br />

(sin cos ) d d <br />

0 0<br />

<br />

2 Ans.<br />

<br />

2 3 2 2 2<br />

Example 72. Show that the <strong>vector</strong> field F 2 x( y z ) iˆ<br />

2x yj ˆ 3xzkˆ<br />

is conservative.<br />

Find its scalar potential and the work done in moving a particle from (–1, 2, 1) to (2, 3, 4).<br />

(A.M.I.E.T.E. June 2010, 2009)<br />

Solution. Here, we have<br />

<br />

2 3 ˆ 2 ˆ 2 2<br />

F 2 x( y z ) i 2x yj 3x z kˆ<br />

<br />

<br />

Curl F F<br />

iˆ<br />

ˆj kˆ<br />

<br />

<br />

x y z<br />

2 3 2 2 2<br />

2 xy ( z ) 2x y 3x z<br />

(00) i(6xz 6 xz ) ˆj(4xy 4 xy)<br />

k = 0<br />

Hence, <strong>vector</strong> field F is irrotational.<br />

To find the scalar potential function <br />

<br />

F <br />

<br />

d dx dy<br />

dz ˆ<br />

<br />

ˆ<br />

ˆ<br />

<br />

i j k . idx ˆ ˆjdy kdz ˆ<br />

<br />

x y z x y z<br />

<br />

<br />

ˆ<br />

<br />

ˆ<br />

<br />

i j kˆ<br />

<br />

<br />

<br />

. d r . d r F.<br />

dr<br />

x y z <br />

2 3 2 2 2<br />

2 xy ( z ) iˆ2x yj ˆ3 x z kˆ<br />

( idx ˆ ˆjdy kdz<br />

ˆ )<br />

<br />

<br />

=<br />

2 2 ˆ<br />

<br />

<br />

2 3 2 2 2<br />

2 x( y z ) dx2x ydy3x z dz<br />

2 3 2 2 2<br />

2 ( ) 2 3 <br />

<br />

xy z dx x ydy x z dz C<br />

2 2 3 2 2<br />

(2 xy dx 2 x ydy ) (2 xz dx 3 x z dz ) + C = x 2 y 2 + x 2 z 3 + C<br />

Hence, the scalar potential is x 2 y 2 + x 2 z 3 + C<br />

Now, for conservative field<br />

Work done =<br />

<br />

(2,3, 4) (2,3, 4)<br />

<br />

F.<br />

d r d<br />

( 1,2,1) ( 1, 2,1)<br />

<br />

<br />

(2,3,4) (2,3,4)<br />

2 2 2 3<br />

x y x z c<br />

( 1,2,1) <br />

( 1,2,1)<br />

<br />

= (36 + 256) – (2 – 1) = 291 Ans.


426 Vectors<br />

<br />

Example 73. A <strong>vector</strong> field is given by F (sin y) iˆ x(1 cos y) ˆj.<br />

Evaluate the line integral<br />

over a circular path x 2 + y 2 = a 2 , z = 0. `(Nagpur University, Winter 2001)<br />

Solution. We have,<br />

F<br />

<br />

C<br />

Work done = . dr<br />

F<br />

<br />

C<br />

<br />

= [(sin ) ˆ (1 cos ) ˆ].[ ˆ ˆ<br />

y i x y j dxi dyj]<br />

( z = 0 hence dz = 0)<br />

<br />

C<br />

. d r = sin ydx x (1 cos y ) dy (sin y dx x cos y dy xdy )<br />

C<br />

C<br />

<br />

= d ( xsin y)<br />

x dy<br />

C<br />

(where d is differential operator).<br />

The parametric equations of given path<br />

x 2 + y 2 = a 2 are x = a cos , y = a sin ,<br />

Where varies form 0 to 2<br />

F<br />

<br />

C<br />

. d r =<br />

=<br />

<br />

<br />

<br />

C<br />

2<br />

2<br />

d [ a cos sin ( a sin )] a cos . a cos d <br />

0 0<br />

2<br />

2<br />

2 2<br />

d [ a cos sin ( a sin )] a cos .<br />

d <br />

0 0<br />

= [ a cos sin ( asin )] a cos d<br />

<br />

<br />

<br />

<br />

2<br />

2<br />

2 2<br />

0 0<br />

2<br />

2 21cos 2 a sin 2 <br />

= 0 a <br />

d<br />

0<br />

<br />

2 2<br />

<br />

2<br />

<br />

0<br />

2<br />

a<br />

2<br />

= .2a<br />

2<br />

Example 74. Determine whether the line integral<br />

Ans.<br />

2 2 2 2<br />

(2 xyz ) dx ( x z z cos yz) dy (2x yz y cos yz)<br />

dz is independent of the path of<br />

<br />

integration ? If so, then evaluate it from (1, 0, 1) to<br />

<br />

0, ,1 .<br />

2 <br />

2 2 2 2<br />

Solution. (2 xy z ) dx ( x z z cos yz) dy (2x yz y cos yz)<br />

dz<br />

=<br />

<br />

c<br />

<br />

c<br />

2 2 2 2<br />

[(2 xy ziˆ) ( x z z cos yz) ˆj (2x yz y cos yz) kˆ].( idx ˆ ˆjdy kdz ˆ )<br />

= Fdr<br />

<br />

<br />

c<br />

This integral is independent of path of integration if<br />

F<br />

= 0<br />

iˆ<br />

ˆj kˆ<br />

F =<br />

F<br />

<br />

<br />

x y z<br />

2 2 2 2<br />

2xyz x z z cos yz 2x yz y cos yz<br />

= (2x 2 z + cos yz – yz sin yz – 2x 2 z – cos yz + yz sin yz) = iˆ<br />

–(4 xyz – 4 xyz) ˆj (2 xz –2 xz ) k<br />

= 0<br />

Hence, the line integral is independent of path.<br />

d = dx <br />

dy <br />

dz<br />

x y z<br />

(Total differentiation)<br />

2<br />

2 2 ˆ


Vectors 427<br />

<br />

B A<br />

<br />

ˆ<br />

ˆ<br />

=<br />

ˆ<br />

<br />

ˆ<br />

<br />

<br />

i j k ( idx ˆ ˆjdy kdz)<br />

= dr F<br />

d r<br />

x y z<br />

2 2 2 2<br />

= [(2 xyz ) iˆ( x z z cos yz) ˆj (2x yz ycos yz) kˆ]. ( idx ˆ ˆjdy kdz ˆ )<br />

= 2xyz 2 dx + (x 2 z 2 + z cos y z) dy + (2x 2 yz + y cos yz) dz<br />

= [(2x dx) yz 2 + x 2 (dy) z 2 + x 2 y (2z dz)] + [(cos yz dy) z + (cos yz dz) y]<br />

= d (x 2 yz 2 ) + d (sin yz)<br />

=<br />

<br />

<br />

2 2 2 2<br />

d ( x yz ) d (sin yz) x yz sin yz<br />

= (B) – (A)<br />

2 2 2 2<br />

= [ x yz sin yz] [ x yz sin yz]<br />

(1, 0,1) =<br />

<br />

0 sin ( 1) [0 0]<br />

(0, ,1)<br />

<br />

2<br />

2<br />

<br />

<br />

<br />

= 1 Ans.<br />

<br />

Example 75. Evaluate A. nˆ<br />

dS, where A 18 ziˆ<br />

–12ˆj 3ykˆ<br />

<br />

and S is the part of the<br />

S<br />

plane 2x + 3y + 6z = 12 included in the first octant. (Uttarakhand, I semester, Dec. 2006)<br />

Solution. Here, A = 18 ziˆ<br />

–12ˆj 3ykˆ<br />

Given surface f (x, y, z) = 2x + 3y + 6z – 12<br />

<br />

ˆ<br />

Normal <strong>vector</strong> = f =<br />

ˆ<br />

<br />

ˆ<br />

<br />

i j k (2x 3y 6 z –12) = 2iˆ3ˆj 6kˆ<br />

x y z<br />

ˆn = unit normal <strong>vector</strong> at any point (x, y, z) of 2x + 3y + 6z = 12<br />

2iˆ3ˆj 6kˆ<br />

1<br />

=<br />

(2iˆ3ˆj 6 kˆ<br />

)<br />

4936<br />

7<br />

dx dy dx dy dxdy 7<br />

dS = dx dy<br />

nˆ . kˆ 1<br />

ˆ ˆ ˆ ˆ 6 6<br />

(2i 3 j 6 k).<br />

k<br />

7 7<br />

Now, A <br />

. ndS<br />

1 7<br />

<br />

ˆ = (18 ˆ –12 ˆ 3 ˆ). (2ˆ 3ˆ<br />

6 ˆ<br />

zi j yk i j k)<br />

dx dy<br />

7 6<br />

dx dy<br />

= (36 z –36<br />

18 y)<br />

= (6 z –6<br />

3 y)<br />

dx dy<br />

6 <br />

Putting the value of 6z = 12 – 2x – 3y, we get<br />

Y<br />

=<br />

=<br />

=<br />

=<br />

1<br />

6 (12 –2 x)<br />

3<br />

0 0<br />

<br />

<br />

(12–2 x –3 y –6<br />

3 y)<br />

dxdy<br />

1<br />

6 (12 –2 x)<br />

3<br />

0 0<br />

1<br />

6 (12 –2 x)<br />

3<br />

(6 –2 x)<br />

dx<br />

0 0<br />

1<br />

6<br />

(12–2 x )<br />

(6 –2 x) dx( y)<br />

3<br />

0 0<br />

<br />

(6 –2 x)<br />

dxdy<br />

dy<br />

6 1<br />

1 6<br />

2<br />

= (6 –2 x) (12 –2 x)<br />

dx = (4 x –36x 72) dx<br />

0<br />

0<br />

3<br />

3<br />

<br />

3<br />

6<br />

14x<br />

2<br />

<br />

–18x<br />

72x<br />

3<br />

3<br />

= 1 [4 36 2–18 36 72 6] = 72 [4 –9 6] 24<br />

Ans.<br />

3<br />

3<br />

0<br />

O<br />

B<br />

2x + 3y = 12<br />

A<br />

X


428 Vectors<br />

EXERCISE 5.10<br />

1. Find the work done by a force yiˆ xj ˆ which displaces a particle from origin to a point ( iˆ ˆj).<br />

Ans. 1<br />

2. Find the work done when a force 2 2<br />

F ( x – y x) iˆ(2 xy y)<br />

ˆj<br />

moves a particle from origin to<br />

(1, 1) along a parabola y 2 = x. Ans. 2 3<br />

3. Show that<br />

<br />

<br />

3 ˆ <br />

2 ˆ <br />

2 ˆ is a conservative field. Find its scalar potential such that<br />

V (2 xy z ) i x j 3xz k<br />

V = grad . Find the work done by the force V in moving a particle from (1, – 2, 1) to (3, 1, 4).<br />

Ans. x 2 y + xz 3 , 202<br />

4. Show that the line integral 2<br />

(2xy 3) dx ( x 4 z) dy 4ydz<br />

<br />

c<br />

where c is any path joining (0, 0, 0) to (1, – 1, 3) does not depend on the path c and evaluate the line<br />

integral. Ans. 14<br />

2 2<br />

x y<br />

5. Find the work done in moving a particle once round the ellipse 1 , z = 0, under the field of<br />

25 16<br />

force given by F = (2x – y + z) î + (x + y – z2 ) ĵ + (3x – 2y + 4z) k ˆ. Is the field of force conservative?<br />

(A.M.I.E.T.E., Winter 2000) Ans. 40 <br />

6.<br />

If 4<br />

= (y 2 – 2xyz 3 ) î + (3 + 2xy – x 2 z 3 ) ĵ + (z 3 – 3x 2 yz 2 z 2 2 3<br />

) k ˆ, find . Ans. 3y xy x yz<br />

<br />

4<br />

R . dR<br />

7. is independent of the path joining any two point if it is. (A.M.I.E.T.E., June 2010)<br />

C<br />

Ans. (i)<br />

(i) irrotational field (ii) solenoidal field (iii) rotational field (iv) <strong>vector</strong> field.<br />

5.34 SURFACE INTEGRAL<br />

A surface r = f(u, v) is called smooth if f (u, v) posses continous<br />

first order partial derivative.<br />

Let F be a <strong>vector</strong> function and S be the given surface.<br />

Surface integral of a <strong>vector</strong> function F over the surface S is defined<br />

as the integral of the components of F <br />

along the normal to the<br />

surface.<br />

Component of F along the normal<br />

= F . ˆn , where n is the unit normal <strong>vector</strong> to an element ds and<br />

grad f<br />

ˆn =<br />

|grad f |<br />

Surface integral of F over S<br />

Note. (1) Flux =<br />

<br />

ds =<br />

dx dy<br />

( nˆ<br />

kˆ<br />

)<br />

= Fn ˆ<br />

= ( Fnˆ<br />

) ds<br />

<br />

S<br />

<br />

<br />

( F nˆ<br />

) ds where, F represents the velocity of a liquid.<br />

S<br />

<br />

<br />

If<br />

( Fn ˆ)<br />

ds = 0, then F is said to be a solenoidal <strong>vector</strong> point function.<br />

S<br />

<br />

<br />

Example 76. Evaluate<br />

( ˆ ˆ ˆ<br />

yzi zxj xyk)<br />

ds where S is the surface of the sphere<br />

S<br />

x 2 + y 2 + z 2 = a 2 in the first octant. (U.P., I Semester, Dec. 2004)<br />

Solution. Here, = x 2 + y 2 + z 2 – a 2


Vectors 429<br />

Vector normal to the surface = =<br />

iˆ<br />

ˆj kˆ<br />

<br />

x y z<br />

<br />

ˆ 2 2 2 2<br />

ˆ<br />

=<br />

ˆ<br />

<br />

ˆ<br />

<br />

i j k ( x y z a ) 2xiˆ 2yj ˆ 2zk<br />

x y z<br />

ˆn<br />

2xiˆ 2yj ˆ 2zkˆ<br />

xiˆ yj ˆ zkˆ<br />

= <br />

<br />

| | 2 2 2 2 2 2<br />

4x 4y 4z x y z<br />

xiˆ<br />

yj ˆ zkˆ<br />

=<br />

[ x 2 + y 2 + z 2 = a 2 ]<br />

a<br />

Here, F = yz iˆ<br />

zxj ˆ xy kˆ<br />

<br />

ˆ ˆ ˆ 3<br />

Fn ˆ = ( ˆ ˆ ˆ<br />

xi yj zk xyz<br />

yz i zxj xy k)<br />

<br />

<br />

a <br />

<br />

a<br />

Now, F<br />

n ˆ ds<br />

<br />

2 2<br />

dx dy a a x 3xyz dx dy<br />

S<br />

= ( Fn ˆ)<br />

<br />

S<br />

| kˆ<br />

. nˆ<br />

<br />

|<br />

00<br />

z <br />

a <br />

a <br />

a<br />

2<br />

x<br />

2<br />

2<br />

a a<br />

2<br />

x<br />

2<br />

a y <br />

= 3 xy dy dx 3 x dx<br />

0<br />

0 0 2 <br />

<br />

=<br />

0<br />

2 2 4<br />

a<br />

4 4 4<br />

3 a 2 2 3 a x x 3 a a 3a<br />

2<br />

x a x dx 0<br />

2 <br />

2 4 <br />

<br />

2 2 4 <br />

8<br />

0 <br />

( ) .<br />

3<br />

Example 77. Show that Fnds ˆ ,<br />

S<br />

where<br />

2<br />

F = 4 xz î – y 2 ĵ + yz ˆk<br />

and S is the surface of the cube bounded by the planes,<br />

x= 0, x = 1, y = 0, y = 1, z = 0, z = 1.<br />

Solution.<br />

<br />

<br />

<br />

= <br />

ˆ<br />

Fnds ˆ<br />

S<br />

Fnds<br />

<br />

<br />

OABC<br />

Fnds ˆ<br />

Fnds<br />

ˆ<br />

<br />

DEFG<br />

<br />

<br />

OAGF<br />

<br />

Fnds ˆ<br />

Fnds<br />

ˆ<br />

BCED<br />

<br />

Fnds<br />

<br />

OCEF<br />

<br />

ABDG<br />

ˆ<br />

...(1)<br />

Now,<br />

<br />

DEFG<br />

Fnds<br />

<br />

<br />

OABC<br />

<br />

2<br />

= (4 ˆ ˆ ˆ<br />

xzi y j yz k)( k)<br />

dx dy = 1 1<br />

OABC<br />

2<br />

(4 xziˆ<br />

y ˆj yz kˆ)<br />

kˆ<br />

dx dy<br />

=<br />

<br />

DEFG<br />

2<br />

1<br />

1 y<br />

<br />

1<br />

= dx [ x]<br />

0<br />

0<br />

0<br />

2 ˆ<br />

1 1<br />

<br />

yz dx dy y (1) dxdy<br />

0 0<br />

1 1<br />

<br />

2 2 2<br />

(4 ˆ ˆ<br />

xz i y j yzk) ( j)<br />

dxdz =<br />

OAGF<br />

0 0<br />

Ans.<br />

S.No. Surface Outward ds<br />

normal<br />

1 OABC – k dx dy z = 0<br />

2 DEFG k dx dy z = 1<br />

3 OAGF – j dx dz y = 0<br />

4 BCED j dx dz y = 1<br />

5 ABDG i dy dz x = 1<br />

6 OCEF – i dy dz x = 0<br />

yz dx dy 0<br />

(as z = 0)<br />

2<br />

y dxdz 0<br />

(as y = 0)<br />

OAGF


430 Vectors<br />

(4 xz iˆ y ˆj yzk)<br />

ˆ<br />

jdxdz =<br />

BCED<br />

2 ˆ<br />

=<br />

2 ˆ<br />

<br />

BCED<br />

1 dx 1 dz x 1 1<br />

0 0<br />

0 z 0<br />

(4 xziˆ y ˆj yzk)<br />

idydz<br />

ˆ<br />

<br />

=<br />

ABDG<br />

=<br />

2<br />

( y ) dxdz<br />

(as y = 1)<br />

( ) ( ) 1<br />

<br />

2<br />

1<br />

1<br />

z 1<br />

<br />

0 <br />

4( y) <br />

4(1) <br />

2<br />

2 <br />

2<br />

(4 ˆ ˆ )( ˆ<br />

xz i y j yzk i)<br />

dydz =<br />

OCEF<br />

On putting these values in (1), we get<br />

=<br />

F<br />

nds ˆ<br />

S<br />

1. Evaluate A <br />

. n ˆ ds ,<br />

S<br />

2 ˆ<br />

0<br />

1 1<br />

<br />

4xz dy dz 4 (1) zdydz<br />

1 1<br />

0 0<br />

1<br />

0 01 2 0 = 3 2<br />

2<br />

EXERCISE 5.11<br />

0 0<br />

where A = 2<br />

( x y ) iˆ<br />

2xj ˆ 2yzkˆ<br />

4xz dy dz 0<br />

(as x = 0)<br />

Proved.<br />

and S is the surface of the plane<br />

2x + y + 2z = 6 in the first octant. Ans. 81<br />

2. Evaluate A <br />

. n ˆ ds ,<br />

S<br />

where A = 2<br />

ziˆ xj ˆ 3y zkˆ<br />

and S is the surface of the cylinder x 2 + y 2 = 16<br />

included in the first octant between z = 0 and z = 5. Ans. 90<br />

3. If r =<br />

tiˆ t ˆ j ( t 1) k<br />

2<br />

ˆ<br />

and S =<br />

2ˆ<br />

tkˆ<br />

evaluate<br />

2ti<br />

6 ,<br />

2 <br />

r S dt.<br />

Ans. 12<br />

0<br />

4. Evaluate FndS<br />

<br />

ˆ , where, F = 18 ziˆ12 ˆj 3ykˆ<br />

and S is the surface of the plane 2x + 3y + 6z = 12<br />

S<br />

in the first octant. Ans. 24<br />

5. Evaluate Fnds<br />

<br />

ˆ ,<br />

2<br />

where, F = 2yxiˆ yzj ˆ xkˆ<br />

over the surface S of the cube bounded by the<br />

S<br />

coordinate planes and planes x = a, y = a and z = a.<br />

<br />

2<br />

6. If F 2yiˆ<br />

3ˆj xkˆ<br />

and S is the surface of the parabolic cylinder y<br />

2<br />

= 8x in the first octant bounded<br />

<br />

by the planes y = 4, and z = 6, then evaluate FndS ˆ .<br />

Ans. 132<br />

S<br />

5.35 VOLUME INTEGRAL<br />

Let F be a <strong>vector</strong> point function and volume V enclosed by a closed surface.<br />

The volume integral =<br />

F <br />

dv<br />

V<br />

Example 78. If F = 2 z î – x ĵ + y ˆk , evaluate<br />

the surfaces<br />

x = 0, y = 0, x = 2, y = 4, z = x 2 , z = 2.<br />

Solution.<br />

(2 ziˆ<br />

xj ˆ yk)<br />

dx dy dz<br />

<br />

= ˆ<br />

<br />

=<br />

=<br />

F <br />

dv<br />

V<br />

2 4 2<br />

0 0 x<br />

2<br />

dx dy (2 ziˆ<br />

xj ˆ ykˆ<br />

) dz<br />

=<br />

<br />

2 4 4 3 2<br />

dx dy [4iˆ 2xj ˆ 2 ykˆ<br />

x iˆ x ˆj x ykˆ]<br />

<br />

0 0<br />

Ans.<br />

1 4<br />

2 a<br />

Fdv<br />

<br />

where, v is the region bounded by<br />

V<br />

<br />

2 4 2<br />

2<br />

dx ˆ ˆ ˆ<br />

0 dy [ z i xzj yzk] 0<br />

x<br />

2


Vectors 431<br />

=<br />

<br />

2<br />

0<br />

<br />

2 2<br />

ˆ ˆ 2 ˆ 4 ˆ 3<br />

4 2<br />

ˆ<br />

x y<br />

dx yi xyj yk x yi x yj kˆ<br />

<br />

<br />

<br />

2 <br />

2 4 3 2<br />

= ˆ ˆ ˆ ˆ ˆ ˆ<br />

(16i 8xj 16k 4xi 4x j 8 xk)<br />

dx<br />

0<br />

2<br />

<br />

5 3<br />

2 4 4 8<br />

= ˆ ˆ ˆ x<br />

16 4 16 ˆ ˆ<br />

x <br />

xi x j xk i x j kˆ<br />

<br />

<br />

5 3 0<br />

128 64<br />

= 32 ˆ 16 ˆ 32 ˆ ˆ 16 ˆ ˆ 32 iˆ<br />

32kˆ<br />

i j k i j k = = 32 (3iˆ<br />

5 kˆ<br />

)<br />

5 3 5 3 15<br />

EXERCISE 5.12<br />

1. If F <br />

= 2<br />

(2x 3) z iˆ2xyj ˆ 4 xkˆ<br />

, then evaluate FdV , where V is bounded by the plane<br />

V<br />

4<br />

0<br />

Ans.<br />

x = 0, y = 0, z = 0 and 2x + 2y + z = 4. Ans. 8 3<br />

2. Evaluate dV ,<br />

V where = 45 x 2 y and V is the closed region bounded by the planes<br />

4x + 2y + z = 8, x = 0, y = 0, z = 0 Ans. 128<br />

3. If F = (2x 2 <br />

– 3z) iˆ<br />

2xyj ˆ 4 xkˆ<br />

, then evaluate FdV<br />

, where V is the closed region bounded<br />

V<br />

8<br />

by the planes x = 0, y = 0, z = 0 and 2x + 2y + z = 4.<br />

Ans. ( ˆ ˆ)<br />

3 j k<br />

4. Evaluate (2 x<br />

y) dV , where V is closed region bounded by the cylinder z = 4 – x 2 and the planes<br />

V<br />

x = 0, y = 0, y = 2 and z = 0. Ans. 80 3<br />

5. If F 2<br />

= 2 ˆ ˆ ˆ<br />

<br />

xz i xj y k,<br />

evaluate<br />

F dV over the region bounded by the surfaces x = 0, y = 0,<br />

y = 6 and z = x 2 , z = 4. Ans. (16iˆ3ˆj 48 kˆ<br />

)<br />

5.36 GREEN’S THEOREM (For a plane)<br />

<br />

Statement. If (x, y), (x, y), and be continuous functions over a region R bounded<br />

y<br />

x<br />

by simple closed curve C in x – y plane, then<br />

( dx<br />

dy)<br />

= <br />

<br />

dx dy<br />

C<br />

R<br />

<br />

x<br />

y<br />

(AMIETE, June 2010, U.P., I Semester, Dec. 2007)<br />

Proof. Let the curve C be divided into two curves C 1<br />

(ABC) and C 1<br />

(CDA).<br />

Let the equation of the curve C 1<br />

(ABC) be y = y 1<br />

(x) and equation of the curve C 2<br />

(CDA) be<br />

y = y 2<br />

(x).<br />

Let us see the value of<br />

<br />

dx dy = xc<br />

y<br />

y<br />

2<br />

( x)<br />

<br />

dy dx<br />

R y<br />

xa y y ( x)<br />

<br />

1 y<br />

<br />

= c 2 ( )<br />

( , )<br />

y <br />

x y<br />

y x<br />

dx<br />

a<br />

y y ( x)<br />

1<br />

c<br />

a<br />

c<br />

= ( x, y2) ( x, y1)<br />

dx = ( x, y2) dx ( x, y1)<br />

dx<br />

a<br />

a<br />

c<br />

= <br />

<br />

( x, y2) dx ( x, y1)<br />

dx<br />

<br />

c<br />

a<br />

<br />

= <br />

<br />

( x , y ) dx ( x , y ) dx<br />

<br />

<br />

c2 c1<br />

<br />

= – ( , )<br />

<br />

c<br />

c<br />

<br />

a<br />

x y dx


432 Vectors<br />

Thus,<br />

dx<br />

c<br />

=<br />

Similarly, it can be shown that<br />

<br />

dx dy<br />

...(1)<br />

R y<br />

<br />

dy<br />

c<br />

= dx dy ...(2)<br />

x<br />

On adding (1) and (2), we get<br />

<br />

<br />

( dx<br />

dy)<br />

= <br />

dx dy<br />

R<br />

Proved.<br />

x<br />

y<br />

<br />

Note. Green’s Theorem in <strong>vector</strong> form<br />

where,<br />

<br />

<br />

Fdr<br />

<br />

= ( F ) ˆ<br />

c <br />

R<br />

k dR<br />

<br />

F iˆ ˆj, r xiˆ yj ˆ,<br />

kˆ<br />

is a unit <strong>vector</strong> along z-axis and dR = dx dy.<br />

<br />

Example 79. A <strong>vector</strong> field F is given by F sin yiˆ x (1 cos y) ˆj.<br />

Evaluate the line integral Fdr<br />

<br />

where C is the circular path given by x 2 + y 2 = a 2 .<br />

C<br />

<br />

Solution. F sin yiˆ x(1 cos y)<br />

ˆj<br />

Fdr<br />

<br />

<br />

C<br />

= [sin ˆ (1 cos ) ˆ] ( ˆ ˆ<br />

yi x y j idx jdy)<br />

C<br />

= sin ydx x (1 cos y)<br />

dy<br />

C<br />

On applying Green’s Theorem, we have<br />

<br />

<br />

( dx dy)<br />

c<br />

= <br />

dx dy<br />

s<br />

<br />

x<br />

y<br />

<br />

= [(1<br />

cos y) cos y]<br />

dx dy<br />

s<br />

where s is the circular plane surface of radius a.<br />

= dx dy<br />

s<br />

= Area of circle = a 2 . Ans.<br />

2 2<br />

Example 80. Using Green’s Theorem, evaluate ( x ydx x dy),<br />

where c is the boundary<br />

described counter clockwise of the triangle with vertices (0, 0), (1, 0), (1, 1).<br />

(U.P., I Semester, Winter 2003)<br />

Solution. By Green’s Theorem, we have<br />

Y<br />

A<br />

<br />

<br />

(1, 1)<br />

=<br />

( dx dy)<br />

c<br />

= dx dy<br />

R<br />

<br />

x<br />

y<br />

<br />

2 2<br />

( x y dx <br />

2<br />

x dy)<br />

= (2 x x ) dxdy<br />

c<br />

1 2<br />

(2 x x ) dx dy =<br />

0 0<br />

x<br />

=<br />

<br />

<br />

R<br />

1 2<br />

0<br />

(2 x x ) dx[ y]x<br />

1 2<br />

(2 x x )( x ) dx =<br />

0<br />

2 1<br />

= <br />

3 4 = 5<br />

12<br />

Example 81. State and verify Green’s Theorem in the plane for<br />

c<br />

0<br />

1 2 3<br />

(2 x x ) dx =<br />

0<br />

<br />

2 2<br />

(0, 0)<br />

(1, 0)<br />

<br />

3 4<br />

2 x x <br />

<br />

<br />

3 4 <br />

<br />

1<br />

0<br />

Ans.<br />

(3 x –8 y ) dx (4 y – 6 xy)<br />

dy where C is the boundary of the region bounded by x 0, y 0 and 2x – 3y = 6.<br />

(Uttarakhand, I Semester, Dec. 2006)<br />

O<br />

y = x<br />

X


Vectors 433<br />

Solution. Statement: See Article 24.4 on page 576.<br />

Here the closed curve C consists of straight lines OB, BA and AO, where coordinates of A and<br />

B are (3, 0) and (0, – 2) respectively. Let R be the region bounded by C.<br />

Then by Green’s Theorem in plane, we have<br />

<br />

2 2<br />

[(3 x –8 y ) dx (4 y –6 xy) dy ]<br />

=<br />

<br />

<br />

x<br />

<br />

y<br />

2 2<br />

(4 y –6 xy)– (3 x –8 y ) dxdy<br />

R<br />

<br />

<br />

...(1)<br />

<br />

= (– 6y 16 y) dx dy 10 ydxdy<br />

R<br />

2<br />

3 0 3 y<br />

= 10 dx 1 ydy 10 dx <br />

<br />

x<br />

2 <br />

0 (2 –6) 0<br />

3 1<br />

(2 x –6)<br />

3<br />

<br />

=<br />

3<br />

5 (2 x – 6) 5 3<br />

= – – (0 6)<br />

9 3<br />

2 54<br />

0<br />

Now we evaluate L.H.S. of (1) along OB, BA and AO.<br />

Along OB, x = 0, dx = 0 and y varies form 0 to –2.<br />

Along BA, x = 1 (6 3 y),<br />

dx 3 dy and y varies from –2 to 0.<br />

2 2<br />

and along AO, y = 0, dy = 0 and x varies from 3 to 0.<br />

L.H.S. of (1) =<br />

=<br />

<br />

OB<br />

<br />

2 2<br />

[(3 x – 8 y ) dx (4 y –6 xy) dy]<br />

3<br />

2 2 2 2<br />

BA<br />

R<br />

<br />

0<br />

<br />

<br />

=<br />

5 3<br />

– (2 – 6)<br />

9<br />

dx x<br />

0<br />

5<br />

– (216) –20 ...(2)<br />

54<br />

[(3 x – 8 y ) dx (4 y –6 xy) dy] [(3 x – 8 y ) dx (4 x –6 xy) dy ]<br />

<br />

<br />

AO<br />

2 2<br />

[(3 x – 8 y ) dx (4 y –6 xy) dy ]<br />

–2 0 3 2 23<br />

<br />

0<br />

2<br />

= 4 ydy (6 3 y) –8 y dy [4 y –3(6 3 y) y] dy 3x dx<br />

0 –2<br />

<br />

4<br />

<br />

2<br />

<br />

<br />

3<br />

=<br />

2 –2 0 9<br />

2 2 2 3 0<br />

[2 y ] <br />

<br />

0<br />

<br />

(63 y) –12y 4 y –18 y –9 y dy ( x )<br />

3<br />

–2<br />

8<br />

<br />

<br />

<br />

0 9<br />

2 2 <br />

= 2[4] <br />

(6 3 y) –21 y –14 y dy (0 –27)<br />

–2<br />

8<br />

<br />

<br />

<br />

0<br />

<br />

3<br />

9(6 3 y) 3 2<br />

216<br />

3 2<br />

=<br />

8 – 7 y – 7 y –27 –19 7(– 2) 7(–2)<br />

8 3 3 8<br />

<br />

<br />

<br />

–2<br />

= – 19 + 27 – 56 + 28 = – 20 ...(3)<br />

With the help of (2) and (3), we find that (1) is true and so Green’s Theorem is verified.<br />

2 2 2 2<br />

Example 82. Apply Green’s Theorem to evaluate [(2 x y ) dx ( x y ) dy],<br />

where C<br />

is the boundary of the area enclosed by the x-axis and the upper half of circle x 2 + y 2 = a 2 .<br />

(M.D.U. Dec. 2009, U.P., I Sem., Dec. 2004)<br />

2 2 2 2<br />

Solution. [(2 x y ) dx ( x y ) dy]<br />

<br />

C<br />

By Green’s Theorem, we’ve ( dx<br />

dy)<br />

=<br />

C<br />

<br />

S<br />

C<br />

<br />

<br />

dx dy<br />

x<br />

y<br />

2


434 Vectors<br />

=<br />

a a<br />

2<br />

x<br />

2<br />

<br />

a<br />

0<br />

a a<br />

2<br />

x<br />

2<br />

a<br />

0<br />

2 2 2 2 <br />

( x y ) (2 x y ) dx dy<br />

x<br />

y<br />

<br />

a a<br />

2<br />

x<br />

2<br />

= (2x<br />

2 y)<br />

dx dy = 2 dx ( x y ) dy<br />

a<br />

2<br />

x<br />

2<br />

2<br />

= 2 a y <br />

dx<br />

xy <br />

a<br />

2 = 2<br />

<br />

=<br />

<br />

a<br />

0<br />

<br />

a<br />

a<br />

2 2 a 2 2<br />

a 2 x a x dx ( a x ) dx<br />

a<br />

<br />

a<br />

0<br />

<br />

2 2<br />

2 2 a x <br />

<br />

x a x <br />

dx<br />

2 <br />

<br />

<br />

<br />

<br />

<br />

<br />

3<br />

a<br />

3<br />

2 x<br />

3 a<br />

2 2<br />

= 0 2 a ( a x ) dx = 2 a x<br />

2<br />

a<br />

3<br />

<br />

0<br />

3 3 <br />

= 4a<br />

3<br />

0<br />

<br />

a<br />

a<br />

( ) 2 ( ) , is even<br />

<br />

<br />

<br />

<br />

0, is odd <br />

f x dx f x dx f<br />

a 0<br />

<br />

f<br />

Example 83. Evaluate y x<br />

dx dy,<br />

where C C<br />

C 2 2 2 2<br />

1UC2<br />

x y x y<br />

with C : x2 + y2 = 1<br />

1<br />

and C 2<br />

: x = 2, y = 2. (Gujarat, I Semester, Jan 2009)<br />

y<br />

x<br />

Solution. dx <br />

C 2 2 2 2<br />

x y x y<br />

dy<br />

Y<br />

x y <br />

= <br />

<br />

dx dy<br />

x<br />

2 2<br />

y<br />

2 2<br />

x y x y <br />

<br />

<br />

<br />

2 2 2 2<br />

( x y )1–2 x( x) ( x y )1–2 y ( y)<br />

<br />

X<br />

<br />

<br />

= <br />

<br />

dx dy<br />

2 2 2 2 2 2 x = – 2<br />

( x y ) ( x y ) <br />

<br />

2 2 2 2 2 2<br />

x y –2 x x y –2y<br />

<br />

= <br />

<br />

dx dy<br />

2 2 2 2 2 2 <br />

( x y ) ( x y ) <br />

2 2 2 2<br />

= y – x x – y <br />

<br />

dx dy<br />

2 2 2 2 2 2<br />

( x y ) ( x y ) <br />

= 0<br />

dx dy 0<br />

( x<br />

2 y<br />

2 )<br />

2<br />

5.37 AREA OF THE PLANE REGION BY GREEN’S THEOREM<br />

Proof. We know that<br />

Mdx Ndy = <br />

N M <br />

– dx dy<br />

A<br />

<br />

x y<br />

C<br />

<br />

...(1)<br />

<br />

N <br />

<br />

M <br />

On putting N = x 1<br />

and M = – y 1<br />

in (1), we get<br />

x <br />

y <br />

– ydx xdy = [1 –(–1)] dx dy<br />

A<br />

= 2 dx dy = 2 A<br />

C<br />

Area = 1 ( – )<br />

2<br />

xdy ydx<br />

C<br />

Example 84. Using Green’s theorem, find the area of the region in the first quadrant bounded<br />

by the curves<br />

y = x, y = 1 x<br />

, y = (U.P. I, Semester, Dec. 2008)<br />

x 4<br />

Solution. By Green’s Theorem Area A of the region bounded by a closed curve C is given by<br />

O<br />

Y<br />

y = 2<br />

x 2 + y 2 = 1<br />

y = – 2<br />

Ans.<br />

X<br />

x = 2<br />

Ans.


Vectors 435<br />

A = 1 Y<br />

( xdy – ydx)<br />

2<br />

C<br />

x<br />

Here, C consists of the curves C 1<br />

: y = , C<br />

4 2<br />

: y = 1 x<br />

and C 3<br />

: y = x So<br />

1 1 1 <br />

A <br />

<br />

<br />

<br />

( I1 I2 I3<br />

)<br />

C<br />

<br />

2C 2 C <br />

1 C <br />

2 C3<br />

<br />

2<br />

<br />

2<br />

<br />

<br />

C<br />

x 1<br />

3 C 1<br />

Along C 1<br />

: y = , dy dx , x :0to 2<br />

x<br />

y = —<br />

4 4<br />

4<br />

1 x <br />

I 1<br />

= ( xdy – ydx) x dx – dx 0<br />

C <br />

1 C<br />

<br />

<br />

1<br />

4 4 <br />

Along C 2<br />

: y = 1 , dy – 1<br />

O (0, 0)<br />

dx, x :2to1<br />

2<br />

x x<br />

1<br />

1 1<br />

I 2<br />

= ( xdy – ydx)<br />

x – <br />

dx – <br />

dx<br />

C<br />

2<br />

2<br />

2 <br />

x 2 <br />

<br />

=<br />

1<br />

[– 2log x] 2log 2<br />

2<br />

Along C 3<br />

: y = x, dy = dx ; x : 1 to 0 ;<br />

I 3<br />

=<br />

<br />

C3<br />

<br />

( xdy – ydx) ( xdx – xdx) 0<br />

A = 1 ( I 1<br />

1 I2 I3) (0 2log 2+0) log 2<br />

Ans.<br />

2 2<br />

EXERCISE 5.13<br />

2 2<br />

1. Evaluate<br />

[(3x 6 yz) dx(2y 3 xz) dy (1 4 xyz ) dz]<br />

from (0, 0, 0) to (1, 1, 1) along the path c<br />

c<br />

given by the straight line from (0, 0, 0) to (0, 0, 1) then to (0, 1, 1) and then to (1, 1, 1).<br />

2 2 3<br />

2. Verify Green’s Theorem in plane for ( x 2 xy) dx ( y x y) dy,<br />

where c is a square with the<br />

C<br />

1<br />

vertices P (0, 0), Q (1, 0), R (1, 1) and S (0, 1).<br />

Ans. <br />

2<br />

2 2<br />

3. Verify Green’s Theorem for ( x 2 xy) dx ( x y 3) dy around the boundary c of the region<br />

c<br />

y 2 = 8 x and x = 2.<br />

2 2 2 2<br />

4. Use Green’s Theorem in a plane to evaluate the integral<br />

[(2 x y ) dx ( x y ) dy]<br />

,<br />

c<br />

where c is the boundary in the xy-plane of the area enclosed by the x-axis and the semi-circle x 2 + y 2 =1<br />

in the upper half xy-plane. Ans. 4 3<br />

5. Apply Green’s Theoem to evaluate [( y sin x) dy cos xdy],<br />

c<br />

where c is the plane triangle enclosed<br />

<br />

2 x<br />

2<br />

8<br />

by the lines y = 0, x = and y .<br />

Ans. <br />

2<br />

<br />

4<br />

6. Either directly or by Green’s Therorem, evaluate the line integral <br />

e x (cos y dx sin ydy),<br />

c<br />

<br />

where c is the rectangle with vertices (0, 0), (, 0,), , and 0, .<br />

Ans. 2 (1 – e – )<br />

2 2<br />

(AMIETE II Sem June 2010)<br />

2<br />

7. Verify the Green’s Theorem to evaluate the line integral (2 y dx 3 x dy),<br />

where c is the boundary<br />

c<br />

of the closed region bounded by y = x and y = x 2 .<br />

(U.P., I Semester, Dec. 20005, AMIETE Summer 2004, Winter 2001) Ans. 27 4<br />

y = x<br />

B (1, 1)<br />

y = — 1 x<br />

A (2, —)<br />

1<br />

2<br />

X


436 Vectors<br />

8. Evaluate F . nds ˆ. ,<br />

s<br />

and s is the region of the plane 2x + 2y + z = 6<br />

<br />

where 2<br />

F xyiˆ x ˆ j ( x z)<br />

kˆ<br />

in the first octant. (A.M.I.E.T.E., Summer 2004, Winter 2001) Ans. 27 4<br />

9. Verify Green’s Theorem for<br />

2 2<br />

( xy y ) dx x dy<br />

<br />

C <br />

where C is the boundary by y = x and y = x 2 .<br />

(AMIETE, June 2010)<br />

5.38 STOKE’S THEOREM (Relation between Line Integral and Surface Integral)<br />

(Uttarakhand, I Sem. 2008, U.P., Ist Semester, Dec. 2006)<br />

Statement. Surface integral of the component of curl F along the normal to the surface S,<br />

taken over the surface S bounded by curve C is equal to the line integral of the <strong>vector</strong> point function<br />

F taken along the closed curve C.<br />

Mathematically<br />

<br />

<br />

F . d r = curl Fnds ˆ<br />

S<br />

where ˆn = cos î + cos ĵ + cos ˆk is a unit<br />

external normal to any surface ds,<br />

Proof. Let r = xiˆ<br />

yj ˆ<br />

zkˆ<br />

dr = idx ˆ ˆj dy kdz ˆ<br />

F = Fi ˆ<br />

ˆ<br />

1 F ˆ 2 j F3 k<br />

<br />

On putting the values of F,<br />

d r in the statement of the theorem<br />

<br />

c<br />

( Fi ˆ ˆ ˆ ˆ ˆ ˆ<br />

1 F2 j Fk 3 ) ( idx j dy kdz)<br />

= <br />

i j k <br />

S<br />

<br />

x y z<br />

( Fiˆ ˆ ˆ ˆ ˆ ˆ<br />

<br />

1 F 2 j F3 k). (cos i cos j cos k)<br />

ds<br />

( F1 dx <br />

F3 F2 1 3<br />

2 1<br />

F2 dy F3<br />

dz)<br />

=<br />

ˆ F F<br />

ˆ<br />

F F<br />

<br />

i j kˆ<br />

<br />

<br />

.<br />

S<br />

<br />

<br />

y z z x x y<br />

<br />

( iˆcos ˆj cos kˆ<br />

cos )<br />

ds<br />

3 2 1 3<br />

2 1<br />

= F<br />

F F F<br />

F F<br />

<br />

cos cos cos ds<br />

S y z<br />

<br />

<br />

z x<br />

<br />

x y<br />

<br />

...(1)<br />

Let us first prove<br />

F c<br />

1 dx<br />

1 1<br />

= F<br />

F<br />

<br />

cos cos ds<br />

S<br />

<br />

<br />

<br />

z<br />

y<br />

<br />

...(2)<br />

Let the equation of the surface S be z = g (x, y). The projection of the surface on x – y plane<br />

is region R.<br />

F 1 ( x , y , z ) dx<br />

c<br />

= F 1 [ x , y , g ( x , y )] dx<br />

c<br />

<br />

= F1 ( x, y, g)<br />

dx dy [By Green’s Theorem]<br />

R y<br />

F1 F1<br />

g <br />

= dx dy<br />

R<br />

<br />

y z y<br />

<br />

...(3)<br />

The direction consines of the normal to the surface z = g(x, y) are given by<br />

cos <br />

= cos cos <br />

<br />

g<br />

g<br />

1<br />

x<br />

y


Vectors 437<br />

And dx dy = projection of ds on the xy-plane = ds cos <br />

Putting the values of ds in R.H.S. of (2)<br />

F<br />

z<br />

F<br />

y<br />

<br />

1 1<br />

cos cos ds<br />

S<br />

<br />

=<br />

=<br />

R<br />

<br />

<br />

<br />

<br />

z<br />

cos y<br />

<br />

<br />

<br />

<br />

F1 cos F <br />

1 dx dy<br />

=<br />

R<br />

F1 F1<br />

dx dy<br />

cos cos <br />

z<br />

y<br />

cos <br />

F1 g<br />

F1<br />

<br />

<br />

dx dy<br />

R<br />

z y y<br />

<br />

F1 F1<br />

g <br />

= dx dy<br />

R<br />

<br />

y z y<br />

<br />

...(4)<br />

From (3) and (4), we get<br />

F c<br />

1 = F1 F1<br />

<br />

cos cos ds<br />

S<br />

<br />

z<br />

y<br />

<br />

...(5)<br />

Similarly, F c<br />

2 2 2<br />

= F<br />

F<br />

<br />

cos cos ds<br />

S<br />

<br />

x<br />

z<br />

<br />

<br />

<br />

...(6)<br />

and F c<br />

3 3 3<br />

= F<br />

F<br />

<br />

cos cos ds<br />

S<br />

<br />

y<br />

x<br />

<br />

...(7)<br />

On adding (5), (6) and (7), we get<br />

( F1 dx F2 dy F3<br />

dz)<br />

c<br />

=<br />

<br />

S<br />

F1 F1 F2 F2<br />

cos cos cos cos <br />

z y x z<br />

F3 F3<br />

<br />

cos cos ds Proved.<br />

y<br />

x<br />

<br />

5.39 ANOTHER METHOD OF PROVING STOKE’S THEOREM<br />

The circulation of <strong>vector</strong> F around a closed curve C is equal to the flux of<br />

the curve of the <strong>vector</strong> through the surface S bounded by the curve C.<br />

<br />

F<br />

d r<br />

<br />

c = curl Fnds curl FdS <br />

S S<br />

Proof : The projection of any curved surface over xy-plane can be treated as kernal of the<br />

surface integral over actual surface<br />

Now,<br />

S<br />

<br />

<br />

( F)<br />

kˆ<br />

dS<br />

= ( ) ( )<br />

<br />

= [( iˆ)( F ˆj) ( ˆj)( Fiˆ)]<br />

dx dy<br />

<br />

F i j dx dy<br />

[ kˆ iˆ<br />

ˆj]<br />

<br />

S<br />

<br />

<br />

F F dx dy<br />

<br />

= ( y) ( x)<br />

S<br />

S<br />

<br />

x<br />

y<br />

Fx<br />

dx Fy<br />

dy [By Green’s theorem]<br />

= [ ]<br />

=<br />

S<br />

[ ˆ ˆ ] ( ˆ ˆ<br />

iFx<br />

jFy<br />

idx j dy)<br />

S<br />

= F <br />

. dr<br />

<br />

c<br />

<br />

curl F ndS ˆ = . d r .<br />

S<br />

c F<br />

where, F = F iˆ F ˆj F kˆand<br />

dr dx iˆ dyj ˆ dz kˆ<br />

x y z<br />

<br />

<br />

Example 85. Evaluate by Strokes theorem ( yz dx zx dy xy dz)<br />

where C is the curve<br />

C<br />

x 2 + y 2 = 1, z = y 2 . (M.D.U., Dec 2009)<br />

Solution. Here we have<br />

yz dx zx dy xy dz<br />

= ( yziˆ zxj ˆ xykˆ<br />

).( idx ˆ ˆjdy kdz)


438 Vectors<br />

= Fdx .<br />

Curl F =<br />

iˆ<br />

ˆj kˆ<br />

<br />

x y z<br />

yz zx xy<br />

= curl F.<br />

ds<br />

= (x – x) î + (y – y) ĵ + (z – z) ˆk<br />

= 0 = 0 Ans.<br />

Example 86. Using Stoke’s theorem or otherwise, evaluate<br />

2 2<br />

[(2 x y) dx yz dy y z dz]<br />

<br />

c<br />

where c is the circle x 2 + y 2 = 1, corresponding to the surface of sphere of unit radius.<br />

(U.P., I Semester, Winter 2001)<br />

2 2<br />

Solution. [(2 x y) dx yz dy y z dz]<br />

c<br />

2 2 ˆ ˆ<br />

= [(2 x y) iˆ yz ˆj y z k] ( iˆdx ˆjdy k dz)<br />

By Stoke’s theorem<br />

Curl F =<br />

<br />

c<br />

<br />

<br />

<br />

F<br />

d r = Curl F n ds<br />

S<br />

...(1)<br />

iˆ<br />

ˆj kˆ<br />

F <br />

=<br />

Putting the value of curl F in (1), we get<br />

= ˆ<br />

ˆ<br />

= kˆ nˆ<br />

ds<br />

<br />

x y z<br />

2 2<br />

2x y yz y z<br />

= (– 2 yz 2 yz) iˆ<br />

–(0–0) ˆj (0 1)<br />

kˆ kˆ<br />

dx dy<br />

dx dy <br />

k n = dx dy<br />

nk ˆ ˆ = Area of the circle = <br />

ds <br />

( nk ˆ ˆ<br />

<br />

<br />

) <br />

<br />

<br />

Example 87. Evaluate F . d r, where F ( x,<br />

y i xj z kand C is the curve of<br />

C<br />

intersection of the plane y + z = 2 and the cylinder x 2 + y 2 = 1. (Gujarat, I sem. Jan. 2009)<br />

<br />

2 2<br />

Solution. F. dr curl F . nds ˆ curl (– y iˆ<br />

x ˆj z kˆ<br />

<br />

) nds ˆ<br />

...(1)<br />

Normal <strong>vector</strong> = <br />

C S S<br />

2 2 ˆ<br />

F (x, y, z) =<br />

Curl F =<br />

F <br />

2 2<br />

, z) – y ˆ ˆ ˆ<br />

y iˆ xj ˆ z k<br />

(By Stoke’s Theorem)<br />

iˆ<br />

ˆj kˆ<br />

<br />

x y z<br />

2 2<br />

– y x z<br />

= iˆ(0 –0)– ˆj (0 –0) kˆ(12 y) (1<br />

2 y)<br />

kˆ<br />

<br />

= ˆ<br />

<br />

ˆ<br />

ˆ <br />

i j k ( y z – 2) ˆj kˆ<br />

<br />

x y z<br />

ˆj<br />

kˆ<br />

Unit normal <strong>vector</strong> ˆn =<br />

2<br />

dx dy<br />

ds =<br />

ˆ . kˆ<br />

O<br />

3y + z = 2<br />

1<br />

x 2 + y 2 = 1


Vectors 439<br />

On putting the values of curl F,<br />

nˆ<br />

and ds in (1), we get<br />

=<br />

=<br />

=<br />

<br />

F<br />

<br />

. dr<br />

<br />

ˆ ˆ<br />

=<br />

ˆ j k dx dy<br />

(1 2 ) .<br />

C y k<br />

S<br />

2 ˆj<br />

kˆ<br />

<br />

. kˆ<br />

2 <br />

<br />

1<br />

2ydxdy<br />

(1 2 y)<br />

dxdy 2<br />

1<br />

<br />

2 1 = (1 2 r sin ) rd d r<br />

0 <br />

0<br />

Y<br />

2<br />

<br />

<br />

2<br />

1 2<br />

<br />

( r 2r sin ) d d r<br />

0 0<br />

1<br />

2 3<br />

2<br />

2<br />

r<br />

2r<br />

1 2 <br />

d sin sin d <br />

2 3<br />

<br />

2 3<br />

<br />

<br />

0 0<br />

<br />

0<br />

2<br />

<br />

2 2 2 <br />

= – cos – – 0<br />

2 3<br />

<br />

0 3 3<br />

<br />

= Ans.<br />

Example 88. Apply Stoke’s Theorem to find the value of<br />

<br />

c<br />

( ydx z dy xdz)<br />

where c is the curve of intersection of x 2 + y 2 + z 2 = a 2 and x + z = a. (Nagpur, Summer 2001)<br />

<br />

Solution. ( ydx z dy xdz)<br />

c<br />

ˆ<br />

ˆ<br />

= ( ˆ ˆ ) ( ˆ ˆ<br />

yi zj xk idx j dy kdz)<br />

c<br />

=<br />

= curl ( yiˆ zj ˆ xkˆ<br />

) nds ˆ<br />

<br />

S<br />

<br />

C<br />

( yiˆ<br />

zj ˆ xkˆ<br />

) dr<br />

(By Stoke’s Theorem)<br />

ˆ<br />

ˆ<br />

= <br />

ˆ<br />

<br />

ˆ<br />

<br />

i j k ( yiˆ zj ˆ xk)<br />

nˆ<br />

ds<br />

S<br />

<br />

x y z<br />

= ˆ ˆ ˆ<br />

( i j k)<br />

nˆ<br />

ds<br />

S<br />

...(1)<br />

where S is the circle formed by the intersection of x 2 + y 2 + z 2 = a 2 and x + z = a.<br />

<br />

ˆ<br />

<br />

ˆ<br />

ˆ <br />

<br />

i j k ( x z a)<br />

ˆn = | = x y z<br />

|<br />

| |<br />

iˆ<br />

kˆ<br />

ˆn = <br />

2 2<br />

Putting the vlaue of ˆn in (1), we have<br />

ˆ ˆ<br />

= ( ˆ ˆ ˆ<br />

i k <br />

i j k)<br />

ds<br />

S<br />

<br />

2 2<br />

<br />

1 1 <br />

= <br />

ds<br />

S<br />

<br />

2 2 <br />

2 2<br />

2 2<br />

a a<br />

= ds <br />

2<br />

S<br />

<br />

2 2 2<br />

Example 89. Directly or by Stoke’s Theorem, evaluate<br />

=<br />

iˆ<br />

kˆ<br />

1<br />

1<br />

<br />

2 2<br />

2 2 2 2 a a <br />

Use<br />

r R p a <br />

<br />

2 2 <br />

Ans.<br />

ˆ , ˆ ˆ ˆ<br />

curl vnds v iy jz kx,<br />

s is<br />

the surface of the paraboloid z = 1 – x 2 – y 2 , z 3 > 0 and ˆn is the unit <strong>vector</strong> normal to s.<br />

s<br />

<br />

<br />

O<br />

r ddr<br />

X


440 Vectors<br />

Solution.<br />

<br />

v =<br />

iˆ<br />

ˆj kˆ<br />

iˆ<br />

ˆj kˆ<br />

x y z<br />

y z x<br />

Obviously ˆn = k ˆ.<br />

Therefore<br />

<br />

( v)<br />

nˆ<br />

= (– iˆ<br />

– ˆj – kˆ). kˆ<br />

–1<br />

Hence<br />

( v)<br />

nds ˆ = ( 1) dx dy<br />

S<br />

=<br />

S<br />

<br />

S<br />

dx dy<br />

= – (1) 2 = – . (Area of circle = r 2 ) Ans.<br />

Example 90. Use Stoke’s Theorem to evaluate<br />

v ,<br />

c dr<br />

2<br />

where v y iˆ xyj ˆ xzkˆ<br />

, and c<br />

is the bounding curve of the hemisphere x 2 + y 2 + z 2 = 9, z > 0, oriented in the positive<br />

direction.<br />

Solution. By Stoke’s theorem<br />

<br />

( v)<br />

nds ˆ =<br />

S<br />

<br />

v <br />

c<br />

<br />

dr = (curl v) nˆ ds ( v)<br />

nds<br />

ˆ<br />

S S<br />

iˆ<br />

ˆj kˆ<br />

ˆ ˆ<br />

ˆ<br />

(0 0) i ( z 0) j ( y 2 y)<br />

k<br />

v =<br />

x y z zj ˆ ykˆ<br />

2<br />

y xy xz<br />

2 2 2<br />

<br />

i j k ( x y z 9)<br />

ˆn = | = x y z<br />

|<br />

| |<br />

=<br />

2 xiˆ 2 yj ˆ 2 zkˆ xiˆ yj ˆ zkˆ xiˆ yj ˆ zkˆ<br />

<br />

<br />

2 2 2 2 2 2<br />

4x 4y 4 z x y z 3<br />

<br />

ˆ ˆ ˆ<br />

( v)<br />

nˆ<br />

2<br />

= ( ˆ ˆ xi yj zk yz yz yz<br />

zj yk)<br />

<br />

3 3 3<br />

ˆ ˆ ˆ<br />

ˆn<br />

kds ˆ = dx dy <br />

xi yj zk . k ˆ<br />

z<br />

dx = dx dy ds = dx dy<br />

3<br />

3<br />

ds = 3 dx dy<br />

z<br />

<br />

2yz<br />

3<br />

<br />

dx dy = 2 ydxdy<br />

3 z <br />

<br />

=<br />

= 2r sin r d dr<br />

3<br />

3<br />

2 r <br />

0<br />

0<br />

<br />

2<br />

3<br />

<br />

2 sin d r dr<br />

<br />

0 0<br />

= 2( cos )<br />

= – 2 (– 1 + 1) 9 = 0 Ans.<br />

3 <br />

Example 91. Evaluate the surface integral curl F . nˆ<br />

dS by transforming it into a line<br />

S<br />

integral, S being that part of the surface of the paraboloid z = 1 – x 2 – y 2 for which<br />

<br />

z0and<br />

F yiˆ<br />

zj ˆ xkˆ<br />

. (K. University, Dec. 2008)<br />

<br />

2


Vectors 441<br />

Solution.<br />

<br />

<br />

F =<br />

iˆ<br />

ˆj kˆ<br />

iˆ<br />

ˆj kˆ<br />

x y z<br />

y z x<br />

Obviously ˆn = k ˆ.<br />

Therefore<br />

<br />

( F)<br />

nˆ<br />

= (– iˆ<br />

– ˆj – kˆ). kˆ<br />

–1<br />

Hence<br />

( F)<br />

nˆ<br />

ds<br />

= ( 1) dx dy<br />

S<br />

<br />

=<br />

S<br />

<br />

S<br />

dx dy<br />

= – (1) 2 = – . (Area of circle = r 2 ) Ans.<br />

Example 92. Evaluate Fdr<br />

<br />

2 2<br />

by Stoke’s Theorem, where F y iˆ x ˆ j ( x z)<br />

kˆ<br />

and<br />

C<br />

C is the boundary of triangle with vertices at (0, 0, 0), (1, 0, 0) and (1, 1, 0).<br />

(U.P., I Semester, Winter 2000)<br />

Solution. We have, curl F =<br />

=<br />

iˆ<br />

ˆj kˆ<br />

<br />

x y z<br />

2 2<br />

F <br />

0. iˆ ˆj 2( x y) kˆ<br />

.<br />

y x ( x z)<br />

We observe that z co-ordinate of each vertex of the triangle is zero.<br />

Therefore, the triangle lies in the xy-plane.<br />

ˆn = ˆk<br />

<br />

curl Fn ˆ [ ˆj 2( x yk ) ˆ]. kˆ<br />

2( x y).<br />

In the figure, only xy-plane is considered.<br />

The equation of the line OB is y = x<br />

By Stoke’s theorem, we have<br />

C<br />

<br />

Fdr <br />

=<br />

<br />

S<br />

<br />

<br />

(curl Fnˆ<br />

) ds<br />

1 x<br />

= 2( )<br />

2<br />

1<br />

x y dx dy<br />

y <br />

x0 = 2<br />

y 0 xy<br />

dx<br />

0<br />

2 0<br />

2<br />

1 2<br />

= 2 x <br />

2<br />

x<br />

dx<br />

0<br />

2 <br />

=<br />

1 x<br />

1<br />

2 dx = 2<br />

0 2<br />

x dx <br />

3<br />

x <br />

=<br />

0 <br />

3 <br />

x<br />

1<br />

0<br />

= 1 .<br />

3<br />

Example 93. Evaluate Fdr<br />

<br />

<br />

by Stoke’s Theorem, where<br />

2 2<br />

F ( x y ) iˆ 2 xy ˆj<br />

C and C<br />

is the boundary of the rectangle x = a, y = 0 and y = b. (U.P., I Semester, Winter 2002)<br />

Solution. Since the z co-ordinate of each vertex of the given rectangle is zero, hence the given<br />

rectangle must lie in the xy-plane.<br />

Here, the co-ordinates of A, B, C and D are (a, 0), (a, b), (– a, b) and (– a, 0) respectively.<br />

Curl F =<br />

iˆ<br />

ˆj kˆ<br />

<br />

= – 4 y k<br />

x y z<br />

2 2<br />

x y 2xy<br />

0<br />

Ans.


442 Vectors<br />

Here,<br />

nˆ kˆ<br />

, so by Stoke’s theorem, we’ve<br />

=<br />

S<br />

Fdr<br />

<br />

<br />

C<br />

= curl ˆ<br />

S<br />

( 4 ykˆ)()<br />

kˆ<br />

d xdy<br />

=<br />

a<br />

<br />

2<br />

y <br />

= 4 dx<br />

2<br />

=<br />

<br />

a<br />

b<br />

0<br />

F n ds<br />

a<br />

b<br />

4 <br />

xa y 0<br />

a<br />

2 2<br />

<br />

2b dx 4ab<br />

a<br />

Example 94. Apply Stoke’s Theorem to calculate<br />

c<br />

<br />

ydxdy<br />

4ydx 2zdy 6ydz<br />

where c is the curve of intersection of x 2 + y 2 + z 2 = 6 z and z = x + 3.<br />

Solution.<br />

<br />

<br />

<br />

F dr = 4 ydx 2z dy 6 ydz<br />

c c<br />

=<br />

F =<br />

F <br />

=<br />

<br />

c<br />

(4 yiˆ 2zj ˆ 6 ykˆ) ( idx ˆ ˆjdy kdz ˆ )<br />

4 yiˆ<br />

2 zj ˆ 6<br />

ykˆ<br />

iˆ<br />

ˆj kˆ<br />

(6 2) iˆ(0 0) ˆj (0 4) kˆ<br />

x y z 4iˆ<br />

4kˆ<br />

4y 2z 6 y<br />

Ans.<br />

S is the surface of the circle x 2 + y 2 + z 2 = 6z, z = x + 3, ˆn is normal to the plane x – z + 3 = 0<br />

<br />

ˆ ˆ ˆ <br />

i j k ( x z 3)<br />

<br />

ˆn = | = x y z<br />

|<br />

| |<br />

( F)<br />

nˆ<br />

=<br />

ˆ ˆ<br />

(4 ˆ 4 ˆ i k<br />

i k)<br />

= 4 4 = 4 2<br />

2 2<br />

=<br />

iˆkˆ<br />

iˆkˆ<br />

<br />

1<br />

1 2<br />

Fdr<br />

<br />

= (curl ) ˆ<br />

c F nds = 4 2 ( dx dz)<br />

S<br />

S<br />

= 4 2 (area of circle)<br />

Centre of the sphere x 2 + y 2 + (z – 3) 2 = 9, (0, 0, 3) lies on the plane z = x + 3. It means that<br />

the given circle is a great circle of sphere, where radius of the circle is equal to the radius of the<br />

sphere.<br />

Radius of circle = 3, Area = (3) 2 = 9 <br />

( F)<br />

nˆ<br />

ds = 4 2(9 ) 36 2 Ans.<br />

S<br />

Example 95. Verify Stoke’s Theorem for the function<br />

F ziˆ x ˆ j ykˆ<br />

, where C is the unit<br />

2 2<br />

circle in xy-plane bounding the hemisphere z = ( 1x y ). (U.P., I Semester Comp. 2002)<br />

Solution. Here F = ziˆ xj ˆ ykˆ<br />

.<br />

...(1)<br />

Also, r = xiˆ<br />

yj ˆ zkˆ<br />

dr = dxiˆ dyj ˆ dz kˆ<br />

.<br />

<br />

F<br />

dr = z dx + x dy + y dz.<br />

<br />

F<br />

dr = ( zdx x dy ydz).<br />

C C<br />

...(2)


Vectors 443<br />

On the circle C, x 2 + y 2 = 1, z = 0 on the xy-plane. Hence on C, we<br />

have z = 0 so that dz = 0. Hence (2) reduces to<br />

F<br />

dr = xdy .<br />

C C<br />

...(3)<br />

Now the parametric equations of C, i.e., x 2 + y 2 = 1 are<br />

x = cos , y = sin . ...(4)<br />

Using (4), (3) reduces to F <br />

. dr<br />

<br />

C<br />

=<br />

2<br />

cos cos d<br />

=<br />

0 2<br />

<br />

2<br />

0<br />

1 cos 2 <br />

2<br />

1 sin 2 <br />

=<br />

2<br />

<br />

2<br />

= ...(5)<br />

0<br />

Let P(x, y, z) be any point on the surface of the hemisphere x 2 + y 2 + z 2 = 1, O origin is the<br />

centre of the sphere.<br />

Radius = OP =<br />

xiˆ<br />

yj ˆ zkˆ<br />

Normal = xiˆ<br />

yj ˆ zkˆ<br />

ˆn<br />

xiˆ<br />

yj ˆ zkˆ<br />

=<br />

xiˆ yj ˆ zkˆ<br />

2 2 2<br />

x y z<br />

(Radius is to tangent i.e. Radius is normal)<br />

x = sin cos , y = sin sin , z = cos ...(6)<br />

ˆn = sin cos î + sin sin ĵ + cos ˆk<br />

iˆ<br />

ˆj kˆ<br />

Also, Curl F = / x / y /<br />

z iˆ<br />

ˆj kˆ<br />

...(7)<br />

z x y<br />

<br />

=<br />

=<br />

d <br />

<br />

Curl Fn ˆ = ( iˆ ˆj kˆ).(sin cos iˆsin sin ˆj sin kˆ)<br />

= sin cos + sin sin + cos <br />

/2 2<br />

Curl Fn ˆ dS = ( ˆ ˆ ˆ<br />

i j k)<br />

0<br />

<br />

0<br />

s<br />

=<br />

<br />

. (sin cos î + sin sin ĵ + cos ˆk ) sin d d<br />

/2 2<br />

<br />

sin d (sin cos sin sin cos )<br />

d<br />

0 0<br />

[ dS = Elementary area on hemisphere = sin d d]<br />

/2<br />

2<br />

sin d<br />

[sin sin sin ( cos ) cos ]<br />

0<br />

0 =<br />

/2<br />

(0 0 2 sin cos ) d =<br />

0<br />

= – (/2) [– 1 – 1] = .<br />

From (5) and (8),<br />

C<br />

<br />

<br />

Fdr =<br />

/2<br />

sin 2 d<br />

=<br />

0<br />

<br />

/2<br />

0<br />

<br />

<br />

<br />

sin d<br />

<br />

cos 2<br />

2<br />

<br />

<br />

<br />

/2<br />

curl F<br />

ndS ˆ , which verifies Stokes’s theorem.<br />

S<br />

Example 96. Verify Stoke’s theorem for the <strong>vector</strong> field F (2 x – y) iˆ<br />

– yz ˆj – y zk over<br />

0<br />

2 2 ˆ<br />

the upper half of the surface x 2 + y 2 + z 2 = 1 bounded by its projection on xy- plane.<br />

(Nagpur University, Summer 2001)<br />

Solution. Let S be the upper half surface of the sphere x 2 + y 2 + z 2 = 1. The boundary C or S<br />

is a circle in the xy plane of radius unity and centre O. The equation of C are x 2 + y 2 = 1,<br />

z = 0 whose parametric form is<br />

x = cos t, y = sin t, z = 0, 0 < t < 2<br />

<br />

<br />

F . dr =<br />

C<br />

<br />

C<br />

2 2<br />

[(2 x – y) iˆ – yz ˆj – y zkˆ].[ iˆ dx ˆjdy kdz ˆ ]


444 Vectors<br />

2 2<br />

= [(2 x – y) dx – yz dy – y z dz]<br />

C<br />

= (2 x – y) dx,<br />

C<br />

since on C, z = 0 and 2z = 0<br />

2<br />

dx 2<br />

= (2cos t –sin t) dt (2cos t –sin t)(–sin t)<br />

dt<br />

0 dt<br />

0<br />

2 2<br />

2 1–cos 2t<br />

<br />

= (– sin 2t sin t) dt – sin 2t dt<br />

0 <br />

0<br />

<br />

<br />

<br />

2 <br />

2<br />

cos 2t t sin 2t<br />

1 1<br />

= – –<br />

2 2 4<br />

<br />

<br />

0 2 2<br />

iˆ<br />

ˆj kˆ<br />

...(1)<br />

<br />

Curl F =<br />

<br />

x y z = (– 2yz 2 yz) iˆ<br />

(0–0) ˆj (01)<br />

kˆ kˆ<br />

2 x – y<br />

2<br />

– yz<br />

2<br />

– y z<br />

<br />

Z<br />

Curl F . nˆ<br />

= kˆ. nˆ<br />

nk ˆ.<br />

ˆ<br />

<br />

Curl F.<br />

nds ˆ = ˆ. ˆ ˆ. ˆ dx dy<br />

. .<br />

S nkds nk<br />

S R ˆ ˆ<br />

Where R is the projection of S on xy-plane.<br />

O<br />

Y<br />

=<br />

2<br />

1 1– x<br />

dx dy =<br />

2<br />

–1 – 1– x<br />

<br />

1 2 1 2<br />

2 1– x dx 4 1– x dx<br />

–1 0<br />

1<br />

2 –1<br />

x<br />

1 1<br />

<br />

= 4 1– x sin x 4 .<br />

2 2<br />

<br />

0 2 2<br />

<br />

<br />

From (1) and (2), we have<br />

<br />

<br />

F . dr = Curl F . nds ˆ which is the Stoke's theorem.<br />

C <br />

<br />

Example 97. Verify Stoke’s Theorem for 2 ˆ ˆ<br />

2<br />

F ( x y 4) i 3 xyj (2 xz z ) kˆ<br />

over the surface of hemisphere x 2 + y 2 + z 2 = 16 above the xy-plane.<br />

<br />

Solution. Fdr , where c is the boundary of the circle x 2 + y 2 + z 2 = 16<br />

c<br />

(bounding the hemispherical surface)<br />

2 2 ˆ<br />

= [( x y 4) iˆ 3 xyj ˆ (2 xz z ) k]( idx ˆ ˆjdy)<br />

Putting<br />

<br />

<br />

c<br />

2<br />

= [( x y 4) dx 3 xy dy)]<br />

c<br />

x = 4 cos , y = 4 sin , dx = – 4 sin d , dy = 4 cos d <br />

=<br />

<br />

2<br />

2 2<br />

0<br />

<br />

[(16 cos 4sin 4) ( 4sin d ) (192 sin cos d )]<br />

2 2 2 2<br />

= 16 [ 4cos sin sin sin 12 sin cos ] d <br />

<br />

<br />

0<br />

2<br />

2 2<br />

= 16 (8 sin cos sin sin ) d <br />

=<br />

=<br />

0<br />

<br />

2<br />

16 sin d<br />

<br />

0<br />

2<br />

<br />

2 2<br />

=<br />

0<br />

16 4 sin d<br />

<br />

To evaluate surface integral<br />

F <br />

=<br />

1<br />

<br />

64 = – 16 .<br />

22 <br />

iˆ<br />

ˆj kˆ<br />

<br />

x y z<br />

X<br />

<br />

<br />

<br />

<br />

<br />

2 2<br />

x y 4 3xy 2 xz z<br />

<br />

<br />

2<br />

0<br />

2<br />

0<br />

C<br />

...(2)<br />

Ans.<br />

n<br />

sin cos d0<br />

<br />

<br />

<br />

n<br />

cos sin d<br />

0


Vectors 445<br />

= (0 – 0) iˆ – (2 z – 0) ˆj (3 y – 1) kˆ – 2 zj ˆ (3 y – 1) kˆ<br />

<br />

ˆ ˆ ˆ 2 2 2<br />

<br />

i j k ( x y z 16)<br />

ˆn = | = x y z<br />

|<br />

| |<br />

2xiˆ<br />

2yj ˆ 2zkˆ<br />

xiˆ<br />

yj ˆ zkˆ<br />

xiˆ<br />

yj ˆ zkˆ<br />

=<br />

=<br />

=<br />

2 2 2 2 2 2<br />

4x 4y 4z<br />

x y z 4<br />

<br />

ˆ ˆ ˆ<br />

( F)<br />

nˆ<br />

= [– 2 ˆ (3 – 1) ˆ xi yj zk<br />

zj y k]<br />

<br />

2 yz (3y 1) z<br />

=<br />

4<br />

4<br />

ˆk n<br />

xiˆ<br />

yj ˆ zkˆ<br />

z<br />

ds = dx dy <br />

. k ds = dx dy ds = dx dy<br />

4<br />

4<br />

<br />

ds = 4 dx dy<br />

z<br />

2 (3 1) 4<br />

( F)<br />

nˆ<br />

ds yz y z dx dy<br />

<br />

<br />

4 z <br />

= [ 2 y (3 y 1)] dx dy = ( y 1) dxdy<br />

On putting x = r cos , y = r sin , dx dy = r d dr, we get<br />

=<br />

=<br />

= ( r sin 1) r d dr<br />

2<br />

3 2<br />

4<br />

<br />

<br />

2<br />

d ( r sin r)<br />

dr<br />

= r<br />

r <br />

d <br />

<br />

sin <br />

3 2 = 64 <br />

d <br />

sin 8 <br />

0 <br />

<br />

3<br />

0 0<br />

<br />

2<br />

64 64 64<br />

= cos 8 <br />

= 16<br />

= – 16 <br />

3 3 3<br />

0<br />

The line integral is equal to the surface integral, hence Stoke’s Theorem is verified. Proved.<br />

<br />

2 2<br />

Example 98. Verify Stoke’s theorem for a <strong>vector</strong> field defined by F ( x – y ) iˆ 2xy ˆj<br />

in<br />

the rectangular in xy-plane bounded by lines x = 0, x = a, y = 0, y = b.<br />

(Nagpur University, Summer 2000)<br />

Solution. Here we have to verify Stoke’s theorem F <br />

. dr<br />

<br />

= ( ). ˆ<br />

C F<br />

nds<br />

S<br />

Where ‘C’ be the boundary of rectangle (ABCD) and S be the surface enclosed by curve C.<br />

2<br />

F 2 2<br />

= ( x – y ) iˆ (2 xy)<br />

ˆj<br />

<br />

F . dr =<br />

2 2<br />

[( x – y ) iˆ 2 xyj ˆ].[ idx ˆ ˆj dy]<br />

<br />

F . dr = (x<br />

2<br />

+ y 2 ) dx + 2xy dy ...(1)<br />

Now, F <br />

. dr<br />

<br />

<br />

= F . dr F . dr F . dr F . dr<br />

C<br />

...(2)<br />

OA AB BC CO<br />

<br />

Along OA, put y = 0 so that k dy = 0 in (1) and F . dr = x 2 dx,<br />

Where x is from 0 to a.<br />

3<br />

a<br />

a<br />

2<br />

F <br />

. dr<br />

<br />

3<br />

x dx<br />

x a<br />

OA = <br />

0 3 3<br />

<br />

Along AB, put x = a so that dx = 0 in (1), we get F .<br />

Where y is from 0 to b.<br />

<br />

b<br />

0<br />

0<br />

<br />

d r<br />

= 2ay dy<br />

...(3)<br />

2 2<br />

F . dr 2aydy<br />

= [ ay ] b<br />

0<br />

ab<br />

...(4)<br />

AB


446 Vectors<br />

Along BC, put y = b and dy = 0 in (1) we get F . dr = (x 2 – b 2 ) dx,<br />

where x is from a to 0.<br />

F<br />

<br />

BC<br />

. dr =<br />

<br />

0<br />

3<br />

0<br />

3<br />

2 2 x 2<br />

– a 2<br />

Along CO, put x = 0 and dx = 0 in (1), we get F . dr 0<br />

( x – b ) dx – bx<br />

ba<br />

a<br />

3 3<br />

...(5)<br />

<br />

<br />

F <br />

. dr<br />

<br />

= 0 ...(6)<br />

CO<br />

Putting the values of integrals (3), (4), (5) and (6) in (2),<br />

we get<br />

<br />

<br />

F . dr =<br />

C<br />

3 3<br />

2 a 2 2<br />

a<br />

ab – ab 0 2ab<br />

...(7)<br />

3 3<br />

Now we have to evaluate R.H.S. of Stoke’s Theorem i.e.<br />

We have,<br />

F <br />

=<br />

iˆ<br />

ˆj kˆ<br />

<br />

x y z<br />

2 2<br />

x – y 2xy<br />

0<br />

<br />

S<br />

a<br />

<br />

( F).<br />

nds ˆ<br />

(2y 2 y) kˆ<br />

4ykˆ<br />

Also the unit <strong>vector</strong> normal to the surface S in outward direction is ˆn<br />

k<br />

(z-axis is normal to surface S)<br />

Also in xy-plane ds = dx dy<br />

<br />

( F). nˆ<br />

. ds = 4 ˆ. ˆ 4 .<br />

S yk kdxdy ydx dy<br />

R R<br />

Where R be the region of the surface S.<br />

Consider a strip parallel to y-axis. This strip starts on line y = 0 (i.e. x-axis) and end on the line<br />

y = b, We move this strip from x = 0 (y-axis) to x = a to cover complete region R.<br />

<br />

<br />

( F). nˆ<br />

. ds =<br />

S<br />

<br />

From (7) and (8), we get<br />

<br />

<br />

=<br />

<br />

<br />

ydy<br />

<br />

dx y dx<br />

<br />

a b 2<br />

4 <br />

a [2 ]<br />

b<br />

0 0 0<br />

0<br />

<br />

a 2 2 a 2<br />

b dx b x<br />

0<br />

ab<br />

...(8)<br />

0 2 2 [ ] 2<br />

F . dr = ( ). ˆ<br />

C F nds and hence the Stoke’s theorem is verified.<br />

S<br />

Example 99. Verify Stoke’s Theorem for the function<br />

<br />

F = xi<br />

2ˆ – xyj ˆ<br />

integrated round the square in the plane z = 0 and bounded by the lines<br />

x = 0, y = 0, x = a, y = a.<br />

Solution. We have, F = xi<br />

2ˆ – xyj ˆ<br />

<br />

F =<br />

iˆ<br />

ˆj kˆ<br />

<br />

x y z<br />

2<br />

x = 0<br />

Y<br />

y = b<br />

C<br />

B<br />

(a, b)<br />

x = a<br />

x xy 0<br />

= (0 –0) iˆ<br />

–(0–0) ˆj (– y –0) kˆ – ykˆ<br />

( ˆn to xy plane i.e. ˆk )<br />

O<br />

y = 0<br />

A<br />

X


Vectors 447<br />

( F)<br />

nˆ<br />

ds = ( yk)<br />

kdxdy<br />

S<br />

S<br />

<br />

=<br />

<br />

To obtain line integral<br />

a<br />

a<br />

dx<br />

ydy =<br />

0 0<br />

a<br />

a<br />

2<br />

y<br />

dx <br />

<br />

<br />

<br />

2 <br />

=<br />

0 0<br />

2<br />

a a<br />

( x) 0 =<br />

2<br />

<br />

F dr 2<br />

= ˆ ˆ ˆ ˆ<br />

2<br />

( x i xyj) ( idx j dy)<br />

= ( x dx xydy)<br />

C<br />

C<br />

C<br />

where c is the path OABCO as shown in the figure.<br />

Also,<br />

<br />

F dr =<br />

C<br />

<br />

Fdr =<br />

OABCO<br />

Along OA, y = 0, dy = 0<br />

F<br />

dr<br />

<br />

=<br />

OA<br />

<br />

2<br />

( xdx<br />

xydy )<br />

OA<br />

a <br />

3<br />

2 x <br />

= xdx = =<br />

0<br />

3 0<br />

Along AB, x = a, dx = 0<br />

Fdr<br />

<br />

2<br />

= ( x dx xyd y)<br />

AB AB<br />

=<br />

a<br />

aydy =<br />

Along BC, y = a, dy = 0<br />

<br />

<br />

0<br />

<br />

Fdr Fdr Fdr F<br />

dr<br />

OA AB BC CO<br />

3<br />

a<br />

...(1)<br />

2<br />

...(2)<br />

a<br />

3<br />

a<br />

3<br />

2<br />

y<br />

a <br />

<br />

<br />

2 <br />

F dr<br />

2<br />

= ( xdx xydy)<br />

=<br />

BC<br />

BC<br />

Along CO, x = 0, dx = 0<br />

<br />

a<br />

0<br />

=<br />

3<br />

a<br />

<br />

2<br />

0 2<br />

xdx<br />

a<br />

=<br />

Fdr <br />

2<br />

= ( )<br />

CO xdx xydy = 0<br />

CO<br />

Putting the values of these integrals in (2), we have<br />

Fdr<br />

<br />

=<br />

C<br />

From (1) and (3),<br />

3 3 3<br />

a a a<br />

0 =<br />

3 2 3<br />

( F)<br />

nˆ<br />

ds =<br />

Hence, Stoke’s Theorem is verified.<br />

S<br />

<br />

<br />

<br />

C<br />

<br />

3<br />

<br />

3<br />

x <br />

<br />

<br />

0<br />

3 a<br />

=<br />

3<br />

a<br />

<br />

3<br />

a<br />

...(3)<br />

2<br />

<br />

F dr<br />

Ans.<br />

Example 100. Verify Stoke’s Theorem for F<br />

<br />

= (x + y) î + (2x – z) ĵ + (y + z) ˆk for the<br />

surface of a triangular lamina with vertices (2, 0, 0), (0, 3, 0) and (0, 0, 6).<br />

(Nagpur University 2004, K. U. Dec. 2009, 2008, A.M.I.E.T.E., Summer 2000)<br />

Solution. Here the path of integration c consists of the straight lines AB, BC, CA where the<br />

co-ordinates of A, B, C and (2, 0, 0), (0, 3, 0) and (0, 0, 6) respectively. Let S be the plane<br />

surface of triangle ABC bounded by C. Let ˆn be unit normal <strong>vector</strong> to surface S. Then by<br />

Stoke’s Theorem, we must have<br />

Fdr<br />

<br />

=<br />

c<br />

curl Fnds ˆ<br />

...(1)<br />

s<br />

<br />

<br />

line Eq. of Lower Upper<br />

line limit limit<br />

OA y = 0 dy = 0 x = 0 x = a<br />

AB x = a dx = 0 y = 0 y = a<br />

BC y = a dy= 0 x = a x = 0<br />

CO x = 0 dx = 0 y = a y = 0


448 Vectors<br />

c<br />

L.H.S. of (1)= Fdr<br />

<br />

= Fdr F dr F dr<br />

ABC AB <br />

BC CA<br />

x y<br />

Along line AB, z = 0, equation of AB is = 1 2 3<br />

y = 3 3<br />

(2 x),<br />

dy =<br />

2<br />

2 dx<br />

At A, x = 2, At B, x = 0, r = xiˆ<br />

yj ˆ<br />

<br />

<br />

F dr =<br />

AB<br />

<br />

<br />

AB<br />

[( x y) iˆ 2 xj ˆ ykˆ<br />

]( idx ˆ ˆjdy)<br />

= ( x y) dx 2xdy<br />

=<br />

<br />

AB<br />

AB<br />

3x<br />

3 <br />

x 3 dx 2x<br />

dx<br />

2 2 <br />

2<br />

0<br />

7x<br />

7x<br />

<br />

= 3 dx<br />

3x<br />

2<br />

<br />

2 <br />

4 <br />

<br />

= (7 – 6) = + 1<br />

y z<br />

Along line BC, x = 0, Equation of BC is = 1 or z = 6 – 2y, dz = – 2dy<br />

3 6<br />

At B, y = 3, At C, y = 0, r =<br />

<br />

<br />

yj ˆ zkˆ<br />

F dr = [ ( ) ] ( )<br />

BC<br />

yi zj y z k jdy kdz = ( )<br />

BC zdy y z dz<br />

BC<br />

0<br />

= ( 6 2 y) dy ( y 6 2 y)( 2 dy)<br />

3<br />

0 2 0<br />

= (4y 18) dy (2y 18 y)<br />

3<br />

3 = 36<br />

x z<br />

Along line CA, y = 0, Eq. of CA, = 1 or z = 6 – 3x, dz = – 3dx<br />

2 6<br />

At C, x = 0, at A, x = 2, r = xiˆ<br />

zkˆ<br />

<br />

<br />

F dr = [ xiˆ (2 x z) ˆj zkˆ][ dxiˆ<br />

dzkˆ]<br />

CA = ( xdx zdz)<br />

CA<br />

=<br />

line Eq. of Lower Upper<br />

line limit limit<br />

AB<br />

x y 3 At A At B<br />

1<br />

dy – dx<br />

2 3 2 x 2 x 0<br />

z = 0<br />

BC<br />

y z<br />

At B At C<br />

1<br />

dz = – 2dy<br />

3 6<br />

y 3 y 0<br />

x = 0<br />

CA<br />

x z<br />

At C At A<br />

1<br />

dz = – 3dx<br />

2 6<br />

x 0 x 2<br />

y = 0<br />

CA<br />

2<br />

xdx (6 3 x )( 3 dx ) =<br />

0<br />

2<br />

0<br />

0<br />

2<br />

<br />

(10 x 18) dx 2 2<br />

= [5x<br />

18 x]<br />

0 = – 16


Vectors 449<br />

c<br />

L.H.S. of (1) =<br />

Fdr<br />

<br />

<br />

ABC<br />

=<br />

<br />

Fdr F dr F dr<br />

= 1 + 36 – 16 = 21 ...(2)<br />

AB BC CA<br />

<br />

Curl F = F =<br />

ˆ<br />

<br />

ˆ<br />

ˆ <br />

i j k [( x y) iˆ (2 x z) ˆj ( y z) k]<br />

x y z<br />

iˆ<br />

ˆj kˆ<br />

=<br />

<br />

x y z<br />

x y 2x z y z<br />

x y z<br />

Equation of the plane of ABC is = 1<br />

2 3 6<br />

Normal to the plane ABC is<br />

(1 1) iˆ(0 0) ˆj (2 1) kˆ<br />

2iˆ<br />

kˆ<br />

<br />

ˆ<br />

=<br />

ˆ<br />

<br />

ˆ<br />

x y z <br />

i j k 1<br />

=<br />

x y z2 3 6 <br />

iˆ<br />

ˆj kˆ<br />

<br />

Unit Normal Vector =<br />

2 3 6<br />

1 1 1<br />

<br />

4 9 36<br />

1<br />

ˆn = (3iˆ<br />

2 ˆj kˆ<br />

)<br />

14<br />

i ˆ ˆ ˆ<br />

<br />

j <br />

k<br />

2 3 6<br />

<br />

R.H.S. of (1) = curl Fnds <br />

= ˆ ˆ 1<br />

ˆ ˆ ˆ dx dy<br />

(2 ) (3 2 )<br />

s i k i j k<br />

s<br />

4 1<br />

(3iˆ<br />

2 ˆj kˆ).<br />

kˆ<br />

14<br />

(6 1) dx dy<br />

= = 7<br />

s<br />

14 1 dx dy = 7 Area of OAB<br />

14<br />

1<br />

<br />

= 7<br />

23<br />

= 21 ...(3)<br />

2<br />

<br />

with the help of (2) and (3) we find (1) is true and so Stoke’s Theorem is verified.<br />

Example 101. Verify Stoke’s Theorem for<br />

<br />

F<br />

= (y – z + 2) î + (yz + 4) ĵ – (xz) ˆk<br />

over the surface of a cube x = 0, y = 0, z = 0, x = 2, y = 2, z = 2 above the XOY plane<br />

(open the bottom).<br />

Solution. Consider the surface of the cube as shown in the figure. Bounding path is OABCO<br />

shown by arrows.<br />

<br />

Fdr = [( y z 2) iˆ ( yz 4) ˆj ( xz) kˆ] ( idx ˆ ˆjdy kdz ˆ )<br />

<br />

<br />

= ( y z 2) dx ( yz 4) dy xzdz<br />

c<br />

<br />

<br />

Fdr = Fdr Fdr Fdr Fdr<br />

c<br />

OA AB BC CO<br />

(1) Along OA, y = 0, dy = 0, z = 0, dz = 0<br />

...(1)


450 Vectors<br />

Line Equ. Lower Upper Fdr .<br />

1 OA<br />

2 AB<br />

3 BC<br />

4 CO<br />

of line limit limit<br />

y 0 dy 0<br />

z 0 dz 0<br />

x = 0 x = 2 2 dx<br />

x 2 dx 0<br />

z 0 dz 0<br />

y = 0 y = 2 4 dy<br />

y 2 dy 0<br />

z 0 dz 0<br />

x = 2 x = 0 4 dx<br />

x 0 dx 0<br />

z 0 dz 0<br />

y = 2 y = 0 4 dy<br />

Fdr =<br />

OA<br />

(2) Along AB, x = 2, dx = 0, z = 0, dz = 0<br />

AB<br />

<br />

<br />

<br />

<br />

Fdr =<br />

(3) Along BC, y = 2, dy = 0, z = 0, dz = 0<br />

F dr =<br />

BC<br />

(4) Along CO, x = 0, dx = 0, z = 0, dz = 0<br />

CO<br />

<br />

<br />

<br />

<br />

2<br />

2<br />

2 dx [2 x]<br />

0 = 4<br />

0<br />

2<br />

2<br />

4dy<br />

4( y)<br />

0 = 8<br />

0<br />

2<br />

0<br />

(2 0 2) dx (4 x)<br />

2 = – 8<br />

0<br />

Fdr = ( y 0 2) 0 (0 4) dy 0<br />

0<br />

= 4 dy 4( y) 2 = – 8<br />

On putting the values of these integrals in (1), we get<br />

Fdr = 4 + 8 – 8 – 8 = – 4<br />

c<br />

<br />

<br />

To obtain surface integral<br />

iˆ<br />

ˆj kˆ<br />

F <br />

=<br />

<br />

x y z<br />

y z 2 yz 4 xz<br />

= (0 – y) î – (– z + 1) ĵ + (0 – 1) ˆk = – y î + (z – 1) ĵ – ˆk<br />

Here we have to integrate over the five surfaces, ABDE, OCGF, BCGD, OAEF, DEFG.<br />

Over the surface ABDE (x = 2), ˆn = i, ds = dy dz<br />

<br />

( F)<br />

nds ˆ = [ yi ( z 1) j k]<br />

idx dz ydydz<br />

<br />

2<br />

= <br />

<br />

R<br />

( , )<br />

3 ( , , ) z f x y<br />

z f ( x, y)<br />

F x y z dx dy<br />

1


Vectors 451<br />

Surface Outward normal<br />

ds<br />

1 ABDE i dy dz x = 2<br />

2 OCGF – i dy dz x = 0<br />

3 BCGD j dx dz y = 2<br />

4 OAEF – j dx dz y = 0<br />

5 DEFG k dx dy z = 2<br />

=<br />

2 2 2<br />

y<br />

2<br />

ydydz [ z] 4<br />

0<br />

2 <br />

0 0 0<br />

2<br />

Over the surface OCGF (x = 0), ˆn = – i, ds = dy dz<br />

<br />

( F)<br />

nds ˆ = [ ˆ ( 1) ˆ ˆ] ( ˆ<br />

yi z j k i)<br />

dy dz<br />

=<br />

2 2 2<br />

y<br />

<br />

ydydz y dy dz 2<br />

4<br />

2 <br />

<br />

0 0 0<br />

2<br />

(3) Over the surface BCGD, (y = 2), ˆn = j, ds = dx dz<br />

<br />

( F)<br />

nds ˆ = [ yiˆ ( z 1) ˆj kˆ<br />

] ˆ<br />

j dx dz<br />

=<br />

= ( z 1)<br />

dxdz<br />

2 2<br />

dx ( z 1) dz =<br />

0 0<br />

( x)<br />

2<br />

0<br />

<br />

2<br />

z<br />

<br />

2<br />

2<br />

<br />

z<br />

<br />

<br />

0<br />

= 0<br />

(4) Over the surface OAEF, (y = 0), ˆn = – ĵ , ds = dx dz<br />

<br />

( F ) nˆ<br />

ds = ˆ<br />

<br />

[ yiˆ ( z 1) ˆj k]( ˆj)<br />

dx dz<br />

= ( z 1)<br />

dxdz = –<br />

2 2<br />

dx ( z 1) dz =<br />

0 0<br />

(5) Over the surface DEFG, (z = 2), ˆn = k, ds = dx dy<br />

( x)<br />

2<br />

0<br />

<br />

2<br />

z<br />

<br />

2<br />

2<br />

<br />

z<br />

<br />

<br />

0<br />

= 0<br />

<br />

( F)<br />

nds ˆ = [ yi ˆ ( z 1) ˆ j k ˆ]<br />

k ˆ<br />

dx dy = – dx dy<br />

= –<br />

2 2<br />

dx<br />

dy 2 2<br />

= [ x] [ y]<br />

= – 4<br />

0 0<br />

0 0<br />

Total surface integral = – 4 + 4 + 0 + 0 – 4 = – 4<br />

Thus<br />

curl F nds ˆ =<br />

S<br />

<br />

<br />

F dr = – 4<br />

c<br />

<br />

which verifies Stoke’s Theorem.<br />

<br />

Ans.


452 Vectors<br />

1. Use the Stoke’s Theorem to evaluate<br />

EXERCISE 5.14<br />

<br />

2<br />

ydx<br />

xydy xzdz,<br />

C<br />

where C is the bounding curve of the hemisphere x 2 + y 2 + z 2 = 1, z 0, oriented in the positive<br />

direction. Ans. 0<br />

2. Evaluate<br />

(curl F) ndA ˆ , using the Stoke’s Theorem, where F yiˆ<br />

zj ˆ xkˆ<br />

and s is the paraboloid<br />

s<br />

z = f (x, y) = 1 – x 2 – y 2 , z 0.<br />

Ans. <br />

<br />

2 2 2<br />

3. Evaluate the integral for ydx z dy xdz, where C is the triangular closed path joining the points<br />

C<br />

(0, 0, 0), (0, a, 0) and (0, 0, a) by transforming the integral to surface integral using Stoke’s Theorem.<br />

<br />

2<br />

4. Verify Stoke’s Theorem for A 3 yiˆ<br />

xzj ˆ yz kˆ<br />

, where S is the surface of the paraboloid 2z = x<br />

2<br />

+ y 2<br />

bounded by z = 2 and c is its boundary traversed in the clockwise direction.<br />

Ans. – 20 <br />

5. Evaluate<br />

F<br />

<br />

d R where<br />

C<br />

Ans.<br />

3<br />

a<br />

.<br />

3<br />

<br />

ˆ 3<br />

F yi xz ˆ j zy<br />

3ˆ k,<br />

C is the circl x 2 + y2 = 4, z = 1.5 Ans. 19 2 <br />

6. If S is the surface of the sphere x 2 + y 2 + z 2 <br />

= 9. Prove that curl F ds = 0.<br />

S<br />

7. Verify Stoke’s Theorem for the <strong>vector</strong> field<br />

<br />

F (2 y z) iˆ<br />

( x – z) ˆj ( y – x)<br />

kˆ<br />

over the portion of the plane x + y + z = 1 cut off by the co-ordinate planes.<br />

8. Evaluate <br />

dr by Stoke’s Theorem for F yziˆ zx ˆ j xyk and C is the curve of intersection of<br />

c<br />

x 2 + y 2 = 1 and y = z 2 . Ans. 0<br />

9.<br />

<br />

If ˆ 3 ˆ 2<br />

F ( x – z) i ( x yz) j 3xy kˆ<br />

and S is the surface of the cone z = a – 2 2<br />

( x y ) above the<br />

xy-plane, show that<br />

<br />

curl FdS = 3 a 4 / 4.<br />

s<br />

<br />

10. If F 3yiˆ<br />

xyj ˆ yz2kˆ<br />

and S is the surface of the paraboloid 2z = x 2 + y 2 bounded by z = 2, show by<br />

<br />

using Stoke’s Theorem that ( F)<br />

dS = 20 .<br />

s<br />

<br />

2 2 2 2 2 2 2 2 2<br />

11. If F ( y z – x ) iˆ<br />

( z x – y ) ˆj ( x y – z ) kˆ<br />

, evaluate curl F nds ˆ integrated over<br />

the portion of the surface x 2 + y 2 – 2ax + az = 0 above the plane z = 0 and verify Stoke’s Theorem; where<br />

ˆn is unit <strong>vector</strong> normal to the surface. (A.M.I.E.T.E., Winter 20002) Ans. 2 a 3<br />

where C is the boundary of<br />

C<br />

12. Evaluate by using Stoke’s Theorem sin zdx cos x dy sin<br />

ydz<br />

rectangle 0 x, 0 y1, z 3 . (AMIETE, June 2010)<br />

5.40 GAUSS’S THEOREM OF DIVERGENCE<br />

(Relation between surface integral and volume integral)<br />

(U.P., Ist Semester, Jan., 2011, Dec, 2006)<br />

Statement. The surface integral of the normal component of a <strong>vector</strong> function F taken around<br />

a closed surface S is equal to the integral of the divergence of F taken over the volume V enclosed<br />

by the surface S.<br />

Mathematically<br />

<br />

S<br />

<br />

F.<br />

nˆ<br />

ds <br />

<br />

V<br />

<br />

div Fdw


Vectors 453<br />

<br />

Proof. Let F Fi ˆ ˆ ˆ<br />

1 F2 j F3 k.<br />

<br />

Putting the values of F,<br />

nˆ<br />

in the statement of the divergence theorem, we have<br />

Fi ˆ ˆ<br />

1 F ˆ 2 j F3 k <br />

nds ˆ<br />

S<br />

= ˆ<br />

<br />

ˆ k ˆ <br />

( ˆ<br />

V F 1 F 2<br />

x y z<br />

ˆ F ˆ<br />

3 ) .<br />

1 2 3<br />

= F<br />

F<br />

F<br />

<br />

dx dy dz<br />

V<br />

<br />

x y z<br />

<br />

We require to prove (1).<br />

...(1)<br />

F3<br />

Let us first evaluate dx dy dz .<br />

V z<br />

V<br />

<br />

z<br />

F3<br />

dx dy dz<br />

=<br />

z f2 ( x, y)<br />

F3<br />

<br />

R<br />

dz dx dy<br />

z<br />

f1( x, y)<br />

z<br />

<br />

<br />

<br />

[ F ( x, y, f ) F ( x, y, f )] dx dy<br />

...(2)<br />

=<br />

R<br />

3 2 3 1<br />

For the upper part of the surface i.e. S 2<br />

, we have<br />

dx dy = ds 2<br />

cos r 2<br />

= ˆn 2<br />

. ˆk ds 2<br />

Again for the lower part of the surface i.e. S 1<br />

, we have,<br />

R<br />

dx dy = – cos r 1<br />

, ds 1<br />

= ˆn 1<br />

. ˆk ds 1<br />

F3 ( x, y, f2)<br />

dx dy ˆ ˆ<br />

F n kds<br />

S<br />

= 3 2 2<br />

= 3 1 1<br />

and F3 ( x, y, f1)<br />

dx dy ˆ ˆ<br />

R F n kds<br />

S1<br />

Putting these values in (2), we have<br />

F3<br />

dv F nˆ<br />

kds ˆ F nˆ<br />

kˆ<br />

ds<br />

V z<br />

S2 S1<br />

Similarly, it can be shown that<br />

F2<br />

dv<br />

ˆ<br />

V<br />

<br />

ˆ<br />

y<br />

F n<br />

jds<br />

S<br />

...(4)<br />

= 2<br />

2<br />

= 3 2 2 3 1 1 = 3<br />

F1<br />

dv =<br />

ˆ<br />

V<br />

1<br />

ˆ<br />

x<br />

F ni<br />

ds<br />

S<br />

...(5)<br />

Adding (3), (4) & (5), we have<br />

<br />

V<br />

F1 F2<br />

F3<br />

<br />

dv<br />

x y z<br />

<br />

= ( Fi ˆ ˆ ˆ<br />

1 F2 j F3 k)<br />

nˆ<br />

ds<br />

<br />

( F)<br />

dv<br />

V<br />

S<br />

= ˆ<br />

F nds<br />

Proved.<br />

S<br />

Example 102. State Gauss’s Divergence theorem<br />

<br />

ˆ ˆ<br />

F n<br />

kds ...(3)<br />

S<br />

F . nds ˆ Div Fdv<br />

S <br />

where S is the<br />

<br />

surface of the sphere x 2 + y 2 + z 2 <br />

= 16 and F 3xiˆ<br />

4y ˆj 5 zkˆ<br />

.<br />

(Nagpur University, Winter 2004)<br />

Solution. Statement of Gauss’s Divergence theorem is given in Art 24.8 on page 597.<br />

Thus by Gauss’s divergence theorem,<br />

<br />

= <br />

. Here F 3xiˆ<br />

4y ˆj 5zkˆ<br />

F . nds ˆ<br />

S<br />

v<br />

<br />

Fdv


454 Vectors<br />

. F <br />

=<br />

Putting the value of . F, we get<br />

Example 103. Evaluate<br />

<br />

<br />

ˆ<br />

<br />

ˆ<br />

ˆ <br />

i j k .(3xiˆ 4yj ˆ 5 zkˆ<br />

)<br />

x y z<br />

. F <br />

= 3 + 4 + 5 = 14<br />

= 14 .<br />

v<br />

F . nds ˆ<br />

S<br />

<br />

S<br />

dv<br />

where v is volume of a sphere<br />

= 14 v<br />

4 3 3584 <br />

= 14 (4)<br />

<br />

3 3<br />

Ans.<br />

<br />

ˆ 2<br />

F . nds ˆ where F 4 xzi – y ˆj yz kˆ<br />

and S is the surface of the<br />

cube bounded by x = 0, x = 1, y = 0, y = 1, z = 0, z = 1.<br />

(U.P., Ist Semester, 2009, Nagpur University, Winter 2003)<br />

Solution. By Divergence theorem,<br />

<br />

<br />

F . nds ˆ = ( . F)<br />

dv<br />

S<br />

<br />

v<br />

=<br />

=<br />

<br />

v<br />

v<br />

<br />

<br />

<br />

ˆ<br />

<br />

ˆ<br />

ˆ <br />

ˆ 2<br />

i j k .(4 xzi – y ˆj yz kˆ<br />

<br />

) dv<br />

x y z<br />

2 <br />

(4 xz) (– y ) ( yz)<br />

dxdydz<br />

x y z <br />

= (4 z –2 y y)<br />

dxdydz<br />

=<br />

=<br />

=<br />

v<br />

<br />

v<br />

2<br />

1 14z<br />

<br />

(4 z – y) dx dy dz – yz dx dy<br />

0<br />

0<br />

2 <br />

<br />

1 1 2 1<br />

1 1<br />

0 0<br />

0<br />

0 0<br />

<br />

(2 z – yz) dxdy (2 – y)<br />

dx dy<br />

1<br />

2<br />

1<br />

<br />

1<br />

y 3<br />

2 y – dx dx<br />

<br />

2 2<br />

=<br />

0 0<br />

0<br />

Note: This question is directly solved as on example 14 on Page 574.<br />

1<br />

0<br />

3 1 3 3<br />

[ x ] 0 (1) Ans.<br />

2 2 2<br />

<br />

2<br />

Example 104. Find Fn ˆ ds,<br />

where F (2x 3) z iˆ<br />

– ( xz y) ˆj ( y 2 z)<br />

kˆ<br />

and S is<br />

the surface of the sphere having centre (3, – 1, 2) and radius 3.<br />

(AMIETE, Dec. 2010, U.P., I Semester, Winter 2005, 2000)<br />

Solution. Let V be the volume enclosed by the surface S.<br />

By Divergence theorem, we’ve<br />

<br />

<br />

<br />

<br />

Fnˆ<br />

ds = div Fdv.<br />

S V<br />

Now, div F 2<br />

= (2x 3) z [ ( xz y)] ( y 2 z)<br />

= 2 – 1 + 2 = 3<br />

x y z<br />

<br />

<br />

= 3<br />

Fnˆ<br />

ds<br />

S dv = 3<br />

V dv = 3V.<br />

V<br />

Again V is the volume of a sphere of radius 3. Therefore<br />

4 3 4 3<br />

V = r = (3) = 36 .<br />

3 3<br />

<br />

Fn ˆ ds = 3V = 3 × 36 = 108 Ans.<br />

S


Vectors 455<br />

Example 105. Use Divergence Theorem to evaluate<br />

A <br />

S ds ,<br />

where 3 3<br />

A x iˆ<br />

y ˆj z<br />

3ˆ k and S is the surface of the sphere x2 + y2 + z2 = a2 .<br />

(AMIETE, Dec. 2009)<br />

Solution.<br />

=<br />

<br />

A <br />

<br />

S ds = div AdV<br />

v<br />

<br />

ˆ<br />

<br />

ˆ<br />

ˆ 3 ˆ 3ˆ<br />

3<br />

i j k ( x i y j z kˆ<br />

) dV<br />

x y z<br />

v<br />

2 2 2<br />

(3 x 3y 3 z ) dV =<br />

v<br />

2 2 2<br />

=<br />

3 ( x y z ) dV<br />

v<br />

On putting x = r sin cos , y = r sin sin , z = r cos , we get<br />

3 r ( r sin dr dd)<br />

= 3 × 8<br />

v<br />

=<br />

2 2<br />

<br />

<br />

2 2<br />

<br />

d sin d<br />

r dr<br />

0 0<br />

5<br />

a<br />

5 5<br />

<br />

= 24 ( )<br />

2<br />

( cos )<br />

2<br />

r a<br />

12a<br />

<br />

0 0<br />

<br />

= 24 ( 0 1)<br />

<br />

5 2 5 5<br />

Ans.<br />

<br />

<br />

<br />

0<br />

Example 106. Use divergence Theorem to show that<br />

2 2 2<br />

( x y z ) d <br />

s = 6 V<br />

S<br />

where S is any closed surface enclosing volume V. (U.P., I Semester, Winter 2002)<br />

<br />

Solution. Here (x 2 + y 2 + z 2 ˆ 2 2 2<br />

) =<br />

ˆ<br />

<br />

i ˆ<br />

<br />

j k ( x y z )<br />

x y z<br />

= 2 xiˆ 2 yj ˆ 2 zkˆ 2 ( xiˆ yj ˆ zkˆ)<br />

<br />

2 2 2<br />

( x y z ) ds<br />

=<br />

S<br />

<br />

S<br />

2 2 2<br />

ˆ<br />

( x y z ) nds<br />

ˆn being outward drawn unit normal <strong>vector</strong> to S<br />

= 2( xiˆ yj ˆ zkˆ<br />

) nds ˆ<br />

<br />

S<br />

a<br />

0<br />

4<br />

= 2 ( ˆ ˆ ˆ<br />

div xi yj zk)<br />

dv<br />

...(1)<br />

V<br />

(By Divergence Theorem)<br />

(V being volume enclosed by S)<br />

Now,<br />

<br />

div. ( xiˆ<br />

yj ˆ zkˆ<br />

ˆ<br />

ˆ<br />

) =<br />

ˆ<br />

<br />

ˆ<br />

<br />

i j k ( xiˆ y ˆj zk)<br />

x y z <br />

x y z<br />

=<br />

<br />

<br />

<br />

x y z<br />

= 3 ...(2)<br />

From (1) & (2), we have<br />

2 2 2<br />

( y z ) dS dv dv = 6 V<br />

V<br />

Proved.<br />

= 2 3 = 6<br />

V<br />

Example 107. Evaluate ˆ ˆ<br />

( y z i z x j z y k ) n ˆ dS , where S is the part of the sphere<br />

S<br />

x 2 + y 2 + z 2 = 1 above the xy-plane and bounded by this plane.<br />

Solution. Let V be the volume enclosed by the surface S. Then by divergence Theorem, we<br />

have<br />

2 2 2 2 2<br />

( y z i ˆ z x ˆ j z y 2ˆ k ) n ˆ dS<br />

2 2 2 2 2<br />

S<br />

= div ( y z iˆ<br />

z x ˆj z yk<br />

2ˆ)<br />

dV<br />

=<br />

<br />

x y z<br />

2 2 2 2 2 2ˆ<br />

<br />

<br />

2 2 2 2 2 2<br />

( y z ) ( z x ) ( z y ) dV<br />

V<br />

<br />

<br />

2 2 2<br />

2<br />

V V<br />

V<br />

<br />

<br />

zy dV zy dV


456 Vectors<br />

Changing to spherical polar coordinates by putting<br />

x = r sin cos , y= r sin sin , z = r cos , dV = r 2 sin dr d d<br />

To cover V, the limits of r will be 0 to 1, those of will be 0 to 2<br />

and those of will be 0 to<br />

2.<br />

<br />

2<br />

2<br />

dV =<br />

V zy<br />

=<br />

=<br />

=<br />

2 /2 1 2 2 2 2<br />

<br />

2 ( r cos )( r sin sin ) r sin drd d<br />

0 0 0<br />

2 /2 1 5 3 2<br />

<br />

2 r sin cos sin<br />

drd d<br />

<br />

0 0 0<br />

<br />

6<br />

1<br />

2 2<br />

3 2<br />

r<br />

<br />

0 <br />

<br />

0<br />

6 0<br />

2 sin cos sin<br />

d d<br />

2 2<br />

=<br />

2<br />

2<br />

sin <br />

6 0 4.2 d<br />

1<br />

12<br />

2<br />

2 <br />

sin d<br />

=<br />

0<br />

12<br />

Example 108. Use Divergence Theorem to evaluate FdS<br />

2<br />

where F 4 xiˆ<br />

–2y ˆj z<br />

2ˆ k<br />

S<br />

and S is the surface bounding the region x 2 + y 2 = 4, z = 0 and z = 3.<br />

(A.M.I.E.T.E., Summer 2003, 2001)<br />

Solution. By Divergence Theorem,<br />

FdS<br />

<br />

= div FdV<br />

S V =<br />

<br />

<br />

V<br />

<br />

ˆ<br />

<br />

ˆ<br />

ˆ <br />

ˆ 2ˆ<br />

2<br />

i j k (4 xi 2 y j z kˆ<br />

) dV<br />

x y z<br />

= (4 4y<br />

2 z)<br />

dx dy dz<br />

=<br />

V<br />

3<br />

dx dy (4 4y 2 z)<br />

dz =<br />

= (12 12y<br />

9)<br />

dxdy<br />

Let us put x = r cos , y = r sin <br />

= (21 12r sin )<br />

r d<br />

dr<br />

=<br />

0<br />

= (21 12 )<br />

=<br />

2<br />

2<br />

2<br />

2 2<br />

21r<br />

3<br />

<br />

d 4r<br />

sin <br />

2<br />

0 <br />

<br />

= <br />

0 0<br />

<br />

0<br />

<br />

<br />

2 2<br />

0<br />

<br />

<br />

2 3<br />

dx dy [4z 4 yz z ] 0<br />

y dxdy<br />

d (21r 12r sin )<br />

dr<br />

<br />

<br />

d (42 32 sin ) (42 32 cos )<br />

2<br />

0<br />

Ans.<br />

= 84 + 32 – 32 = 84 Ans.<br />

^<br />

Example 109. Apply the Divergence Theorem to compute u nds, where s is the surface of<br />

the cylinder x 2 + y 2 = a 2 bounded by the planes z = 0, z = b and where u ix ˆ – ˆjy kz ˆ .<br />

Solution. By Gauss’s Divergence Theorem<br />

u nds<br />

<br />

ˆ = ( u)<br />

dv<br />

V<br />

<br />

ˆ<br />

ˆ<br />

=<br />

ˆ<br />

<br />

ˆ<br />

<br />

i j k ( ix ˆ ˆ<br />

<br />

jy kzdv )<br />

V<br />

<br />

x y z<br />

= x y z<br />

dv<br />

V<br />

<br />

x y z<br />

= 111<br />

dv<br />

V<br />

= dv dx dy dz<br />

V = Volume of the cylinder = a 2 b Ans.<br />

=<br />

V


Vectors 457<br />

Example 110. Apply Divergence Theorem to evaluate<br />

F . ˆ , where<br />

V<br />

F<br />

4x 3 iˆ x 2 y ˆj x 2 zk ˆ and S is the surface of the cylinder x 2 + y 2 = a 2 bounded by the<br />

planes z = 0 and z = b.<br />

Solution. We have,<br />

(U.P. Ist Semester, Dec. 2006)<br />

F<br />

4x 3 iˆ<br />

x 2 y ˆj x 2 zk ˆ<br />

<br />

ˆ<br />

3 2 2 ˆ<br />

div F =<br />

ˆ<br />

<br />

ˆ<br />

<br />

i j k (4 x iˆ x yj ˆ x zk)<br />

x y z<br />

3 2 2<br />

= (4 x ) ( x y) ( x z)<br />

x y z<br />

= 12x 2 – x 2 + x 2 = 12 x 2<br />

Now,<br />

V<br />

<br />

div FdV<br />

=<br />

2 2<br />

12 a a <br />

x b<br />

xay a<br />

2<br />

x<br />

2<br />

z 0<br />

x<br />

2<br />

dzdydx<br />

a a<br />

2<br />

x<br />

2<br />

2 b<br />

xa y a<br />

2<br />

x<br />

2<br />

=<br />

= 12 x ( z)<br />

0 dydx<br />

a<br />

a<br />

<br />

a 2 2<br />

2 a <br />

x<br />

a a<br />

2<br />

x<br />

2<br />

12 b x ( y)<br />

dx<br />

2 2 2<br />

2 2 2<br />

= 12 b x .2 a x dx = 24 b<br />

x a x dx<br />

=<br />

=<br />

<br />

a<br />

2 2 2<br />

48 b x a x dx<br />

<br />

0<br />

/2 2 2<br />

48 b a sin a cos a cos d<br />

<br />

0<br />

a<br />

a<br />

[Put x = a sin , dx = a cos d]<br />

3 3<br />

4 /2<br />

2 2<br />

= 48 ba sin cos<br />

d<br />

4<br />

= 48 ba<br />

2 2<br />

0<br />

23<br />

1 1<br />

<br />

4 2 2<br />

= 48 ba<br />

= 3 b a 4 Ans.<br />

22 <br />

Example 111. Evaluate surface integral Fnds ˆ ,<br />

<br />

<br />

where F = (x 2 + y 2 + z 2 ) iˆ<br />

ˆj kˆ<br />

( ), S<br />

is the surface of the tetrahedron x = 0, y = 0, z = 0, x + y + z = 2 and n is the unit normal in<br />

the outward direction to the closed surface S.<br />

Solution. By Divergence theorem<br />

Fnds<br />

<br />

ˆ = div Fdv <br />

S V<br />

where S is the surface of tetrahedron x = 0, y = 0, z = 0, x + y + z = 2<br />

=<br />

<br />

<br />

<br />

i ˆ<br />

<br />

ˆ j k ˆ 2 2 2<br />

( x y z )( i ˆ ˆ j k ˆ)<br />

dv<br />

V<br />

<br />

x y z<br />

= (2x 2y 2 z)<br />

dv<br />

V<br />

<br />

= 2 ( x y z)<br />

dx dy dz<br />

=<br />

=<br />

V<br />

2 2x 2 x y<br />

<br />

2 dx dy ( x y z)<br />

dz<br />

2<br />

<br />

0 0 0<br />

2<br />

2 x<br />

y<br />

2 2 x <br />

z<br />

dx dy <br />

xz<br />

yz<br />

2 <br />

<br />

<br />

0 0<br />

0


458 Vectors<br />

=<br />

<br />

2<br />

2 2 x <br />

2 2 (2 x<br />

y)<br />

<br />

dx dy x x xy y xy y<br />

0 <br />

<br />

<br />

0 <br />

2 2 2<br />

<br />

3 3<br />

2 <br />

2 2 2 y (2 x<br />

y)<br />

<br />

= 2<br />

dx 2xy x y xy y <br />

<br />

0<br />

<br />

3 6 <br />

2<br />

2<br />

x<br />

3 3<br />

2 <br />

2 2 2 (2 x) (2 x)<br />

<br />

= 2<br />

dx 2 x(2 x) x (2 x) x (2 x) (2 x)<br />

<br />

0<br />

<br />

3 6 <br />

3 3<br />

2<br />

2 2 3 2 3 2 (2 x) (2 x)<br />

<br />

= 2<br />

4x2x 2x x 4x 4 x x (2 x)<br />

<br />

0 <br />

<br />

<br />

3 6 <br />

2<br />

<br />

3 4 3 4 3 4 4<br />

2 4 2 4 (2 ) (2 ) (2 )<br />

= 2 2 x x 2<br />

x x x x x <br />

x x <br />

3 4 3 4 3 12 24 <br />

<br />

3 4 4<br />

(2 x) (2 x) (2 x)<br />

8 16 16 <br />

= 2 = 2 <br />

3 12 24 3 12 24<br />

<br />

= 4 Ans.<br />

0<br />

Example 112. Use the Divergence Theorem to evaluate<br />

<br />

S<br />

( xdydz y dz dx zdxdy)<br />

where S is the portion of the plane x + 2 y + 3 z = 6 which lies in the first Octant.<br />

(U.P., I Semester, Winter 2003)<br />

<br />

Solution. ( f1 dydz f2 dx dz f3<br />

dxdy)<br />

S<br />

f1 f2<br />

f3<br />

<br />

= <br />

dxdydz<br />

V<br />

<br />

x y z<br />

<br />

where S is a closed surface bounding a volume V.<br />

<br />

( xdydz y dz dx zdxdy)<br />

S<br />

<br />

=<br />

V<br />

x y z<br />

dx dy dz<br />

x y z<br />

= 3 V 2<br />

= (1 11)<br />

dx dy dz dx dy dz<br />

V<br />

= 3 (Volume of tetrahedron OABC)<br />

1<br />

= 3[( Area of the base OAB) height OC]<br />

3<br />

1 1<br />

<br />

= 3<br />

63<br />

2<br />

3 2<br />

= 18 Ans.<br />

<br />

Example 113. Use Divergence Theorem to evaluate : ( xdydz y dz dx zdxdy)<br />

Solution.<br />

over the surface of a sphere radius a.<br />

Here, we have<br />

(K. University, Dec. 2009)<br />

xdydz y dx dz zdxdy<br />

<br />

S<br />

<br />

<br />

f1 f2<br />

f3<br />

<br />

x y z<br />

dx dy dz<br />

V<br />

<br />

dx dy dz<br />

x y z<br />

<br />

<br />

V <br />

x y z<br />

(1 + 1 + 1) dx dy dz = 3 (volume of the sphere)<br />

V<br />

4<br />

3 <br />

= 3 a<br />

<br />

3 = 4 a3 Ans.<br />

<br />

0<br />

<br />

0


Vectors 459<br />

Example 114. Using the divergence theorem, evaluate the surface integral<br />

( yz dy dz zxdz dx xy dy dx)<br />

where S : x 2 + y 2 + z 2 = 4.<br />

S<br />

<br />

Solution. ( f1 dydz f2 dx dz f3<br />

dxdy)<br />

S<br />

<br />

f1 f2<br />

f3<br />

<br />

= <br />

dx dy dz<br />

v<br />

<br />

x y z <br />

where S is closed surface bounding a volume V.<br />

<br />

( yz dy dz zx dx dz xy dx dy)<br />

S<br />

(AMIETE, Dec. 2010, UP, I Sem., Dec 2008)<br />

( ) ( ) ( )<br />

= yz zx xy <br />

dx dy dz<br />

v<br />

<br />

x y z <br />

= (0 0 0) dx dy dz<br />

v<br />

= 0 Ans.<br />

<br />

2 2 3 2<br />

Example 115. Evaluate xz dy dz ( x y z ) dzdx (2 xy y z)<br />

dxdy<br />

S<br />

where S is the surface of hemispherical region bounded by<br />

z =<br />

2 2 2<br />

a x y and z = 0.<br />

=<br />

Solution. ( f1 dydz f2 dz dx f3<br />

dxdy)<br />

S<br />

where S is a closed surface bounding a volume V.<br />

<br />

<br />

=<br />

S<br />

<br />

2 2 3 2<br />

xz dy dz ( x y z ) dzdx (2 xy y z)<br />

dxdy<br />

<br />

<br />

V<br />

V<br />

f1 f2<br />

f3<br />

<br />

dx dy dz<br />

x y z<br />

<br />

2 2 3 <br />

2 <br />

( xz ) ( x y z ) (2 xy y z)<br />

dxdydz<br />

x y z<br />

<br />

(Here V is the volume of hemisphere)<br />

2 2 2<br />

= ( z x y ) dx dy dz<br />

V<br />

Let x = r sin cos , y = r sin sin , z = r cos <br />

=<br />

=<br />

Example 116. Evaluate<br />

2 2<br />

r ( r sin dr d d )<br />

=<br />

5<br />

a<br />

2 /2<br />

r<br />

<br />

( ) 0 ( cos )<br />

0 <br />

<br />

5 <br />

<br />

0<br />

=<br />

<br />

2 a<br />

2 sin <br />

0 0 0<br />

<br />

d d r dr<br />

5<br />

2 ( 0 1) 5<br />

a<br />

=<br />

2a<br />

5<br />

4<br />

5<br />

Ans.<br />

F . n ˆ ds over the entire surface of the region above the xy-plane<br />

S<br />

bounded by the cone z 2 = x 2 + y 2 2<br />

and the plane z = 4, if F = 4xz iˆ<br />

xyz ˆj 3 zkˆ<br />

.<br />

Solution. If V is the volume enclosed by S, then V is bounded by the surfaces z = 0, z = 4,<br />

z 2 = x 2 + y 2 .<br />

By divergence theorem, we have<br />

=<br />

F . n ˆ ds<br />

S<br />

=<br />

=<br />

<br />

<br />

<br />

V<br />

V<br />

V<br />

<br />

div Fdxdydz<br />

2 <br />

(4 xz) ( xyz ) (3) z dxdydz<br />

x y z<br />

<br />

2<br />

(4z xz 3) dxdydz<br />

Limits of z are<br />

2 2<br />

x y and 4.


460 Vectors<br />

4 2<br />

x<br />

2<br />

y<br />

2 <br />

(4z xz 3) dzdydx =<br />

<br />

<br />

3<br />

2 xz <br />

2z 3z<br />

dy dx<br />

3 <br />

4<br />

x<br />

2<br />

y<br />

2<br />

<br />

64x<br />

2 2 2 2 3/2 2 2 <br />

= 32 12 {2( x y ) xx ( y ) 3 x y } dydx<br />

3<br />

<br />

<br />

<br />

<br />

64x<br />

2 2 2 2 3/2 2 2 <br />

= 44 2( x y ) xx ( y ) 3<br />

x y dydx<br />

3<br />

<br />

Putting x = r cos and y = r sin , we have<br />

64r<br />

cos 2 3 <br />

= 44 2r r cos r 3r<br />

rddr<br />

3<br />

<br />

Limits of r are 0 to 4.<br />

and limits of are 0 to 2<br />

=<br />

=<br />

=<br />

=<br />

=<br />

<br />

<br />

<br />

<br />

<br />

2<br />

2 4 64r<br />

cos 3 5 2<br />

44 2 cos 3<br />

<br />

0 <br />

0 <br />

<br />

<br />

r r r r ddr<br />

3<br />

<br />

3 4 6<br />

2 <br />

2 64 r cos r r<br />

3<br />

0<br />

2 <br />

0<br />

2 <br />

0<br />

2 <br />

0<br />

<br />

22r cos r d<br />

<br />

9 2 6 <br />

3 4 6<br />

2 64 (4) cos (4) (4)<br />

3<br />

22(4) cos (4)<br />

d<br />

<br />

9 2 6<br />

<br />

6<br />

64 64 (4) <br />

352 cos 128 cos 64<br />

d<br />

9 6<br />

<br />

<br />

6<br />

64 64 (4) <br />

160 cos d<br />

<br />

<br />

9 6 <br />

<br />

<br />

<br />

2<br />

<br />

6<br />

64 64 (4) <br />

<br />

6<br />

64 64 (4) <br />

= 160 <br />

sin <br />

9 6 = 160 (2 ) sin 2<br />

<br />

<br />

<br />

<br />

9 6 <br />

0<br />

<br />

<br />

= 320 Ans.<br />

Example 117. The <strong>vector</strong> field 2<br />

F x iˆ<br />

zj ˆ yzkˆ<br />

is defined over the volume of the cuboid<br />

given by 0 x a, 0 y b, 0 z c, enclosing the surface S. Evaluate the surface integral<br />

<br />

<br />

F . ds<br />

(U.P., I Semester, Winter 2001)<br />

S<br />

Solution. By Divergence Theorem, we have<br />

2 ˆ<br />

2<br />

( x iˆ zj ˆ yz k). ds div ( x iˆ zj ˆ yz kˆ) dv,<br />

<br />

S<br />

<br />

v<br />

where V is the volume of the cuboid enclosing the surface S.<br />

<br />

ˆ<br />

2<br />

ˆ<br />

=<br />

ˆ<br />

<br />

ˆ<br />

<br />

i j k .( x iˆ z ˆ<br />

<br />

j yzk)<br />

dv<br />

v<br />

<br />

x y z<br />

2 <br />

= ( x ) ( z) ( yz)<br />

dx dy dz<br />

v<br />

x y z <br />

=<br />

a b c<br />

=<br />

(2 x<br />

y ) dx dy dz<br />

0 0 0<br />

x y z <br />

a b a b<br />

c<br />

= 0<br />

0 0 0 0<br />

<br />

dx [2 xz yz] dy dx (2 xc yc)<br />

dy<br />

4<br />

0<br />

a b c<br />

<br />

dx dy (2 x y ) dz<br />

0 0 0


Vectors 461<br />

=<br />

=<br />

a b a b<br />

2 a<br />

2<br />

y b <br />

cdx(2 x y) dy c2xy dx c 2bx<br />

dx<br />

2<br />

2 <br />

<br />

0 0 0 0 0<br />

2 2<br />

a<br />

2<br />

2<br />

2<br />

bx b x ab b <br />

c c ab abc a<br />

<br />

2 2 2 2<br />

0<br />

Example 118. Verify the divergence Theorem for the function F = 2 x2 yi – y 2 j + 4 x z 2 k<br />

taken over the region in the first octant bounded by y 2 + z 2 = 9 and x = 2.<br />

Solution.<br />

<br />

FdV<br />

=<br />

V<br />

<br />

= (4 xy 2y 8 xz)<br />

dxdydz<br />

=<br />

=<br />

<br />

ˆ<br />

<br />

ˆ<br />

ˆ 2 ˆ 2ˆ<br />

2<br />

i j k (2x yi y j 4 xz k)<br />

dV<br />

x y z<br />

=<br />

<br />

<br />

2 3 9 y<br />

2<br />

<br />

dx dy (4 xy 2y 8 xz)<br />

dz<br />

0 0 0<br />

2 3 9<br />

2<br />

2 y<br />

dx (4 2 4 )<br />

0 dy xyz yz xz<br />

0<br />

0<br />

2 3 2 2 2<br />

<br />

dx [4 xy 9 y 2y 9 y 4 x(9 y )] dy<br />

0 0<br />

3<br />

2 4x<br />

2 2 3/2 2 2 3/2 4xy<br />

<br />

= dx (9 y ) (9 y ) 36<br />

xy <br />

0<br />

2 3 3 3 0<br />

2<br />

2<br />

= (0 0 108 x 36 x 36 x 18) dx<br />

2<br />

= (108 x 18) dx<br />

<br />

2<br />

x <br />

0 = 108 18 x<br />

0<br />

2 0<br />

= 216 – 36 = 180 ...(1)<br />

Here Fnds<br />

<br />

ˆ = Fnˆ ds Fnˆ ds Fnˆ ds Fnˆ ds F<br />

nˆ<br />

ds<br />

S<br />

OABC OCE OADE ABD BDEC<br />

F nˆ<br />

2 2 2<br />

ds (2 x yiˆ<br />

y ˆj 4 xzkˆ<br />

). nds ˆ<br />

=<br />

BDEC<br />

<br />

BDEC<br />

Normal <strong>vector</strong><br />

<br />

= = ˆ<br />

<br />

i ˆ<br />

<br />

j kˆ<br />

<br />

<br />

x y z (y2 + z 2 – 9)<br />

= 2 yj ˆ 2 zkˆ<br />

2 yj ˆ 2zkˆ<br />

yj ˆ zkˆ<br />

Unit normal <strong>vector</strong> = ˆn =<br />

=<br />

2 2 2 2<br />

4y<br />

4z<br />

y z<br />

yj ˆ zkˆ<br />

yj ˆ zkˆ<br />

= =<br />

9 3<br />

ˆ<br />

2 ˆ 2ˆ<br />

2 yj zk 1<br />

(2x yi y j 4 xzk)<br />

ds<br />

3 3<br />

= ( 4 )<br />

3<br />

BDEC<br />

3<br />

y xz ds<br />

BDEC<br />

<br />

<br />

<br />

yj ˆ zkˆ<br />

<br />

( ˆ ) ˆ z dxdy <br />

dx dy ds nk ds k ds or ds <br />

<br />

3 3 z<br />

<br />

<br />

<br />

3 <br />

3<br />

1<br />

3 3 dxdy<br />

2 3<br />

y 2<br />

<br />

= ( y 4 xz )<br />

3<br />

= dx 4 xz dy<br />

z<br />

0 <br />

<br />

0<br />

z <br />

BDEC<br />

<br />

<br />

3<br />

3<br />

2<br />

27 sin <br />

2<br />

2<br />

<br />

= dx<br />

4 x (9cos )<br />

0 <br />

0<br />

3cos <br />

<br />

3<br />

Ans.<br />

y<br />

3sin ,<br />

<br />

<br />

z<br />

3cos


462 Vectors<br />

=<br />

=<br />

F nˆ<br />

ds =<br />

OABC<br />

<br />

<br />

=<br />

2 2 2<br />

dx 27 108 x<br />

0<br />

<br />

3 3<br />

=<br />

2<br />

<br />

2<br />

0<br />

( 18 72 x ) dx<br />

<br />

2<br />

18x<br />

36 x <br />

<br />

<br />

<br />

= 108 ...(2)<br />

0<br />

2 2 2<br />

(2 x yiˆ y ˆj 4 xz kˆ) ( kˆ)<br />

ds<br />

<br />

OABC<br />

<br />

2<br />

4 xz ds = 0 ...(3) because in OABC xy-plane, z = 0<br />

OABC<br />

F nˆ<br />

ds =<br />

OADE<br />

<br />

<br />

Fnds ˆ<br />

=<br />

OCE<br />

<br />

<br />

Fnds ˆ =<br />

ABD<br />

<br />

<br />

=<br />

(2 ˆ ˆ 4 ) ( ˆ<br />

x yi y j xzkj)<br />

ds =<br />

OADE<br />

2 2 2 ˆ<br />

(2 ˆ ˆ 4 ) ( ˆ<br />

x yi y j xzki)<br />

ds =<br />

OCE<br />

2 2 2 ˆ<br />

(2 x yiˆ y ˆj 4 xzk) () iˆ<br />

ds =<br />

2<br />

2 x y ds<br />

ABD<br />

2<br />

2 x y dy dz =<br />

2 2 2 ˆ<br />

9 z<br />

2<br />

<br />

2<br />

y ds = 0 ...(4)<br />

OADE<br />

because in OADE xz-plane, y = 0<br />

2<br />

2 x y ds = 0 ...(5)<br />

OCE<br />

<br />

because in OCE yz-plane, x = 0<br />

ABD<br />

3 9<br />

z<br />

2<br />

2<br />

dz 2(2) ydy<br />

0 <br />

because in ABD plane, x = 2<br />

0<br />

2<br />

3<br />

= 8 y<br />

<br />

dz <br />

0<br />

2<br />

=<br />

3 <br />

3<br />

2<br />

z <br />

4 dz (9 z ) = 49z<br />

<br />

0<br />

<br />

3<br />

0<br />

<br />

On adding (2), (3), (4), (5) and (6), we get<br />

<br />

<br />

3<br />

0<br />

= 4 [27 – 9] = 72 ...(6)<br />

Fnds ˆ = 108 + 0 + 0 + 0 + 72 = 180 ...(7)<br />

S<br />

From (1) and (7), we have<br />

V<br />

<br />

FdV<br />

Fnds<br />

<br />

<br />

S<br />

<br />

= <br />

ˆ<br />

Hence the theorem is verified.<br />

Example 119. Verify the Gauss divergence Theorem for<br />

= (x 2 – yz) î + (y 2 – zx) ĵ + (z 2 – xy) ˆk taken over the rectangular parallelopiped<br />

0 x a, 0 y b, 0 z c. (U.P., I Semester, Compartment 2002)<br />

Solution. We have<br />

div F = F <br />

ˆ<br />

2 2 2 ˆ<br />

=<br />

ˆ<br />

<br />

ˆ<br />

<br />

i j k [( x yz) iˆ( y zx) ˆj ( z xy) k]<br />

x y z<br />

2 2 2<br />

= ( x yz) ( y zx) ( z xy)<br />

= 2x + 2y + 2z<br />

x y z<br />

F <br />

Volume integral =<br />

=<br />

=<br />

<br />

FdV<br />

= 2( x y z)<br />

dV<br />

V<br />

V<br />

a b c<br />

=<br />

2 ( x y z ) dx dy dz<br />

2<br />

x 0 y 0 z<br />

0<br />

<br />

dx dy <br />

xz yz <br />

<br />

2<br />

a b z<br />

0 0<br />

c<br />

<br />

2 <br />

<br />

=<br />

0<br />

b<br />

2 2<br />

= 2 a y c y <br />

dx cxyc<br />

<br />

0 <br />

2 2 <br />

<br />

<br />

= 2<br />

0<br />

2<br />

<br />

<br />

a b c<br />

<br />

2 dx dy ( x y z)<br />

dz<br />

0 0 0<br />

<br />

dx dy <br />

cx cy <br />

<br />

2<br />

a b c<br />

0 0<br />

a bc bc<br />

dx bcx<br />

0 <br />

<br />

2 2<br />

2 2<br />

<br />

2


Vectors 463<br />

a<br />

<br />

2 2 2<br />

bc x b cx bc x <br />

= 2 = [a<br />

2 2 2<br />

2 bc + ab 2 c + abc 2 ]<br />

0<br />

= abc (a + b + c) ...(A)<br />

<br />

<br />

To evaluate Fnds ˆ , where S consists of six plane surfaces.<br />

S<br />

<br />

<br />

= ˆ ˆ ˆ<br />

OABC <br />

DEFG OAFG<br />

Fnds ˆ<br />

S<br />

Fnds <br />

OABC<br />

<br />

ˆ =<br />

<br />

<br />

F nds ˆ<br />

DEFG<br />

<br />

Fnds Fnds Fnds<br />

<br />

<br />

Fnds ˆ Fnds ˆ Fnds ˆ<br />

=<br />

=<br />

=<br />

=<br />

=<br />

<br />

<br />

BCDE ABEF OCDG<br />

OABC<br />

ab<br />

<br />

00<br />

ab<br />

00<br />

2 2 2<br />

{( x yzi ) ˆ ( y xz) ˆj ( z xy) kˆ}( kˆ)<br />

dxdy<br />

2<br />

( z xy)<br />

dxdy<br />

(0 xy)<br />

dx dy =<br />

<br />

DEFG<br />

ab<br />

ab<br />

2<br />

( z xy)<br />

dxdy =<br />

<br />

00<br />

00<br />

a<br />

2<br />

b a<br />

<br />

2 xy <br />

c y dx<br />

2<br />

= <br />

0 0<br />

0<br />

<br />

2 2<br />

2 x b <br />

= cbx<br />

<br />

4 <br />

Fnds<br />

<br />

<br />

OAFG<br />

<br />

ˆ =<br />

=<br />

=<br />

<br />

<br />

a<br />

OAFG<br />

c<br />

0 0<br />

<br />

ˆ<br />

2<br />

<br />

a<br />

0<br />

=<br />

OAFG<br />

( y zx)<br />

dxdz<br />

dx<br />

(0 zx)<br />

dz =<br />

2 2<br />

a b<br />

4<br />

...(1)<br />

2 2 2<br />

{( x yz) iˆ<br />

( y xz) ˆj ( z xy) kˆ}( kˆ)<br />

dxdy<br />

2<br />

( c xy)<br />

dxdy<br />

<br />

2 xb<br />

cb<br />

<br />

2<br />

2 2<br />

2<br />

<br />

dx<br />

<br />

<br />

2 a b<br />

abc ...(2)<br />

4<br />

{( x yz) iˆ ( y zx) ˆj ( z xy) k}( ˆj)<br />

dxdz<br />

2 2 2 ˆ<br />

a<br />

2<br />

xz<br />

dx <br />

<br />

<br />

2 <br />

=<br />

<br />

c<br />

0 0<br />

a xc<br />

2<br />

dx =<br />

2<br />

0<br />

x c<br />

<br />

4<br />

F nds 2 2 2<br />

= {( x yz ) i ˆ ( y zx ) ˆ j ( z xy ) k ˆ}<br />

ˆ jdxdz<br />

BCDE = <br />

Fnds<br />

<br />

<br />

ABEF<br />

=<br />

<br />

a c<br />

a<br />

2<br />

c<br />

<br />

2<br />

2 xz <br />

dx ( b xz)<br />

dz =<br />

<br />

b z <br />

2 <br />

0 0<br />

0 0<br />

2 2<br />

a<br />

2 2<br />

2 x c 2 ac<br />

2 2<br />

dx =<br />

<br />

<br />

<br />

a<br />

0<br />

a<br />

<br />

0<br />

=<br />

2 2<br />

a c<br />

4<br />

BCDE<br />

<br />

2 xc<br />

<br />

b c <br />

2<br />

2<br />

...(3)<br />

( y xz)<br />

dxdz<br />

2<br />

<br />

<br />

dx<br />

<br />

<br />

= b cx = ab c ...(4)<br />

4 <br />

4<br />

0<br />

2 2 2<br />

<br />

ˆ<br />

ˆ<br />

= {( ) ˆ ( ) ˆ ( ) } ˆ<br />

x yz i y xz j z x y k idydz<br />

ABEF<br />

=<br />

2<br />

( x yz)<br />

dydz =<br />

ABEF<br />

S.No.<br />

Surface Outward normal<br />

ds<br />

1 OABC – k dx dy z = 0<br />

2 DEFG k dx dy z = c<br />

3 OAFG – j dx dz y = 0<br />

4 BCDE j dx dz y = b<br />

5 ABEF i dy dz x = a<br />

6 OCDG – i dy dz x = 0<br />

b c<br />

b<br />

2<br />

c<br />

2<br />

<br />

2 yz <br />

dy ( a yz)<br />

d z = dy<br />

<br />

a z <br />

2 <br />

0 0<br />

0 0


464 Vectors<br />

Fnds<br />

<br />

<br />

OCDG<br />

=<br />

<br />

ˆ =<br />

=<br />

b<br />

<br />

2<br />

2 yc <br />

<br />

ac<br />

dy<br />

2 <br />

0 <br />

=<br />

<br />

OCDG<br />

<br />

2 2<br />

2 yc <br />

acy<br />

<br />

4 <br />

2 2 2 ˆ<br />

b<br />

0<br />

=<br />

2 2<br />

2 b c<br />

a bc ...(5)<br />

4<br />

{( x yz) iˆ ( y zx) ˆj ( z x y) k} ( iˆ<br />

) dydz<br />

b c<br />

x 2 yz dydz<br />

=<br />

0 0 ( )<br />

2<br />

b yc <br />

2 2<br />

y c <br />

= dy = =<br />

0 2 4 0<br />

Adding (1), (2), (3), (4), (5) and (6), we get<br />

b<br />

<br />

b<br />

c<br />

<br />

=<br />

dy ( yz ) dz<br />

0 0<br />

2 2<br />

b c<br />

4<br />

2<br />

b yz<br />

dy <br />

<br />

<br />

0<br />

2 <br />

c<br />

0<br />

...(6)<br />

Fnds<br />

<br />

ˆ<br />

2 2 2 2 2 2 2 2<br />

2 2<br />

=<br />

a b abc a b a c ab c <br />

a c <br />

4 4 4 4 <br />

<br />

<br />

2 2 2 2<br />

b c <br />

2 b c <br />

a bc <br />

4 4 <br />

<br />

= abc 2 + ab 2 c + a 2 bc<br />

= abc (a + b + c) ...(B)<br />

From (A) and (B), Gauss divergence Theorem is verified.<br />

Verified.<br />

<br />

2<br />

Example 120. Verify Divergence Theorem, given that F 4 xziˆ<br />

– y ˆj yz kˆ<br />

and S is the<br />

surface of the cube bounded by the planes x = 0, x = 1, y = 0, y = 1, z = 0, z = 1.<br />

Solution.<br />

Volume Integral =<br />

F <br />

ˆ<br />

2 ˆ<br />

=<br />

ˆ<br />

<br />

ˆ<br />

<br />

i j k (4 zxiˆ y ˆj yzk)<br />

x y z<br />

= 4 z – 2 y + y<br />

= 4 z – y<br />

<br />

Fdv<br />

<br />

= (4 z y)<br />

dx dy dz<br />

=<br />

1 1 1<br />

<br />

dx dy (4 z y ) dz<br />

0 0 0<br />

=<br />

=<br />

1 dx 1 dy 2 1<br />

(2 z yz )<br />

0 0<br />

0<br />

=<br />

<br />

2<br />

1 y <br />

dx<br />

2 y <br />

0 2 =<br />

<br />

1<br />

0<br />

<br />

1 1<br />

<br />

dx dy (2 y)<br />

0 0<br />

<br />

1 1<br />

dx 2 <br />

<br />

0<br />

<br />

2 <br />

= 3 1 3 ( ) 1<br />

0<br />

0<br />

2 dx x = 3 2 2<br />

...(1)<br />

To evaluate Fnds ˆ , where S consists of six plane surfaces.<br />

S<br />

<br />

<br />

Over the face OABC , z = 0, dz = 0, ˆn = – ˆk , ds = dx dy<br />

<br />

<br />

1 1 2<br />

F. nds ˆ (– y ˆj) (– kˆ<br />

) dxdy 0<br />

<br />

0 0<br />

Over the face BCDE, y = 1, dy = 0


Vectors 465<br />

<br />

<br />

Fnds ˆ =<br />

1 1<br />

<br />

0 0<br />

(4 xziˆ ˆj zkˆ<br />

) ( ˆj)<br />

dx dz<br />

ˆn = ˆ, j ds dxdz =<br />

=<br />

1 1<br />

dx dz =<br />

0 0<br />

1 1<br />

<br />

0 0<br />

dxdz<br />

1 1<br />

0 z 0<br />

( x) ( ) = – (1) (1) = – 1<br />

Over the face DEFG, z = 1, dz = 0, ˆn = ˆk , ds = dx dy<br />

<br />

<br />

Fnds ˆ =<br />

=<br />

1 1 2<br />

<br />

0 0<br />

[4 x (1) y ˆj y(1) kˆ]()<br />

kˆ<br />

dx dy<br />

1 1<br />

ydxdy =<br />

0 0<br />

1 1<br />

dx ydy = ( x)<br />

0 0<br />

Over the face OCDG, x = 0, dx = 0, ˆn = – iˆ,<br />

<br />

<br />

1<br />

0<br />

ds = dy dz<br />

Fnds ˆ = 1 1 ˆ 2 ˆ ˆ ˆ<br />

(0 i y j yzk ) ( i ) dydz = 0<br />

0 0<br />

Over the face AOGF, y = 0, dy = 0, ˆn = – ĵ ,<br />

<br />

<br />

Fnds ˆ =<br />

1 1<br />

(4 ˆ) ( ˆ<br />

xzi j)<br />

dxdz = 0<br />

0 0<br />

Over the face ABEF, x = 1, dx = 0, ˆn = î ,<br />

<br />

<br />

Fnds ˆ =<br />

1 1 2<br />

0 0<br />

[(4 zi ˆ y ˆ j yzk ˆ)()]<br />

i ˆ dydz<br />

=<br />

1 dy<br />

1<br />

4<br />

0 0<br />

1 2 1<br />

dy (2 z )<br />

0<br />

0<br />

1<br />

<br />

2<br />

y <br />

<br />

2 <br />

<br />

0<br />

ds = dx dz<br />

ds = dy dz<br />

1<br />

0<br />

= 1 2<br />

1 1<br />

4 zdydz<br />

0 0<br />

= zdz = = 2 dy = 2( y) 2<br />

On adding we see that over the whole surface<br />

Fnds<br />

1 <br />

ˆ = 01 0 0<br />

2<br />

2 = 3 2<br />

From (1) and (2), we have<br />

1. Use Divergence Theorem to evaluate<br />

V<br />

<br />

Fdv<br />

Fnds<br />

<br />

<br />

S<br />

<br />

= <br />

ˆ<br />

EXERCISE 5.15<br />

<br />

2 2 2 2 2<br />

( y z i ˆ z x ˆ j x y 2ˆ k ). ds ,<br />

s<br />

1<br />

0<br />

...(2)<br />

Verified.<br />

where S is the upper part of the sphere x 2 + y 2 + z 2 = 9 above xy- plane. Ans. 243 <br />

8<br />

<br />

<br />

2. Evaluate ( F). ds,<br />

where S is the surface of the paraboloid x 2 + y 2 + z = 4 above the xy-plane and<br />

<br />

S<br />

2 ˆ ˆ<br />

2<br />

F ( x y 4) i 3 xyj (2 xz z ) kˆ<br />

.<br />

Ans. – 4 <br />

<br />

2 2 3 2<br />

3. Evaluate [ xz dy dz ( x y z ) dzdx (2 xy y z) dxdy],<br />

where S is the surface enclosing a<br />

s<br />

region bounded by hemisphere x 2 + y 2 + z 2 = 4 above XY-plane.<br />

4. Verify Divergence Theorem for 2<br />

F x iˆ zj ˆ yzkˆ<br />

, taken over the cube bounded by<br />

x = 0, x = 1, y = 0, y = 1, z = 0, z = 1.<br />

2<br />

5. Evaluate (2 ˆ ˆ ˆ<br />

xyi yz j xz k)<br />

ds <br />

over the surface of the region bounded by<br />

S<br />

x = 0, y = 0, y = 3, z = 0 and x + 2 z = 6 Ans. 351<br />

2


466 Vectors<br />

6.<br />

<br />

Verify Divergence Theorem for 2<br />

F ( x y ) iˆ<br />

– 2 xj ˆ 2 yzkˆ<br />

and the volume of a tetrahedron bounded<br />

by co-ordinate planes and the plane 2 x + y + 2 z = 6.<br />

(Nagpur, Winter 2000, A.M.I.E.T.E.. Winter 2000)<br />

7. Verify Divergence Theorem for the function<br />

x 2 + y 2 = 9, z = 0 and z = 2.<br />

8. Use the Divergence Theorem to evaluate<br />

<br />

s<br />

ˆ ˆ 2 ˆ over the region bounded by<br />

F yi xj z k<br />

3 2 2<br />

x dydz x ydzdx x zdxdy,<br />

where S is the surface of the region bounded by the closed cylinder<br />

x 2 + y 2 = a 2 , (0 z b) and z = 0, z = b.<br />

<br />

Ans.<br />

4<br />

5 ab<br />

4<br />

2<br />

9. Evaluate the integral ( z x) dy dz xy dx dz 3 zdxdy,<br />

where S is the surface of closed region<br />

s<br />

bounded by z = 4 – y 2 and planes x = 0, x = 3, z = 0 by transforming it with the help of Divergence<br />

Theorem to a triple integral. Ans. 16<br />

10. Evaluate<br />

<br />

s<br />

ds<br />

2 2 2 2 2 2<br />

a x b y c z<br />

over the closed surface of the ellipsoid ax 2 + by 2 + cz 2 = 1 by<br />

applying Divergence Theorem. Ans.<br />

11. Apply Divergence Theorem to evaluate 2 2 2<br />

( lx my nz ) ds<br />

<br />

4<br />

( abc)<br />

taken over the sphere (x – a) 2 + (y – b) 2 + (z – c) 2 = r 2 , l, m, n being the direction cosines of the external<br />

normal to the sphere. (AMIETE June 2010, 2009) Ans.<br />

8 ( )<br />

3<br />

3 a b c r<br />

12. Show that ( uV u<br />

V ) dv<br />

V<br />

<br />

<br />

= .<br />

s<br />

<br />

uV ds<br />

13. If E = grad and 2<br />

= 4 , prove that E <br />

n ds = 4 dv<br />

S<br />

V<br />

where n is the outward unit normal <strong>vector</strong>, while dS and dV are respectively surface and volume<br />

elements.<br />

Pick up the correct option from the following:<br />

14. If F <br />

is the velocity of a fluid particle then F.<br />

dr represents.<br />

(a) Work done (b) Circulation<br />

C<br />

(c) Flux (d) Conservative field.<br />

(U.P. Ist Semester, Dec 2009) Ans. (b)<br />

15. If f = ax i by j cz k , a, b, c, constants, then f.<br />

dS where S is the surface of a unit sphere is<br />

<br />

(a) ( )<br />

3 a b c (b) 4 ( a b c)<br />

(c) 2 ( a b c)<br />

(d) (a + b + c)<br />

3<br />

(U.P., Ist Semester, 2009) Ans. (b)<br />

16. A force field F is said to be conservative if<br />

<br />

(a) Curl F 0 (b) grad F 0 (c) Div F 0 (d) Curl (grad F ) = 0<br />

(AMIETE, Dec. 2006) Ans. (a)<br />

17. The line integral<br />

<br />

<br />

2 2<br />

x dx y dy, where C is the boundary of the region x 2 + y 2 < a 2 equals<br />

c<br />

(a) 0, (b) a (c) a 2 1 2<br />

(d) a<br />

2<br />

(AMIETE, Dec. 2006) Ans. (b)

Hooray! Your file is uploaded and ready to be published.

Saved successfully!

Ooh no, something went wrong!